AeroElectric-Archive.digest.vol-qb

January 19, 2021 - March 01, 2021



      
      On Jan 19, 2021, at 7:24 PM, Charlie England  
      wrote:
      
      On 1/19/2021 5:23 PM, David and Elaine Lamphere wrote:
      > sorry for the cutting error
      > 
      > On Jan 19, 2021, at 4:43 PM, Charlie England > wrote:
      > What does it do if you leave the field leads disconnected, start the 
      motor, and then connect the field?
      > 
      > ANSWER:
      > It also starts charging and buss is 14V!
      > 
      > Any theories?
      I think the interwebs have caught the Covid. ;-)
      I sent the following a couple of hours ago, but it isn't showing up in 
      the AEC web view.
      >>
      
      No theories, but you might want to let PP know your results, and ask 
      them. Detail the hookup, and be specific about it failing to come on 
      line if the 'field' (actually a 'turn-on' signal terminal) is held at 
      battery voltage prior to spin-up, and that it does come on line if the 
      control terminal is left floating until after spin-up.
      
      My personal solution is to spend about 20% of the money (on an 
      off-the-shelf IR automotive alternator and an overvoltage module/relay). 
      ;-)
      
      One theory that just occurred to me (purely speculative) is the 
      possibility that since the alt has an automotive 'heritage', the 
      internal control logic could be set up to only allow the alternator to 
      come on line if the control terminal goes 'hi' (on) *after* the 
      alternator is spinning. Some automotive applications keep the control 
      terminal in 'lo' state until after the ignition switch goes from 'start' 
      to 'run'; the alt's control logic may be 'enforcing' that.
      
      Have you tried connecting everything, spinning up, then opening/closing 
      the control line?
      
      Charlie
      
      
________________________________________________________________________________
Date: Jan 19, 2021
From: "Robert L. Nuckolls, III" <nuckolls.bob(at)aeroelectric.com>
Subject: Re: Alternator behavior
>>If I shut off the motor and do not disconnect anything and start >>the motor again - it does not charge! >> >>Why? >> >>And how on earth am I going to get this to work in the airplane?? >What does it do if you leave the field leads disconnected, start the >motor, and then connect the field? Agreed. It doesn't operate in the airplane like you're testing it on the bench. The internal regulator is fitted with certain 'features' that don't exist in legacy regulators popular in aircraft. The thing MIGHT be designed to keep the alternator dormant if field control and motive power come up at the same time. Modern engine control modules wait until the engine is stable after cranking before adding alternator loads. Bob . . . Un impeachable logic: George Carlin asked, "If black boxes survive crashes, why don't they make the whole airplane out of that stuff?" ________________________________________________________________________________
Date: Jan 19, 2021
From: "Robert L. Nuckolls, III" <nuckolls.bob(at)aeroelectric.com>
Subject: Re: Alternator behavior
>I suppose I could get in the habit of starting >the engine in the plane with the main power >switch set to battery, then turn on the field >power - but that=99s something that just >doesn=99t seem right to me. Never had to do that >with the PP alternator I have had in the >Tailwind for the past 12 years. Still, that >might be an angle to get some more phase 1 >flying in and qualify as in the aircraft (for PP troubleshooting). > >By the control line do you mean the wire >powering the field? I=99ll try that in the morning. > >Thanks for your help. The PP alternator of a decade ago may have a completely different control philosophy. I've always left the alternator OFF until the engine is running smoothly. Works good. Bob . . . Un impeachable logic: George Carlin asked, "If black boxes survive crashes, why don't they make the whole airplane out of that stuff?" ________________________________________________________________________________
Date: Jan 19, 2021
From: "Robert L. Nuckolls, III" <nuckolls.bob(at)aeroelectric.com>
Subject: Re: Power distribution parts
At 11:53 AM 1/19/2021, you wrote: > > >I tried to distribute the parts equitably. > >Paul in Pennsylvania got a 10-fuse block, I got a 24-tab ground >block, and Sebastien in Canada got two ground blocks and three fuse >blocks. The 4-fuse block is orphaned but warm and dry. Good show! Bob . . . Un impeachable logic: George Carlin asked, "If black boxes survive crashes, why don't they make the whole airplane out of that stuff?" ________________________________________________________________________________
Subject: Re: Alternator behavior
From: Charlie England <ceengland7(at)gmail.com>
Date: Jan 19, 2021
Isn't the PP alt internally regulated? Assuming it is, then the 'field' terminal isn't actually a field terminal; it's simply a control terminal. A true field terminal is tied directly to one end of the field coil in the alternator; not the case with a PP or other internally regulated alternator. When I said to call PP, I meant to tell them what you're now seeing. Install it in the plane (to satisfy PP), crank with the alt switch off, turn the switch on, and see if it starts charging. Assuming it does, explain the process you went through to PP & ask them if the behavior is 'normal'. You may just be experiencing PP's 'failure to communicate' how the alt is supposed to act. If they tell you that it's 'normal', then I guess that unfortunately, you get to decide whether to live with it, or pick a different product. Good luck, Charlie On 1/19/2021 8:20 PM, David and Elaine Lamphere wrote: > Charlie, > > This alternator just came back from Hartzell/PlanePower. *They told me > it passed their tests and denied warranty replacement.* > > I informed them of my intent to bench test before putting it back in > the plane. > The engineer I was communicating with implied they could only help > with aircraft installation troubleshooting. > I have sent them the same information I have posted and as of yet not > heard back from them. > > I suppose I could get in the habit of starting the engine in the plane > with the main power switch set to battery, then turn on the field > power - but thats something that just doesnt seem right to me. Never > had to do that with the PP alternator I have had in the Tailwind for > the past 12 years. Still, that might be an angle to get some more > phase 1 flying in and qualify as in the aircraft (for PP troubleshooting). > By the control line do you mean the wire powering the field? Ill try > that in the morning. > > Thanks for your help. > > Dave > > On Jan 19, 2021, at 7:24 PM, Charlie England > wrote: > > On 1/19/2021 5:23 PM, David and Elaine Lamphere wrote: >> sorry for the cutting error >> >> On Jan 19, 2021, at 4:43 PM, Charlie England > > wrote: >> What does it do if you leave the field leads disconnected, start the >> motor, and then connect the field? >> >> ANSWER: >> It also starts charging and buss is 14V! >> >> Any theories? > I think the interwebs have caught the Covid. ;-) > I sent the following a couple of hours ago, but it isn't showing up in > the AEC web view. > >> > > No theories, but you might want to let PP know your results, and ask > them. Detail the hookup, and be specific about it failing to come on > line if the 'field' (actually a 'turn-on' signal terminal) is held at > battery voltage prior to spin-up, and that it does come on line if the > control terminal is left floating until after spin-up. > > My personal solution is to spend about 20% of the money (on an > off-the-shelf IR automotive alternator and an overvoltage > module/relay). ;-) > > One theory that just occurred to me (*purely speculative*) is the > possibility that since the alt has an automotive 'heritage', the > internal control logic could be set up to only allow the alternator to > come on line if the control terminal goes 'hi' (on) *after* the > alternator is spinning. Some automotive applications keep the control > terminal in 'lo' state until after the ignition switch goes from > 'start' to 'run'; the alt's control logic may be 'enforcing' that. > > Have you tried connecting everything, spinning up, then > opening/closing the control line? > > Charlie > -- This email has been checked for viruses by Avast antivirus software. https://www.avast.com/antivirus ________________________________________________________________________________
From: David and Elaine Lamphere <dalamphere(at)comcast.net>
Subject: Re: Alternator behavior
Date: Jan 20, 2021
Now that is interesting! I wonder if anyone else with new PP alternators has to do the same thing? Anyway, thank you for the information! Dave On Jan 19, 2021, at 10:14 PM, Robert L. Nuckolls, III wrote: > I suppose I could get in the habit of starting the engine in the plane with the main power switch set to battery, then turn on the field power - but that=99s something that just doesn=99t seem right to me. Never had to do that with the PP alternator I have had in the Tailwind for the past 12 years. Still, that might be an angle to get some more phase 1 flying in and qualify as in the aircraft (for PP troubleshooting). > > By the control line do you mean the wire powering the field? I=99l l try that in the morning. > > Thanks for your help. The PP alternator of a decade ago may have a completely different control philosophy. I've always left the alternator OFF until the engine is running smoothly. Works good. Bob . . . Un impeachable logic: George Carlin asked, "If black boxes survive crashes, why don't they make the whole airplane out of that stuff?" ________________________________________________________________________________
From: David and Elaine Lamphere <dalamphere(at)comcast.net>
Subject: Re: Alternator behavior
Date: Jan 20, 2021
Charlie, I ran the test that you suggested (all connections, start motor, disconnect and reconnect field) this morning. Sure enough, it started charging after the reconnection. Having the driveway graveled this morning. When I can get to the airport next, I=99ll reinstall the alternator and go from there. Thanks again. Dave On Jan 19, 2021, at 10:34 PM, Charlie England wrote: Isn't the PP alt internally regulated? Assuming it is, then the 'field' terminal isn't actually a field terminal; it's simply a control terminal. A true field terminal is tied directly to one end of the field coil in the alternator; not the case with a PP or other internally regulated alternator. When I said to call PP, I meant to tell them what you're now seeing. Install it in the plane (to satisfy PP), crank with the alt switch off, turn the switch on, and see if it starts charging. Assuming it does, explain the process you went through to PP & ask them if the behavior is 'normal'. You may just be experiencing PP's 'failure to communicate' how the alt is supposed to act. If they tell you that it's 'normal', then I guess that unfortunately, you get to decide whether to live with it, or pick a different product. Good luck, Charlie On 1/19/2021 8:20 PM, David and Elaine Lamphere wrote: > Charlie, > > This alternator just came back from Hartzell/PlanePower. They told me it passed their tests and denied warranty replacement. > > I informed them of my intent to bench test before putting it back in the plane. > The engineer I was communicating with implied they could only help with aircraft installation troubleshooting. > I have sent them the same information I have posted and as of yet not heard back from them. > > I suppose I could get in the habit of starting the engine in the plane with the main power switch set to battery, then turn on the field power - but that=99s something that just doesn=99t seem right to me. Never had to do that with the PP alternator I have had in the Tailwind for the past 12 years. Still, that might be an angle to get some more phase 1 flying in and qualify as in the aircraft (for PP troubleshooting). > > By the control line do you mean the wire powering the field? I=99l l try that in the morning. > > Thanks for your help. > > Dave > > On Jan 19, 2021, at 7:24 PM, Charlie England > wrote: > > On 1/19/2021 5:23 PM, David and Elaine Lamphere wrote: >> sorry for the cutting error >> >> On Jan 19, 2021, at 4:43 PM, Charlie England > wrote: >> What does it do if you leave the field leads disconnected, start the motor, and then connect the field? >> >> ANSWER: >> It also starts charging and buss is 14V! >> >> Any theories? > I think the interwebs have caught the Covid. ;-) > I sent the following a couple of hours ago, but it isn't showing up in the AEC web view. > >> > > No theories, but you might want to let PP know your results, and ask them. Detail the hookup, and be specific about it failing to come on line if the 'field' (actually a 'turn-on' signal terminal) is held at battery voltage prior to spin-up, and that it does come on line if the control terminal is left floating until after spin-up. > > My personal solution is to spend about 20% of the money (on an off-the-shelf IR automotive alternator and an overvoltage module/relay). ;-) > > One theory that just occurred to me (purely speculative) is the possibility that since the alt has an automotive 'heritage', the internal control logic could be set up to only allow the alternator to come on line if the control terminal goes 'hi' (on) *after* the alternator is spinning. Some automotive applications keep the control terminal in 'lo' state until after the ignition switch goes from 'start' to 'run'; the alt's control logic may be 'enforcing' that. > > Have you tried connecting everything, spinning up, then opening/closing the control line? > > Charlie > ________________________________________________________________________________
Subject: Z101B Aux Alternator and Overvoltage protection
From: "Patrick Nelson" <panelsonms(at)hotmail.com>
Date: Jan 20, 2021
For those with Z101B, what is your procedure for using the aux alternator? My initial thought was to turn on the aux alternator with the main alternator and use a lower voltage setpoint for the aux regulator. This allows the aux alternator to take over without intervention if the main alternator fails. This causes a different problem, however. The main regulator could fail resulting in an overvoltage condition. The crowbar overvoltage protection circuits would blow the aux alt fuse and trip the main field circuit breaker. Z101B shows the aux alt fuse forward of the firewall so it can't be replaced in flight. Essentially, an overvoltage event on the main regulator will take out both alternators. The solutions I can think of are: * Enable the aux alternator only in case of main alternator failure. * Move the aux field breaker/fuse to the main bus so it can be in the cabin, but this removes some fault tolerance of the design. * Come up with an arrangement for the aux regulator that allows a slightly longer period before the overvoltage protection activates. This seems like a fragile solution though. Note: My aux alternator is a BC410 with an LR3C regulator. Interested in hearing people's procedures for the aux alt switch and/or if they've deviated from the Z101B wiring diagram to solve this problem Thanks! -Patrick Read this topic online here: http://forums.matronics.com/viewtopic.php?p=500358#500358 ________________________________________________________________________________
Subject: Re: Z101B Aux Alternator and Overvoltage protection
From: Charlie England <ceengland7(at)gmail.com>
Date: Jan 20, 2021
On 1/20/2021 8:36 PM, Patrick Nelson wrote: > > For those with Z101B, what is your procedure for using the aux alternator? My initial thought was to turn on the aux alternator with the main alternator and use a lower voltage setpoint for the aux regulator. This allows the aux alternator to take over without intervention if the main alternator fails. > > This causes a different problem, however. The main regulator could fail resulting in an overvoltage condition. The crowbar overvoltage protection circuits would blow the aux alt fuse and trip the main field circuit breaker. Z101B shows the aux alt fuse forward of the firewall so it can't be replaced in flight. Essentially, an overvoltage event on the main regulator will take out both alternators. > > The solutions I can think of are: > * Enable the aux alternator only in case of main alternator failure. > * Move the aux field breaker/fuse to the main bus so it can be in the cabin, but this removes some fault tolerance of the design. > * Come up with an arrangement for the aux regulator that allows a slightly longer period before the overvoltage protection activates. This seems like a fragile solution though. > > Note: My aux alternator is a BC410 with an LR3C regulator. > > Interested in hearing people's procedures for the aux alt switch and/or if they've deviated from the Z101B wiring diagram to solve this problem > Thanks! > -Patrick IIRC, the B&C regulator that's made for the backup alt is supposed to ignore an OV event caused by the main alt, and not react to OV except from its own alt. Protection for the B-lead would be the same whether the fuse (or breaker) is on the fat wire tie point, or on a bus. The fuse/CB is protecting the wire from the battery; not the alternator. Charlie -- This email has been checked for viruses by Avast antivirus software. https://www.avast.com/antivirus ________________________________________________________________________________
Subject: Re: Z101B Aux Alternator and Overvoltage protection
From: "johnbright" <john_s_bright(at)yahoo.com>
Date: Jan 20, 2021
Here's a Bob quote from 5 Dec, 2019: "If you're using B&C alternator controllers in Z-12, the ov control systems are 'selective trip' . . . when an ov condition is detected, each ov system looks to see if 'my alternator' has field voltage on it. If so, then the 'my regulator' is bad and I trip only 'my alternator'. If the field voltage is low or zero, the ov condition is generated by failure of the 'other regulator' it is expected to manage it's own shutdown operation." -------- John Bright, RV-6A, at FWF, O-360 Z-101 single batt dual alt SDS EM-5-F. john_s_bright(at)yahoo.com, Newport News, Va https://docs.google.com/document/d/1YOtPiA3AdUsQEYR4nodBESNAo21rxdnx4pFs7VxXfuI/edit?usp=sharing Read this topic online here: http://forums.matronics.com/viewtopic.php?p=500360#500360 ________________________________________________________________________________
Subject: Re: Z101B Aux Alternator and Overvoltage protection
From: "Patrick Nelson" <panelsonms(at)hotmail.com>
Date: Jan 20, 2021
Ah, thanks! That makes sense. If the voltage is over the set point then the field should have no voltage. Presumably LR-3 OV protection kicks in if voltage at pin 3 exceeds 16V and there is voltage present on pin 4 (field winding). That's easy enough to test next time I'm working on wiring. Thank you for the info. -Patrick Read this topic online here: http://forums.matronics.com/viewtopic.php?p=500361#500361 ________________________________________________________________________________
Date: Jan 21, 2021
From: "Robert L. Nuckolls, III" <nuckolls.bob(at)aeroelectric.com>
Subject: Re: Z101B Aux Alternator and Overvoltage protection
At 08:36 PM 1/20/2021, you wrote: > > >For those with Z101B, what is your procedure for using the aux >alternator? My initial thought was to turn on the aux alternator >with the main alternator and use a lower voltage setpoint for the >aux regulator. This allows the aux alternator to take over without >intervention if the main alternator fails. Not recommended. There's no good argument for an automatic changeover. That feature was incorporated into the B&C LS series regulators in anticipation of STCs onto tC aircraft. In this case, automatic changeover WITH annunciation and LOAD shedding advice was attractive to the spam can crowd. Main alternator failure is NOT an event requiring split second detection and action. When the low volts light comes on you can finish your cup of coffee, fold up the map, then turn on the aux alternator and turn off the main alternator field supply . . . load shed in accordance with your plan-B protocols . . . then pour another cup of coffee. Automating any part of this increases complexity and truncates constellation of regulators suited to task (Z101 as published can use ANY regulator/alternator combo). >This causes a different problem, however. The main regulator could >fail resulting in an overvoltage condition. The crowbar overvoltage >protection circuits would blow the aux alt fuse and trip the main >field circuit breaker. Z101B shows the aux alt fuse forward of the >firewall so it can't be replaced in flight. Essentially, an >overvoltage event on the main regulator will take out both alternators. > >The solutions I can think of are: >* Enable the aux alternator only in case of main alternator failure. >* Move the aux field breaker/fuse to the main bus so it can be in >the cabin, but this removes some fault tolerance of the design. >* Come up with an arrangement for the aux regulator that allows a >slightly longer period before the overvoltage protection >activates. This seems like a fragile solution though. > >Note: My aux alternator is a BC410 with an LR3C regulator. > >Interested in hearing people's procedures for the aux alt switch >and/or if they've deviated from the Z101B wiring diagram to solve this problem It's not a 'problem' . . . as you've correctly deduced, the aux alternator of any style and companion voltage regulator can be left in standby readiness with a low probability of ever being needed. I wouldn't even put ov protection on the aux alternator. It's pre-flight tested and known to be functioning properly. It's only in service for a most . . . a few hours to comfortably terminate a flight with main alternator down. The changes of have a dual failure (main alt + aux regulator) during any given tank full of fuel is essentially zero. In the well maintained airplane, the aux alternator should have no more than a few hours service over the lifetime of the airplane. It's primary purpose is to REDUCE the size of battery you need to carry to meet endurance goals. Keep it simple. Write your own plan-b procedure for aux alternator failure. Note that Z101 manages the WHOLE system on three switches . . . switches unlikely to be mis-positioned in a way that increases risk. Bob . . . Un impeachable logic: George Carlin asked, "If black boxes survive crashes, why don't they make the whole airplane out of that stuff?" ________________________________________________________________________________
Date: Jan 21, 2021
From: "Robert L. Nuckolls, III" <nuckolls.bob(at)aeroelectric.com>
Subject: Re: Z101B Aux Alternator and Overvoltage protection
>IIRC, the B&C regulator that's made for the >backup alt is supposed to ignore an OV event >caused by the main alt, and not react to OV except from its own alt. > >Protection for the B-lead would be the same >whether the fuse (or breaker) is on the fat wire >tie point, or on a bus. The fuse/CB=C2 is >protecting the wire from the battery; not the alternator. Correct . . . but I perceive no good reason for adding the complexity of a full-up alternator controller to the aux alternator system. Bob . . . Un impeachable logic: George Carlin asked, "If black boxes survive crashes, why don't they make the whole airplane out of that stuff?" ________________________________________________________________________________
Subject: Re: Zeftronics Regulator Connection Question
From: "farmrjohn" <faithvineyard(at)yahoo.com>
Date: Jan 22, 2021
Based on the lack of replies I'll go with "if it ain't broke, don't fix it" as it seems to be working OK. John Read this topic online here: http://forums.matronics.com/viewtopic.php?p=500380#500380 ________________________________________________________________________________
Date: Jan 22, 2021
From: "Robert L. Nuckolls, III" <nuckolls.bob(at)aeroelectric.com>
Subject: Re: Zeftronics Regulator Connection Question
At 05:51 PM 1/22/2021, you wrote: I have a Zeftronics R15100 voltage regulator that has the following terminals: I-A-S-F. Only three terminals have wires connected to them, I-A-F. For low or high voltage indication it looks like an Electronics International VA-1 voltmeter is used. Everything seems to be working (system shows 14.1 volts when engine running, alternator on), should I move the I lead to the S terminal? Man! You got me! That regulator is FAA-PMA replacement for the stock 'ford' regulators common to single engine, 14v Cessnas for probably over a decade. That regulator is featured in my Z-Figures and written about in the 'Connection'. Attached is a page excerpted from a Cessna maintenance manual on single engine airplanes. Clearly, the wiring depicted is consistent with our understanding of how that regulator works. Why your regulator appears to be working is a mystery. It's sound logic to not mess with a working system . . . but . . . I would sure like to understand why it is working. Having said that, we also know that while certain after-market, plug-n-play 'ford' regulators are not form-fit-and-function in a Cessna. I tried to replace a failed regulator in a C-172XP rental and was treated with an overvoltage condition when I turned the alternator on. I don't recall details of the variation but clearly, risks for differences were not zero. This Zeftronics device includes extra features (diagnostics) not part of the generic 'ford' regulator. https://tinyurl.com/y6le6gdo There may be other nuances not obvious to us . . . but I'm mystified as to how/why your system appears to be functional. Perhaps a call to Zeftronics? Bob . . . Un impeachable logic: George Carlin asked, "If black boxes survive crashes, why don't they make the whole airplane out of that stuff?" ________________________________________________________________________________
Subject: Re: Antenna Ground Planes
From: "AdventureD" <dobrien(at)microfoundations.com>
Date: Jan 22, 2021
Thank you Bob, Joe, all, for all this advice. Happy and safe flying, Dan Read this topic online here: http://forums.matronics.com/viewtopic.php?p=500383#500383 ________________________________________________________________________________
Date: Jan 23, 2021
From: "Robert L. Nuckolls, III" <nuckolls.bob(at)aeroelectric.com>
Subject: Re: Zeftronics Regulator Connection Question
I've updated the drawing that was attached to my last reply to this thread. Interested readers can download at: https://tinyurl.com/yxg6hfrj There are additional nuances to the Cessna alternator incorporation philosophy that I may add later on a second page to this drawing . . . will give a heads-up when posted. Bob . . . Un impeachable logic: George Carlin asked, "If black boxes survive crashes, why don't they make the whole airplane out of that stuff?" ________________________________________________________________________________
Subject: Re: Zeftronics Regulator Connection Question
From: "racerjerry" <gnking2(at)verizon.net>
Date: Jan 23, 2021
Bob, Zeftronics offers an excellent troubleshooting guide for their voltage regulators online. The included diagrams MAY help to clear up some of the mystery; unfortunately, it's still a bit opaque for me. And yes, my guess is "If it ain't broke - Don't fix it" still applies. https://www.zeftronics.com/wp-content/uploads/2014/06/Z13PS.pdf -------- Jerry King Read this topic online here: http://forums.matronics.com/viewtopic.php?p=500385#500385 ________________________________________________________________________________
Subject: Small battery choice
From: "AdventureD" <dobrien(at)microfoundations.com>
Date: Jan 23, 2021
Ordering two batteries for my ship (Z14, rear bats). Some years ago, the good doctor recommended three: > A few exemplar brands and part numbers are: > Panasonic LC-RD1217 > Odyssey PC680 > Power Sonic PSH-12180FR The Power Sonic is cheapest (only $60! ... quite a schoke after spending $600 on my Mooney batteries!) and available at Amazon. I figure there is not a lot of difference between these, but wanted to check if anyone had reason to pick one over the other (or another in same roughly 7"-7"-3" profile). Cheers, Dan Read this topic online here: http://forums.matronics.com/viewtopic.php?p=500386#500386 ________________________________________________________________________________
Subject: Re: Small battery choice
From: C&K <yellowduckduo(at)gmail.com>
Date: Jan 23, 2021
I've had very good results from the Dekka power sports batteries for many years. The last ones I bought were ordered for pickup at Lowes. American made. Ken On 23/01/2021 3:14 PM, AdventureD wrote: > > Ordering two batteries for my ship (Z14, rear bats). Some years ago, the good doctor recommended three: > > >> A few exemplar brands and part numbers are: >> Panasonic LC-RD1217 >> Odyssey PC680 >> Power Sonic PSH-12180FR > > The Power Sonic is cheapest (only $60! ... quite a schoke after spending $600 on my Mooney batteries!) and available at Amazon. I figure there is not a lot of difference between these, but wanted to check if anyone had reason to pick one over the other (or another in same roughly 7"-7"-3" profile). > > Cheers, > Dan > > > Read this topic online here: > > http://forums.matronics.com/viewtopic.php?p=500386#500386 > > ________________________________________________________________________________
Subject: Re: Zeftronics Regulator Connection Question
From: "farmrjohn" <faithvineyard(at)yahoo.com>
Date: Jan 23, 2021
This installation is not in a Cessna but on an Aircoupe with an O200 engine wtih alternator vs. the C90 it came with. It also does not have the over voltage sensor installed, rather utilizes the EI voltage meter for warning. It looks like the middle diagram on page 3 of the Zeftronics trouble shooting guide would show what happens and how it works if the alternator control switch is connected direct to the I terminal vs. the S terminal if I'm interpreting it correctly. John Read this topic online here: http://forums.matronics.com/viewtopic.php?p=500389#500389 ________________________________________________________________________________
Subject: Re: Stratux RS-232 Wx+Trfc Out to various navigators...
From: "farmrjohn" <faithvineyard(at)yahoo.com>
Date: Jan 23, 2021
Instead if a USB port for the RS232 out would it be possible to utilize the ethernet port with something like: https://tinyurl.com/yyqm2fgz Read this topic online here: http://forums.matronics.com/viewtopic.php?p=500390#500390 ________________________________________________________________________________
From: Charlie England <ceengland7(at)gmail.com>
Date: Jan 23, 2021
Subject: Re: Small battery choice
On Sat, Jan 23, 2021 at 2:20 PM AdventureD wrote: > dobrien(at)microfoundations.com> > > Ordering two batteries for my ship (Z14, rear bats). Some years ago, the > good doctor recommended three: > > > > A few exemplar brands and part numbers are: > > Panasonic LC-RD1217 > > Odyssey PC680 > > Power Sonic PSH-12180FR > > > The Power Sonic is cheapest (only $60! ... quite a schoke after spending > $600 on my Mooney batteries!) and available at Amazon. I figure there is > not a lot of difference between these, but wanted to check if anyone had > reason to pick one over the other (or another in same roughly 7"-7"-3" > profile). > > Cheers, > Dan > Hi Dan, I've run Power Sonic, and at least 2 or 3 other 'no name' brands in a/c over the past 20+ years (probably everything *except* Odyssey), and I've never been able to detect any 'real world' difference among the brands. I buy the cheapest version I can find at the time, and it 'just works'. FWIW, there's been some internet chatter lately among the RV-x drivers that the current (pardon the pun) crop of Odysseys have had less than stellar lifespans (and they obviously extract a premium from your pocketbook). I've always gotten 4-6 years out of the no-name versions, with the only exception being one that had a pin hole in the case (shipping damage that I missed prior to installation). Even it lasted about a year before I noticed diminished capacity. FWIW, there are some higher AH versions available in almost identical case sizes. The naming convention for most is like the PowerSonic: 12(voltage)18(AH)followed by another zero. If you search for '12200 SLA battery' or '12220 SLA battery' (and read the specs carefully), you can get a 20 AH or 22AH that will fit in the same space, for about the same money. One thing to notice on the 'no names' is that many are intended for deep discharge applications (powered wheelchairs, etc), and will have slightly higher internal resistance than a PC680 'starting battery'. By going to a 20 AH or 22AH version, the internal impedance will be roughly the same as the PC680 (meaning same or better CCA), and the battery's *capacity* (total energy content) will be 10%-15% greater than a PC680 (stuff will run longer after an alternator failure). My experience since late in the last century; YMMV, etc etc. Charlie ________________________________________________________________________________
Subject: Re: Small battery choice
From: "AdventureD" <dobrien(at)microfoundations.com>
Date: Jan 23, 2021
A bit more on this -- with the redundancy I have in the Z14 system, I am not looking for a high $$ battery. I am happy to change one out every year IF the batteries do a good job cranking the engine. I have a high compression IO550, and the ability to close the cross feed to crank. How can I tell from the data sheets whether these batteries either alone or in combination are up to the task? The Power Sonic datasheet reports a max of 270 amps for 5 seconds. There seems to be all kinds of different reports online about what is required to start an engine. I don't want to get batters that aren't up to the task. Any thoughts appreciated. Dan Read this topic online here: http://forums.matronics.com/viewtopic.php?p=500392#500392 ________________________________________________________________________________
Subject: Re: Small battery choice
From: "AdventureD" <dobrien(at)microfoundations.com>
Date: Jan 23, 2021
Charlie, what engine are you cranking? Best, Dan Read this topic online here: http://forums.matronics.com/viewtopic.php?p=500393#500393 ________________________________________________________________________________
Subject: Re: Small battery choice
From: Kelly McMullen <kellym(at)aviating.com>
Date: Jan 23, 2021
Certainly not my experience. In aircraft, in Aridzona the typical Concorde or Gill is good for 3-3.5 years. I have an Odyssey in my homebuilt...a bigger 925 which has served well for 6 years. My tug uses batteries in the size you are talking about. The manufacturer put in Duracells. I tried generic and got 4 months. Paid twice as much for the Duracells, and they are doing quite well. On 1/23/2021 3:45 PM, Charlie England wrote: > > > On Sat, Jan 23, 2021 at 2:20 PM AdventureD > wrote: > > > Hi Dan, > > I've run Power Sonic, and at least 2 or 3 other 'no name' brands in a/c > overthe past 20+ years (probably everything *except* Odyssey), and I've > never been able to detect any 'real world' difference among the brands. > I buy the cheapest version I can find at the time, and it 'just works'. > FWIW, there's been some internet chatter lately among the RV-x drivers > that the current (pardon the pun) crop of Odysseyshave had less than > stellar lifespans (and they obviously extract a premium from your > pocketbook). I've always gotten 4-6 years out of the no-name versions, > with the only exception being one that had a pin holein the case > (shipping damage that I missed prior to installation). Even it lasted > about a year before I noticed diminished capacity. > > FWIW, there are some higher AH versions available in almost identical > case sizes. The naming convention for most is like the PowerSonic: > 12(voltage)18(AH)followed by another zero. If you search for '12200 SLA > battery' or '12220 SLA battery' (and read the specs carefully), you can > get a 20 AH or 22AH that will fit in the same space, for about the same > money. One thing to notice on the 'no names' is that many are intended > for deep discharge applications (powered wheelchairs, etc), and will > have slightly higher internal resistance than a PC680 'starting > battery'. By going to a 20 AH or 22AH version, the internal impedance > will be roughly the same as the PC680 (meaning same or better CCA), and > the battery's *capacity* (total energy content) will be 10%-15% greater > than a PC680 (stuff will run longer after an alternator failure). > > My experience since late in the last century; YMMV, etc etc. > > Charlie ________________________________________________________________________________
Subject: Re: Small battery choice
From: "user9253" <fransew(at)gmail.com>
Date: Jan 23, 2021
Compare the weight. Some batteries made in China claim to be 18 or 22 AH. But when you look at the weight, they weigh less than a PC680 (15.4 pounds). One deceiving trick is to use a larger case than necessary to hold the meager contents. -------- Joe Gores Read this topic online here: http://forums.matronics.com/viewtopic.php?p=500395#500395 ________________________________________________________________________________
Subject: Re: Small battery choice
From: Charlie England <ceengland7(at)gmail.com>
Date: Jan 23, 2021
On 1/23/2021 5:04 PM, AdventureD wrote: > > Charlie, what engine are you cranking? > > Best, > Dan > O-320 8.5-1 compression for most of that time. Joe makes a reasonable point for reality checks, but if you parallel a pair, just about anything you purchase would crank just about any engine you choose. Charlie -- This email has been checked for viruses by Avast antivirus software. https://www.avast.com/antivirus ________________________________________________________________________________
Subject: Re: Small battery choice
From: "user9253" <fransew(at)gmail.com>
Date: Jan 23, 2021
Here is another option: Mighty Max ML22-12GEL 12 Volt 22 AH Can be ordered from Home Depot or Lowes -------- Joe Gores Read this topic online here: http://forums.matronics.com/viewtopic.php?p=500398#500398 ________________________________________________________________________________
Subject: Re: Small battery choice
From: Charlie England <ceengland7(at)gmail.com>
Date: Jan 23, 2021
On 1/23/2021 9:23 PM, user9253 wrote: > > Here is another option: Mighty Max ML22-12GEL 12 Volt 22 AH > Can be ordered from Home Depot or Lowes > > -------- > Joe Gores That looks like a reasonable option to me. One measure for comparison (the easiest to find) is CCA (cold cranking amps). A harder one to find in docs is internal resistance (impedance). The PC680 is 7 milliohms (0.007). Most of the no-name stuff will be around 0.008-0.015. The MM referenced is 0.014, which is still plenty good enough to crank a 4 cyl Lyc, and would probably crank a 6 without too much effort. Odyssey https://www.odysseybattery.com/products/ods-agm16l-battery-pc680/ MightyMax https://smile.amazon.com/Mighty-Max-Battery-Brand-Product/dp/B01JNYYXII/ref=smi_www_rco2_go_smi_g4368549507?_encoding=UTF8&%2AVersion%2A=1&%2Aentries%2A=0&ie=UTF8 Charlie -- This email has been checked for viruses by Avast antivirus software. https://www.avast.com/antivirus ________________________________________________________________________________
Date: Jan 24, 2021
From: "Robert L. Nuckolls, III" <nuckolls.bob(at)aeroelectric.com>
Subject: Re: Small battery choice
At 04:59 PM 1/23/2021, you wrote: > > >A bit more on this -- with the redundancy I have in the Z14 system, >I am not looking for a high $$ battery. I am happy to change one >out every year IF the batteries do a good job cranking the >engine. I have a high compression IO550, and the ability to close >the cross feed to crank. How can I tell from the data sheets >whether these batteries either alone or in combination are up to the task? The data sheets are no help. Too many variables. Hands-on experience and observation are orders of the day. >The Power Sonic datasheet reports a max of 270 amps for 5 >seconds. There seems to be all kinds of different reports online >about what is required to start an engine. I don't want to get >batters that aren't up to the task. They will be fine for a time . . . the question is how long based on how YOU use them. The varied 'reports' are understandable for there are few constants. Hence observation/experience data will be all over the map. Give them a try and let us know what YOU observe. Bob . . . Un impeachable logic: George Carlin asked, "If black boxes survive crashes, why don't they make the whole airplane out of that stuff?" ________________________________________________________________________________
Date: Jan 24, 2021
From: "Robert L. Nuckolls, III" <nuckolls.bob(at)aeroelectric.com>
Subject: Re: Small battery choice
At 06:50 PM 1/23/2021, you wrote: > >Compare the weight. Some batteries made in China claim to be 18 or 22 AH. >But when you look at the weight, they weigh less than a PC680 (15.4 pounds). >One deceiving trick is to use a larger case than necessary to hold >the meager contents. Good point! Bob . . . Un impeachable logic: George Carlin asked, "If black boxes survive crashes, why don't they make the whole airplane out of that stuff?" ________________________________________________________________________________
Date: Jan 24, 2021
From: "Robert L. Nuckolls, III" <nuckolls.bob(at)aeroelectric.com>
Subject: Re: Small battery choice
At 09:23 PM 1/23/2021, you wrote: > >Here is another option: Mighty Max ML22-12GEL 12 Volt 22 AH >Can be ordered from Home Depot or Lowes > >-------- >Joe Gores Check to see if this is a true 'gel' cell fitted battery. These are generally optimized for deep cycle service like wheel chairs and trolling motors and not particularly good for engine cranking -OR- cold weather performance. AGM, SVLA is the way to go. Bob . . . Un impeachable logic: George Carlin asked, "If black boxes survive crashes, why don't they make the whole airplane out of that stuff?" ________________________________________________________________________________
Subject: Re: Small battery choice
From: Charlie England <ceengland7(at)gmail.com>
Date: Jan 24, 2021
On 1/24/2021 9:19 AM, Robert L. Nuckolls, III wrote: > At 09:23 PM 1/23/2021, you wrote: >> >> Here is another option: Mighty Max ML22-12GEL 12 Volt 22 AH >> Can be ordered from Home Depot or Lowes >> >> -------- >> Joe Gores > > Check to see if this is a true 'gel' cell > fitted battery. These are generally optimized > for deep cycle service like wheel chairs and > trolling motors and not particularly good > for engine cranking -OR- cold weather > performance. > > AGM, SVLA is the way to go. > > > Bob . . . > My experience when communicating with the battery vendors is that the meaning of 'gel' has 'ebbed and flowed' over time and is different with different vendors. I questioned a vendor (not Mighty Max) about their two models with the same capacity; one identified as 'gel' and the other as simple SLA. Their answer was that the 'gel' battery was an SLA designed for starting duty, and their other SLA was their deep discharge version. This is obviously opposite of what most of us would expect to hear. Bottom line: ya gotta ask. Charlie -- This email has been checked for viruses by Avast antivirus software. https://www.avast.com/antivirus ________________________________________________________________________________
Subject: Re: Small battery choice
From: "user9253" <fransew(at)gmail.com>
Date: Jan 24, 2021
The Mighty Max ML22-12GEL documentation does not mention AGM or SVLA. But printed on the front of the battery in large letters is: "Deep CYCLE AGM Battery" The above battery uses M5 bolts whereas the PC680 uses M6 bolts. The smaller size bolts might mean that the Might Max battery is not designed for starting. -------- Joe Gores Read this topic online here: http://forums.matronics.com/viewtopic.php?p=500405#500405 ________________________________________________________________________________
Date: Jan 24, 2021
From: "Robert L. Nuckolls, III" <nuckolls.bob(at)aeroelectric.com>
Subject: Re: Small battery choice
>The MM referenced is 0.014, which is still plenty good enough to >crank a 4 cyl Lyc . . . Aha! that's almost certain to be a 'gel-cell' battery. But if the price is right and you reside/fly in moderate climate conditions, they might serve your purposes well. Bill Bainbridge and I ran some cold delivery tests on new flooded, gel and AGM batteries as part of the qualification of his STC efforts on AGM batteries. I did the first proof installation on his batteries in a C150 waaayyyy back when. Of the three batteries cold-soaked and load tested, the flooded and gel batteries were pretty sad compared to the AGM. I don't recall now how the flooded and gel compared . . . it kind of sticks in my mind that the gel was worst which sorta goes along with the higher spec'd internal resistance. Bob . . . Un impeachable logic: George Carlin asked, "If black boxes survive crashes, why don't they make the whole airplane out of that stuff?" ________________________________________________________________________________
From: Charlie England <ceengland7(at)gmail.com>
Date: Jan 24, 2021
Subject: Re: Small battery choice
Kelly, Not questioning your experience with the generics, but it is worth noting that your tug is a different application (and only one example). I've never used an SLA in a true deep discharge environment (other than my computer UPSs, which are only online long enough for the generator to start). But my experience with the generics (which, for the most part, are designed for deep discharge applications) I'm (mis)using as *starting batteries* is that they do the job well, and last a long time. (Remember, in a starting application, you only pull 3-5% of the capacity to start the engine, and then the alternator is doing all the work through the rest of the flight.) On the other hand, if you search 'Odyssey' on the VAF RV forum for the last couple of years, you will find multiple accounts of owners who got great life from their original Odyssey but when they replaced it, the new one had very short life. I realize that this is 2nd hand info, so take it for whatever you think it's worth, but the stories are out there. Charlie On Sun, Jan 24, 2021 at 8:32 AM Kelly McMullen wrote: > kellym(at)aviating.com> > > Certainly not my experience. In aircraft, in Aridzona the typical > Concorde or Gill is good for 3-3.5 years. I have an Odyssey in my > homebuilt...a bigger 925 which has served well for 6 years. My tug uses > batteries in the size you are talking about. The manufacturer put in > Duracells. I tried generic and got 4 months. Paid twice as much for the > Duracells, and they are doing quite well. > > On 1/23/2021 3:45 PM, Charlie England wrote: > > > > > > On Sat, Jan 23, 2021 at 2:20 PM AdventureD > > wrote: > > > > > > Hi Dan, > > > > I've run Power Sonic, and at least 2 or 3 other 'no name' brands in a/c > > over the past 20+ years (probably everything *except* Odyssey), and I've > > never been able to detect any 'real world' difference among the brands. > > I buy the cheapest version I can find at the time, and it 'just works'. > > FWIW, there's been some internet chatter lately among the RV-x drivers > > that the current (pardon the pun) crop of Odysseys have had less than > > stellar lifespans (and they obviously extract a premium from your > > pocketbook). I've always gotten 4-6 years out of the no-name versions, > > with the only exception being one that had a pin hole in the case > > (shipping damage that I missed prior to installation). Even it lasted > > about a year before I noticed diminished capacity. > > > > FWIW, there are some higher AH versions available in almost identical > > case sizes. The naming convention for most is like the PowerSonic: > > 12(voltage)18(AH)followed by another zero. If you search for '12200 SLA > > battery' or '12220 SLA battery' (and read the specs carefully), you can > > get a 20 AH or 22AH that will fit in the same space, for about the same > > money. One thing to notice on the 'no names' is that many are intended > > for deep discharge applications (powered wheelchairs, etc), and will > > have slightly higher internal resistance than a PC680 'starting > > battery'. By going to a 20 AH or 22AH version, the internal impedance > > will be roughly the same as the PC680 (meaning same or better CCA), and > > the battery's *capacity* (total energy content) will be 10%-15% greater > > than a PC680 (stuff will run longer after an alternator failure). > > > > My experience since late in the last century; YMMV, etc etc. > > > > Charlie > > ________________________________________________________________________________
Date: Jan 24, 2021
From: "Robert L. Nuckolls, III" <nuckolls.bob(at)aeroelectric.com>
Subject: Re: Small battery choice
>My experience when communicating with the battery vendors is that the meaning >of 'gel' has 'ebbed and flowed' over time and is different with different >vendors. Right. In the early days of SVLA being offered at OSH, many vendors referred to their products as "gel". Had the same experience with folks behind the counters in store-front battery marts. The kicker is "AGM" which is absorbed glass mat . . . no way can AGM and Gel co-exist. There can be deep-discharge offerings in all three technologies which usually means more active material between plates and perhaps some tweaking of plate chemistry . . . For the techno-nerds among us, here's an excellent description of the gel technology as it compares to the other two lead-acid technologies. https://tinyurl.com/yxwlh8xm Also, note the website . . . batteryuniveristy.com This is the go-to place for the straight skinny on all things battery. The article say Sonnenschein was producer of the first gel cell devices. That's the brand that B&C was selling at OSH in 1986. Bob . . . Un impeachable logic: George Carlin asked, "If black boxes survive crashes, why don't they make the whole airplane out of that stuff?" ________________________________________________________________________________
From: Sebastien <cluros(at)gmail.com>
Date: Jan 24, 2021
Subject: Re: Small battery choice
> Also, note the website . . . batteryuniveristy.com > This is the go-to place for the straight skinny > on all things battery. Uhh ... personally I don't have anything against spinach but even though it's not nearly as destructive as the right-wing variety, left-wing click bait doesn't interest me. I think you meant batteryuniversity.com Bob. On Sun, Jan 24, 2021 at 10:01 AM Robert L. Nuckolls, III < nuckolls.bob(at)aeroelectric.com> wrote: > My experience when communicating with the battery vendors is that the > meaning > of 'gel' has 'ebbed and flowed' over time and is different with different > vendors. > > > Right. In the early days of SVLA being > offered at OSH, many vendors referred to > their products as "gel". Had the same > experience with folks behind the counters > in store-front battery marts. > > The kicker is "AGM" which is absorbed glass > mat . . . no way can AGM and Gel co-exist. > > There can be deep-discharge offerings in all three > technologies which usually means more active material > between plates and perhaps some tweaking of > plate chemistry . . . > > For the techno-nerds among us, here's an excellent > description of the gel technology as it compares to > the other two lead-acid technologies. > > https://tinyurl.com/yxwlh8xm > > Also, note the website . . . batteryuniveristy.com > This is the go-to place for the straight skinny > on all things battery. > > The article say Sonnenschein was producer of > the first gel cell devices. That's the brand that > B&C was selling at OSH in 1986. > > Bob . . . > > Un impeachable logic: George Carlin asked, "If black boxes > survive crashes, why don't they make the whole airplane > out of that stuff?" > ________________________________________________________________________________
Date: Jan 24, 2021
From: "Robert L. Nuckolls, III" <nuckolls.bob(at)aeroelectric.com>
Subject: Re: Small battery choice
At 11:19 AM 1/24/2021, you wrote: > >The Mighty Max ML22-12GEL documentation does not mention AGM or SVLA. >But printed on the front of the battery in large letters is: >"Deep CYCLE AGM Battery" >The above battery uses M5 bolts whereas the PC680 uses M6 bolts. >The smaller size bolts might mean that the Might Max battery is not >designed for starting. https://tinyurl.com/y35venox This claims to be thixotropic electrolyte (gel) -AND- macromolecule polymer separators (glass mat analog). But of course whether the electrolyte is liquid or gel -AND- whether free of barriers to circulation it must still have some form of separator. Glass-mat or polymer would seem to fill the bill. But the AGM moniker is not appropriate because the mats are not glass. The cells are vacuum charged with electrolyte . . . as are all the SVLA devices so it's probably a given that the electrolyte is still 'absorbed' within the separators even tho it's gelled. I'm not sure what advantage is gained by gelling the electrolyte. The terminals are solid copper posts so they're suited to high discharge rates like cranking an engine but the 14 milliohm internal resistance in a 22AH battery suggests it might be chemically weak. The spec sheet says it will support an 11A load for about one hour . . . same as a PC680 which is rated at only 16AH. I see them listed on Amazon for a range of prices with the cheaper ones featuring pierced-tab terminals as opposed to tapped-posts. I would be interesting to get one in to quantify for performance but right now, I've got too many things on my plate! I think I'd steer clear of this puppy Bob . . . Un impeachable logic: George Carlin asked, "If black boxes survive crashes, why don't they make the whole airplane out of that stuff?" ________________________________________________________________________________
Date: Jan 24, 2021
From: "Robert L. Nuckolls, III" <nuckolls.bob(at)aeroelectric.com>
Subject: Re: Small battery choice
>I think you meant <http://batteryuniversity.com>batteryuniversity.com Bob. Correct. Thanks! Bob . . . Un impeachable logic: George Carlin asked, "If black boxes survive crashes, why don't they make the whole airplane out of that stuff?" ________________________________________________________________________________
Subject: Re: Small battery choice
From: "AdventureD" <dobrien(at)microfoundations.com>
Date: Jan 24, 2021
Here's what I'm doing. I'm buying two Power-sonics. They're so cheap that if they don't work well, it's water under the bridge. I can use them in my motorcycle! I'll report back on the results. Hopefully first start will occur in the next two months. Cant wait. Thanks again. Dan Read this topic online here: http://forums.matronics.com/viewtopic.php?p=500413#500413 ________________________________________________________________________________
Date: Jan 25, 2021
From: "Robert L. Nuckolls, III" <nuckolls.bob(at)aeroelectric.com>
Subject: Re: Small battery choice
At 08:41 PM 1/24/2021, you wrote: > > >Here's what I'm doing. I'm buying two Power-sonics. They're so >cheap that if they don't work well, it's water under the bridge. I >can use them in my motorcycle! > >I'll report back on the results. Hopefully first start will occur >in the next two months. Cant wait. Looking forward to your experience and observations . . . Bob . . . Un impeachable logic: George Carlin asked, "If black boxes survive crashes, why don't they make the whole airplane out of that stuff?" ________________________________________________________________________________
Subject: bench power
From: "bobnoffs" <icubob(at)gmail.com>
Date: Jan 25, 2021
soon as the weather warms up a little [50 deg] i want to run my avionics and get limits etc. set. i have decided i do not want a power supply that could fry something if a knob gets bumped. i have gotten along 50 yrs without a power supply on my bench and can easily live without a variable source. anyone see any problems with connecting bench power thru the 'cig lighter' plug in? it's 18 ga wire and i would never pull more than a few amps. had i read something that powering an alternate field wire is not healthy? Read this topic online here: http://forums.matronics.com/viewtopic.php?p=500416#500416 ________________________________________________________________________________
Subject: Manual Battery Switch
From: C&K <yellowduckduo(at)gmail.com>
Date: Jan 25, 2021
Any recommendations for a reliable single pole manual battery switch that would replace a traditional battery contactor? This is for a rotax so starting current is relatively modest. There are lots of them listed on Amazon, ebay, and automotive suppliers but all of them seem to have complaints of falling apart or failing to work after a short time. I would like to mount it on the engine side of the firewall and arrange mechanical linkage to operable it from inside the cabin. thanks Ken ________________________________________________________________________________
Subject: Re: bench power
From: Charlie England <ceengland7(at)gmail.com>
Date: Jan 25, 2021
On 1/25/2021 1:38 PM, bobnoffs wrote: > > soon as the weather warms up a little [50 deg] i want to run my avionics and get limits etc. set. i have decided i do not want a power supply that could fry something if a knob gets bumped. i have gotten along 50 yrs without a power supply on my bench and can easily live without a variable source. > anyone see any problems with connecting bench power thru the 'cig lighter' plug in? it's 18 ga wire and i would never pull more than a few amps. > had i read something that powering an alternate field wire is not healthy? That's how I get 'shore power' to my RV6. But don't just assume that 'it's only a few amps'; verify actual load. My panel is probably pretty typical for VFR panels with 'glass' flight instruments, and the 18 ga supply wire gets fairly warm. I've just built another cable using 16 ga, & will see how that goes. If the field circuit concerns you, just pull the field CB or open the switch. I understand your concern about accidental OV, but there are numerous advantages to a fully controllable supply where you can limit current as well as voltage. Charlie -- This email has been checked for viruses by Avast antivirus software. https://www.avast.com/antivirus ________________________________________________________________________________
Date: Jan 25, 2021
From: "Robert L. Nuckolls, III" <nuckolls.bob(at)aeroelectric.com>
Subject: Re: bench power
>anyone see any problems with connecting bench power thru the 'cig >lighter' plug in? What is the power source? I used to sell a power supply in this class for about 3x the money. Sold quite a few. Builders would power up accessories on the bench or, on occasion, power up their airplanes with it. https://tinyurl.com/y3vww5uv > it's 18 ga wire and i would never pull more than a few amps. It can't do anything worse than not work and you're not going to crash your workbench. Certainly gazillions of the things are still in service in automobiles after about 100 years . . . so why not? > > had i read something that powering an alternate field wire > is not healthy? Not sure what this question is about . . . Bob . . . Un impeachable logic: George Carlin asked, "If black boxes survive crashes, why don't they make the whole airplane out of that stuff?" ________________________________________________________________________________
Date: Jan 25, 2021
From: "Robert L. Nuckolls, III" <nuckolls.bob(at)aeroelectric.com>
Subject: Re: Manual Battery Switch
At 02:25 PM 1/25/2021, you wrote: > >Any recommendations for a reliable single pole >manual battery switch that would replace a >traditional battery contactor?=C2 This is for a >rotax so starting current is relatively >modest.=C2 There are lots of them listed on >Amazon, ebay, and automotive suppliers but all >of them seem to have complaints of falling apart >or failing to work after a short time.=C2 I would >like to mount it on the engine side of the >firewall and arrange mechanical linkage to operable it from inside the cabin. >thanks There are no doubt some excellent choices out there but one that I'm familiar with is Cole-Hersee products. Check out this puppy. https://tinyurl.com/yyydc77r Bob . . . Un impeachable logic: George Carlin asked, "If black boxes survive crashes, why don't they make the whole airplane out of that stuff?" ________________________________________________________________________________
Subject: Re: Manual Battery Switch
From: "rparigoris" <rparigor(at)hotmail.com>
Date: Jan 25, 2021
Hi Ken Not cheap but I'm using a Flaming River FR1013 on the starboard headrest of Rotax 914 powered Europa. Battery is in rear. I looked at a number of switches, even bought some and didn't like the looks of them, not robust enough. I'm not flying yet but do like the looks of the Flaming River. They sell a number of battery disconnect switches: https://www.flamingriver.com/battery-disconnect-switches?p=1 Ron P. Read this topic online here: http://forums.matronics.com/viewtopic.php?p=500421#500421 ________________________________________________________________________________
Subject: Re: bench power
From: "bobnoffs" <icubob(at)gmail.com>
Date: Jan 25, 2021
when i use the 'bench power 'source i plan to only power a small part of my panel at a time. a couple amps. my questions are1. is there something i may be applying power to that may cause harm the way i plan to do it and 2. can anything detrimental happen to the charging system if power is applied to the field wire. sorry if i wasn't clear. i know this is kind of ambiguous as panel wiring varies. Read this topic online here: http://forums.matronics.com/viewtopic.php?p=500422#500422 ________________________________________________________________________________
Date: Jan 25, 2021
From: b j <njrwwj(at)yahoo.com>
Subject: Re: AeroElectric-List Digest: 14 Msgs - 01/23/21
Remove from all lists Sent from Yahoo Mail for iPad On Sunday, January 24, 2021, 1:30 AM, AeroElectric-List Digest Server wrote: * ======================== =C2- Online Versions of Today's List Digest Archive ======================== Today's complete AeroElectric-List Digest can also be found in either of th e two Web Links listed below.=C2- The .html file includes the Digest format ted in HTML for viewing with a web browser and features Hyperlinked Indexes and Message Navigation.=C2- The .txt file includes the plain ASCII versio n of the AeroElectric-List Digest and can be viewed with a generic text edito r such as Notepad or with a web browser. HTML Version: =C2- =C2- http://www.matronics.com/digest/digestview.php?Style=82701& View=html&Chapter 21-01-23&Archive=AeroElectric Text Version: =C2- =C2- http://www.matronics.com/digest/digestview.php?Style=82701& View=txt&Chapter 21-01-23&Archive=AeroElectric ======================== ======================= =C2- EMail Version of Today's List Digest Archive ======================== ======================= =C2- =C2- =C2- =C2- =C2- ---------------------------------------- ------------------ =C2- =C2- =C2- =C2- =C2- =C2- =C2- =C2- =C2- =C2- =C2 - =C2- =C2- AeroElectric-List Digest Archive =C2- =C2- =C2- =C2- =C2- =C2- =C2- =C2- =C2- =C2- =C2 - =C2- =C2- =C2- =C2- =C2- =C2- =C2- =C2- --- =C2- =C2- =C2- =C2- =C2- =C2- =C2- =C2- =C2- =C2- Total Messages Posted Sat 01/23/21: 14 =C2- =C2- =C2- =C2- =C2- ---------------------------------------- ------------------ Today's Message Index: ---------------------- =C2- =C2- 1. 11:01 AM - Re: Re: Zeftronics Regulator Connection Questio n=C2- (Robert L. Nuckolls, III) =C2- =C2- 2. 11:20 AM - Re: Zeftronics Regulator Connection Question=C2 - (racerjerry) =C2- =C2- 3. 12:15 PM - Small battery choice=C2- (AdventureD) =C2- =C2- 4. 12:38 PM - Re: Small battery choice=C2- (C&K) =C2- =C2- 5. 01:05 PM - Re: Zeftronics Regulator Connection Question=C2 - (farmrjohn) =C2- =C2- 6. 02:51 PM - Re: Stratux RS-232 Wx+Trfc Out to various navig ators...=C2- (farmrjohn) =C2- =C2- 7. 02:52 PM - Re: Small battery choice=C2- (Charlie England ) =C2- =C2- 8. 03:00 PM - Re: Small battery choice=C2- (AdventureD) =C2- =C2- 9. 03:05 PM - Re: Small battery choice=C2- (AdventureD) =C2- =C2- 10. 04:04 PM - Re: Small battery choice=C2- (Kelly McMullen ) =C2- =C2- 11. 04:51 PM - Re: Small battery choice=C2- (user9253) =C2- =C2- 12. 06:21 PM - Re: Re: Small battery choice=C2- (Charlie En gland) =C2- =C2- 13. 07:24 PM - Re: Small battery choice=C2- (user9253) =C2- =C2- 14. 07:59 PM - Re: Re: Small battery choice=C2- (Charlie En gland) ________________________________=C2- Message 1=C2- ____________________ _________________ From: "Robert L. Nuckolls, III" <nuckolls.bob(at)aeroelectric.com> Subject: Re: AeroElectric-List: Re: Zeftronics Regulator Connection=C2- Q uestion I've updated the drawing that was attached to my last reply to this thread. Interested readers can download at: https://tinyurl.com/yxg6hfrj There are additional nuances to the Cessna alternator incorporation philosophy that I may add later on a second page to this drawing . . . will give a heads-up when posted. =C2- Bob . . . =C2- Un impeachable logic: George Carlin asked, "If black boxes =C2- survive crashes, why don't they make the whole airplane =C2- out of that stuff?" ________________________________=C2- Message 2=C2- ____________________ _________________ Subject: AeroElectric-List: Re: Zeftronics Regulator Connection Question From: "racerjerry" <gnking2(at)verizon.net> Bob, Zeftronics offers an excellent troubleshooting guide for their voltage regulators online.=C2- The included diagrams MAY help to clear up some of the myster y; unfortunately, it's still a bit opaque for me.=C2- And yes, my guess is " If it ain't broke - Don't fix it" still applies. https://www.zeftronics.com/wp-content/uploads/2014/06/Z13PS.pdf -------- Jerry King Read this topic online here: http://forums.matronics.com/viewtopic.php?p=500385#500385 ________________________________=C2- Message 3=C2- ____________________ _________________ Subject: AeroElectric-List: Small battery choice From: "AdventureD" <dobrien(at)microfoundations.com> Ordering two batteries for my ship (Z14, rear bats).=C2- Some years ago, the good doctor recommended three: > A few exemplar brands and part numbers are: > Panasonic LC-RD1217 > Odyssey PC680 > Power Sonic PSH-12180FR The Power Sonic is cheapest (only $60! ... quite a schoke after spending $6 00 on my Mooney batteries!) and available at Amazon.=C2- I figure there is not a lot of difference between these, but wanted to check if anyone had reason to pi ck one over the other (or another in same roughly 7"-7"-3" profile). Cheers, Dan Read this topic online here: http://forums.matronics.com/viewtopic.php?p=500386#500386 ________________________________=C2- Message 4=C2- ____________________ _________________ Subject: Re: AeroElectric-List: Small battery choice From: C&K <yellowduckduo(at)gmail.com> I've had very good results from the Dekka power sports batteries for many years. The last ones I bought were ordered for pickup at Lowes. American made. Ken On 23/01/2021 3:14 PM, AdventureD wrote: > > Ordering two batteries for my ship (Z14, rear bats).=C2- Some years ago , the good doctor recommended three: > > >> A few exemplar brands and part numbers are: >> Panasonic LC-RD1217 >> Odyssey PC680 >> Power Sonic PSH-12180FR > > The Power Sonic is cheapest (only $60! ... quite a schoke after spending $600 on my Mooney batteries!) and available at Amazon.=C2- I figure there is n ot a lot of difference between these, but wanted to check if anyone had reason to pi ck one over the other (or another in same roughly 7"-7"-3" profile). > > Cheers, > Dan > > > Read this topic online here: > > http://forums.matronics.com/viewtopic.php?p=500386#500386 > > ________________________________=C2- Message 5=C2- ____________________ _________________ Subject: AeroElectric-List: Re: Zeftronics Regulator Connection Question From: "farmrjohn" <faithvineyard(at)yahoo.com> This installation is not in a Cessna but on an Aircoupe with an O200 engine wtih alternator vs. the C90 it came with.=C2- It also does not have the over v oltage sensor installed, rather utilizes the EI voltage meter for warning.=C2- I t looks like the middle diagram on page 3 of the Zeftronics trouble shooting guide =C2- would show what happens and how it works if the alternator control switch is conn ected direct to the I terminal vs. the S terminal if I'm interpreting it correctl y. John Read this topic online here: http://forums.matronics.com/viewtopic.php?p=500389#500389 ________________________________=C2- Message 6=C2- ____________________ _________________ Subject: AeroElectric-List: Re: Stratux RS-232 Wx+Trfc Out to various navig ators... From: "farmrjohn" <faithvineyard(at)yahoo.com> Instead if a USB port for the RS232 out would it be possible to utilize the ethernet port with something like:=C2- https://tinyurl.com/yyqm2fgz Read this topic online here: http://forums.matronics.com/viewtopic.php?p=500390#500390 ________________________________=C2- Message 7=C2- ____________________ _________________ From: Charlie England <ceengland7(at)gmail.com> Subject: Re: AeroElectric-List: Small battery choice On Sat, Jan 23, 2021 at 2:20 PM AdventureD wrote: > dobrien(at)microfoundations.com> > > Ordering two batteries for my ship (Z14, rear bats).=C2- Some years ago , the > good doctor recommended three: > > > > A few exemplar brands and part numbers are: > > Panasonic LC-RD1217 > > Odyssey PC680 > > Power Sonic PSH-12180FR > > > The Power Sonic is cheapest (only $60! ... quite a schoke after spending > $600 on my Mooney batteries!) and available at Amazon.=C2- I figure the re is > not a lot of difference between these, but wanted to check if anyone had > reason to pick one over the other (or another in same roughly 7"-7"-3" > profile). > > Cheers, > Dan > Hi Dan, I've run Power Sonic, and at least 2 or 3 other 'no name' brands in a/c over the past 20+ years (probably everything *except* Odyssey), and I've never been able to detect any 'real world' difference among the brands. I buy the cheapest version I can find at the time, and it 'just works'. FWIW, there's been some internet chatter lately among the RV-x drivers that the current (pardon the pun) crop of Odysseys have had less than stellar lifespans (and they obviously extract a premium from your pocketbook). I've always gotten 4-6 years out of the no-name versions, with the only exception being one that had a pin hole in the case (shipping damage that I missed prior to installation). Even it lasted about a year before I noticed diminished capacity. FWIW, there are some higher AH versions available in almost identical case sizes. The naming convention=C2- for most is like the PowerSonic: 12(voltage)18(AH)followed by another zero. If=C2- you search for '12200 S LA battery' or '12220 SLA battery' (and read the specs carefully), you can get a 20 AH or 22AH that will fit in the same space, for about the same money. One thing to notice on the 'no names' is that many are intended for deep discharge applications (powered wheelchairs, etc), and will have slightly higher internal resistance than a PC680 'starting battery'. By going to a 20 AH or 22AH version, the internal impedance will be roughly the same as the PC680 (meaning same or better CCA), and the battery's *capacity* (total energy content) will be 10%-15% greater than a PC680 (stuff will run longer after an alternator failure). My experience since late in the last century; YMMV, etc etc. Charlie ________________________________=C2- Message 8=C2- ____________________ _________________ Subject: AeroElectric-List: Re: Small battery choice From: "AdventureD" <dobrien(at)microfoundations.com> A bit more on this -- with the redundancy I have in the Z14 system, I am no t looking for a high $$ battery.=C2- I am happy to change one out every year IF the batteries do a good job cranking the engine.=C2- I have a high compression IO550, and the ability to close the cross feed to crank.=C2- How can I tell from the data sheets whether these batteries either alone or in combination are up to the task?=C2- The Power Sonic datasheet reports a max of 270 amps for 5 secon ds.=C2- There seems to be all kinds of different reports online about what is required to start an engine.=C2- I don't want to get batters that aren't up to the ta sk. Any thoughts appreciated. Dan Read this topic online here: http://forums.matronics.com/viewtopic.php?p=500392#500392 ________________________________=C2- Message 9=C2- ____________________ _________________ Subject: AeroElectric-List: Re: Small battery choice From: "AdventureD" <dobrien(at)microfoundations.com> Charlie, what engine are you cranking? Best, Dan Read this topic online here: http://forums.matronics.com/viewtopic.php?p=500393#500393 ________________________________=C2- Message 10=C2- ___________________ _________________ Subject: Re: AeroElectric-List: Small battery choice From: Kelly McMullen <kellym(at)aviating.com> Certainly not my experience. In aircraft, in Aridzona the typical Concorde or Gill is good for 3-3.5 years. I have an Odyssey in my homebuilt...a bigger 925 which has served well for 6 years. My tug uses batteries in the size you are talking about. The manufacturer put in Duracells. I tried generic and got 4 months. Paid twice as much for the Duracells, and they are doing quite well. On 1/23/2021 3:45 PM, Charlie England wrote: > > > On Sat, Jan 23, 2021 at 2:20 PM AdventureD > wrote: > >=C2- =C2- > Hi Dan, > > I've run Power Sonic, and at least 2 or 3 other 'no name' brands in a/c > overthe past 20+ years (probably everything *except* Odyssey), and I've > never been able to detect any 'real world' difference among the brands. > I buy the cheapest version I can find at the time, and it 'just works'. > FWIW, there's been some internet chatter lately among the RV-x drivers > that the current (pardon the pun) crop of Odysseyshave had less than > stellar lifespans (and they obviously extract a premium from your > pocketbook). I've always gotten 4-6 years out of the no-name versions, > with the only exception being one that had a pin holein the case > (shipping damage that I missed prior to installation). Even it lasted > about a year before I noticed diminished capacity. > > FWIW, there are some higher AH versions available in almost identical > case sizes. The naming convention for most is like the PowerSonic: > 12(voltage)18(AH)followed by another zero. If you search for '12200 SLA > battery' or '12220 SLA battery' (and read the specs carefully), you can > get a 20 AH or 22AH that will fit in the same space, for about the same > money. One thing to notice on the 'no names' is that many are intended > for deep discharge applications (powered wheelchairs, etc), and will > have slightly higher internal resistance than a PC680 'starting > battery'. By going to a 20 AH or 22AH version, the internal impedance > will be roughly the same as the PC680 (meaning same or better CCA), and > the battery's *capacity* (total energy content) will be 10%-15% greater > than a PC680 (stuff will run longer after an alternator failure). > > My experience since late in the last century; YMMV, etc etc. > > Charlie ________________________________=C2- Message 11=C2- ___________________ _________________ Subject: AeroElectric-List: Re: Small battery choice From: "user9253" <fransew(at)gmail.com> Compare the weight.=C2- Some batteries made in China claim to be 18 or 22 AH.=C2- But when you look at the weight, they weigh less than a PC680 (15.4 pounds) . One deceiving trick is to use a larger case than necessary to hold the meag er contents. -------- Joe Gores Read this topic online here: http://forums.matronics.com/viewtopic.php?p=500395#500395 ________________________________=C2- Message 12=C2- ___________________ _________________ Subject: Re: AeroElectric-List: Re: Small battery choice From: Charlie England <ceengland7(at)gmail.com> On 1/23/2021 5:04 PM, AdventureD wrote: > > Charlie, what engine are you cranking? > > Best, > Dan > O-320 8.5-1 compression for most of that time. Joe makes a reasonable point for reality checks, but if you parallel a pair, just about anything you purchase would crank just about any engine you choose. Charlie -- This email has been checked for viruses by Avast antivirus software. https://www.avast.com/antivirus ________________________________=C2- Message 13=C2- ___________________ _________________ Subject: AeroElectric-List: Re: Small battery choice From: "user9253" <fransew(at)gmail.com> Here is another option: Mighty Max ML22-12GEL 12 Volt 22 AH Can be ordered from Home Depot or Lowes -------- Joe Gores Read this topic online here: http://forums.matronics.com/viewtopic.php?p=500398#500398 ________________________________=C2- Message 14=C2- ___________________ _________________ Subject: Re: AeroElectric-List: Re: Small battery choice From: Charlie England <ceengland7(at)gmail.com> On 1/23/2021 9:23 PM, user9253 wrote: > > Here is another option: Mighty Max ML22-12GEL 12 Volt 22 AH > Can be ordered from Home Depot or Lowes > > -------- > Joe Gores That looks like a reasonable option to me. One measure for comparison (the easiest to find) is CCA (cold cranking amps). A harder one to find in docs is internal resistance (impedance). The PC680 is 7 milliohms (0.007). Most of the no-name stuff will be around 0.008-0.015. The MM referenced is 0.014, which is still plenty good enough to crank a 4 cyl Lyc, and would probably crank a 6 without too much effort. Odyssey https://www.odysseybattery.com/products/ods-agm16l-battery-pc680/ MightyMax https://smile.amazon.com/Mighty-Max-Battery-Brand-Product/dp/B01JNYYXII/ref =smi_www_rco2_go_smi_g4368549507?_encoding=UTF8&%2AVersion%2A=1&%2Aen tries%2A=0&ie=UTF8 Charlie -- This email has been checked for viruses by Avast antivirus software. https://www.avast.com/antivirus - S - WIKI - - =C2- =C2- =C2- =C2- =C2- -Matt Dralle, List Admin. ________________________________________________________________________________
Date: Jan 26, 2021
From: Jeff Luckey <jluckey(at)pacbell.net>
Subject: Re: bench power
Hello Bob, "is there something i may be applying power to that may cause harm.." Probably the riskiest thing would be that you have power wires reversed on some gizmo/accessory.=C2- If you put reverse polarity on a device that is not protected for it, you could ruin it, even if it is turned off.=C2- S o be sure to triple check polarity. "...can anything detrimental happen to the charging system if power is appl ied to the field wire ..." No.=C2- The only issue with having power on the field is that it will hea t-up the field inside the alternator and with no rotation there would be li ttle cooling.=C2- I don't know your setup but typically you would=C2-re move power from the regulator by opening=C2-the field breaker. Another thing to consider is keying-up a transmitter without an antenna.=C2 - In the old days you could damage the output stage of the transmitter wi thout an antenna load.=C2- Another Lister with more knowledge about trans mitters might provide more info. -Jeff m> wrote: when i use the 'bench power 'source i plan to only power a small part of my panel at a time. a couple amps. my questions are1. is there something i ma y be applying power to that may cause harm the way i plan to do it and 2. c an anything detrimental happen to the charging system if power is applied t o the field wire. sorry if i wasn't clear. i know this is kind of ambiguous as panel wiring varies. Read this topic online here: http://forums.matronics.com/viewtopic.php?p=500422#500422 - S - WIKI - - =C2- =C2- =C2- =C2- =C2- -Matt Dralle, List Admin. ________________________________________________________________________________
Date: Jan 26, 2021
From: "Robert L. Nuckolls, III" <nuckolls.bob(at)aeroelectric.com>
Subject: Re: bench power
>"...can anything detrimental happen to the charging system if power >is applied to the field wire ..." > >No. The only issue with having power on the field is that it will >heat-up the field inside the alternator and with no rotation there >would be little cooling. I don't know your setup but typically you >would remove power from the regulator by opening the field breaker. or having the OFF/BAT/ALT+BAT switch in OFF or BAT >Another thing to consider is keying-up a transmitter without an antenna. >In the old days you could damage the output stage of the transmitter >without an antenna load. Another Lister with more knowledge about >transmitters might provide more info. That's sorta true in the vacuum tube days . . . but still valid for long intervals of unloaded transmission. Early solid state transmitters were germanium transistors and fragile compared to modern FETS. It wasn't an instantaneous thing but yeah . . . long-winded 'tests' were hard on them. This also pre-dated built-in SWR monitor circuits that reduce or shut off drive to output stage if poorly terminated. There probably hasn't been a VHF Comm transceiver in the last 40 years that doesn't have high SWR protection. Bob . . . Un impeachable logic: George Carlin asked, "If black boxes survive crashes, why don't they make the whole airplane out of that stuff?" ________________________________________________________________________________
Subject: Re: bench power
From: "bobnoffs" <icubob(at)gmail.com>
Date: Jan 27, 2021
thanks for the input. i plan to put a cig. lighter plug on output of the power source. my rv12 is wired to easily handle current for one source at a time thru the cig. plug. instead of the mapbox, when i open the door on the panel my fuse panel is right there. i will keep the fuse panel empty [ field wire] except for what i am working on. if i never need a lower voltage from the power supply i can run it thru a bucks regulator which i have. usually i don't know what i don't know about electricity. Read this topic online here: http://forums.matronics.com/viewtopic.php?p=500426#500426 ________________________________________________________________________________
Subject: Rectifier/Regulator
From: Charles Davis <charlesdavis(at)iuncapped.co.za>
Date: Jan 27, 2021
greetings, all Does anyone recognize this rectifier/regulator ? It was given to me for my project (Was in a Rotax-powered plane which was destroyed, and I am busy installing a 912) Only 2 of the three yellow wires were connected - I assume they were /are for a 3-phase alternator, whereas the rotax only has 2 wires from the alternator coils ... The 2 red and 2 green are terminated in a 2-pin male and a 2-pin female molex-type connector - I assume they are +ve and -ve to the battery the 2 white wires in the pic are a 20,000 uF cap, 25v Can I test the functionality by supplying 9v, 50Hz from an isolation transformer, and connect the red/green pairs to a gel 12v battery ? There are no markings or numbers of any sort on the unit What would be the simplest way to have a charging/not charging indication - an idiot-light would be my preferance Charles ________________________________________________________________________________
Date: Jan 27, 2021
From: "Robert L. Nuckolls, III" <nuckolls.bob(at)aeroelectric.com>
Subject: Re: Rectifier/Regulator
At 09:56 AM 1/27/2021, you wrote: >greetings, all > >Does anyone recognize this rectifier/regulator ? > >It was given to me for my project (Was in a Rotax-powered plane >which was destroyed, and I am busy installing a 912) > >Only 2 of the three yellow wires were connected - I assume they were >/are for a 3-phase alternator, whereas the rotax only has 2 wires >from the alternator coils ... > >The 2 red and 2 green are terminated in a 2-pin male and a 2-pin >female molex-type connector - I assume they are +ve and -ve to the battery > >the 2 white wires in the pic are a 20,000 uF cap, 25v > >Can I test the functionality by supplying 9v, 50Hz from an isolation >transformer, and connect the red/green pairs to a gel 12v battery ? > >There are no markings or numbers of any sort on the unit > >What would be the simplest way to have a charging/not charging >indication - an idiot-light would be my preferance > >Charles Low voltage warning light is the PRIMARY function monitor in the simple electrical system. Voltage above 13.5, alternator MUST be working . . . light OFF. Voltage below 13.5; alternator is not working . . . light flashing. Bob . . . Un impeachable logic: George Carlin asked, "If black boxes survive crashes, why don't they make the whole airplane out of that stuff?" ________________________________________________________________________________
Subject: Re: Rectifier/Regulator
From: "rparigoris" <rparigor(at)hotmail.com>
Date: Jan 27, 2021
Hi Charles It kinda looks like a Silent: https://www.silent-hektik.de/UL_R_912.htm Sorry I don't know too much about the unit. You may be able to obtain what you need if you translate page? I believe Vans Aircraft now sells them for their RVs, perhaps you can see if they can help? On my 914 I'm using a B&C AVC1. Ron P. Read this topic online here: http://forums.matronics.com/viewtopic.php?p=500430#500430 ________________________________________________________________________________
Subject: Re: Rectifier/Regulator
From: "user9253" <fransew(at)gmail.com>
Date: Jan 27, 2021
Charles, is your alternator a single phase permanent magnet type? If so, buy a new AM101406 regulator from eBay for $19 and free shipping. Mount it with heat conductive paste and keep it cool with an air blast tube. That is what I did and it has lasted 4 years so far. The wires in your picture look heat damaged where they enter the case. Your old one can not be tested with 9 volts. A 16 volt transformer might work. But why bother when a new is inexpensive? -------- Joe Gores Read this topic online here: http://forums.matronics.com/viewtopic.php?p=500431#500431 ________________________________________________________________________________
Subject: Some Thoughts on Z101
From: "Eric Page" <edpav8r(at)yahoo.com>
Date: Jan 27, 2021
I'm starting to sneak up on wiring my Kitfox project, so I've been looking over Z101 in detail. Take a look at this idea to simplify the Brown-Out Booster and AUX BUS Normal Feed path, and tell me if I'm out to lunch: Z101, for reference: https://tinyurl.com/1120mgeh 1. Delete the AUX BUS normal feed path from the bridge rectifier. 2. Remove the Brown-Out Booster input feed from the Engine Start pushbutton switch, and instead connect it directly to the Battery Contactor with a short-as-practical 14AWG wire. 3. Remove the output of the Brown-Out Booster from the normally-closed contact of the AUX BUS Alternate Feed Relay, and instead connect it to the AUX BUS normal feed point that was previously supplied by the bridge rectifier, again with a short-as-practical 14AWG wire. Since the DC-to-DC boost topology (see attached image) offers no barrier to forward current flow, it can pass normal feed current for the AUX BUS even when it isn't needed for boost duty. Its rectifier will prevent backfeed from the AUX BUS to the MAIN BUS in the event that the AUX BUS Alternate Feed Relay is closed. If the Brown-Out Booster's output is set to 12V, then its controller IC will simply stop switching any time its output voltage exceeds that value. During engine start, the controller IC will already be "awake" and will start switching the instant its input sags to one diode drop above 12V. A few issues that I'm still mulling over: 1. This proposal assumes that the Brown-Out Booster is tested to ensure that its inductor and rectifier can continuously pass full running current for the AUX BUS, and any later additions to the AUX BUS will have to be considered with this limitation in mind. 2. Is it acceptable -- or even a good idea -- to connect a Brown-Out Booster directly to the Battery Contactor with no fuse, as if it were integral to the AUX BUS? 3. If the answer to #2 is no, then does it make sense to fuse the Brown-Out Booster/AUX BUS Normal Feed, and if so, at what value? This is complicated by the fact that Booster input current increases faster than output current, and it also increases as input voltage decreases. Thoughts? Read this topic online here: http://forums.matronics.com/viewtopic.php?p=500432#500432 Attachments: http://forums.matronics.com//files/simplified_boost_topology_619.jpg ________________________________________________________________________________
Subject: Re: Rectifier/Regulator
From: "user9253" <fransew(at)gmail.com>
Date: Jan 27, 2021
If you install an AM101406, connect as follows: B+ goes to the main power bus through a 30 amp fuse. The AC terminals connect to the dynamo. Terminal "L" is for an indicator lamp. IGW gets battery power through a switch labeled "Alternator". -------- Joe Gores Read this topic online here: http://forums.matronics.com/viewtopic.php?p=500433#500433 ________________________________________________________________________________
From: Jared Yates <email(at)jaredyates.com>
Date: Jan 27, 2021
Subject: Coax Mixing RG-400 and RG58
My hangar neighbor's airplane already has rg58 for the transponder antenna, but it is 15 inches short. I happen to have a piece that would fix it, bit it is RG400. Is there any problem running the two different types of cable in the same feed? Do we need to try and round up a piece of RG58 instead? ________________________________________________________________________________
From: Charlie England <ceengland7(at)gmail.com>
Date: Jan 27, 2021
Subject: Re: Coax Mixing RG-400 and RG58
Same impedance, so I know that *I'd* certainly try it (using the proper connectors), before ordering a 2' jumper or re-running the whole thing. Worst case scenario would be ATC telling you it isn't working. Charlie On Wed, Jan 27, 2021 at 4:42 PM Jared Yates wrote: > My hangar neighbor's airplane already has rg58 for the transponder > antenna, but it is 15 inches short. I happen to have a piece that would fix > it, bit it is RG400. Is there any problem running the two different types > of cable in the same feed? Do we need to try and round up a piece of RG58 > instead? > ________________________________________________________________________________
Subject: Re: Some Thoughts on Z101
From: "user9253" <fransew(at)gmail.com>
Date: Jan 27, 2021
Eric, what advantage does your proposal have over Z-101? Why eliminate the normal diode feed path to the aux bus? You can rewire the brownout booster whether the diode is there or not. It is a bad idea to have two fuses in series. Z-101 does not fuse the aux bus. Is a brown out booster really necessary? Doesn't your EFIS have an internal backup battery? My radio recovers in a few seconds after engine cranking. My transponder takes longer to establish its GPS location. But so what? Why even supply a whole bus with brownout protection? Why is a separate aux bus needed? Just connect one or two loads to a brownout booster using dual diodes. Consider the attached electrical drawing. Just the relay can be energized for ground operations. In the unlikely event that the main contactor fails in flight, the relay can be energized. With a single bus, there is no need to remember what loads are on what bus. If it is desired to conserve the battery, just shut off unneeded loads. -------- Joe Gores Read this topic online here: http://forums.matronics.com/viewtopic.php?p=500436#500436 Attachments: http://forums.matronics.com//files/contactor_parallel_relay_207.jpg ________________________________________________________________________________
Subject: Re: Coax Mixing RG-400 and RG58
From: Kelly McMullen <kellym(at)aviating.com>
Date: Jan 27, 2021
I don't know precise years, but RG-400 came along sometime after I put a transponder in my first airplane, in the mid-80s. Back then RG-58 was used for every antenna, except some pricier airplanes used a couple other 75 ohm cables with more insulation between core and shield. (RG174? I think). RG-400 is better for DME, transponders and GPS that operate above 900 Mhz, but RG-58 worked okay. On 1/27/2021 3:39 PM, Charlie England wrote: > Same impedance, so I know that *I'd* certainly try it (using the proper > connectors), before ordering a 2' jumper or re-running the whole thing. > Worstcase scenario would be ATC telling you it isn't working. > > Charlie > > On Wed, Jan 27, 2021 at 4:42 PM Jared Yates > wrote: > > __ > My hangar neighbor's airplane already has rg58 for the transponder > antenna, but it is 15 inches short. I happen to have a piece that > would fix it, bit it is RG400. Is there any problem running the two > different types of cable in the same feed? Do we need to try and > round up a piece of RG58 instead? > ________________________________________________________________________________
Subject: Re: Coax Mixing RG-400 and RG58
From: "user9253" <fransew(at)gmail.com>
Date: Jan 27, 2021
When faced with a decision like this, do what is best in the long run, not what is easiest or least expensive. -------- Joe Gores Read this topic online here: http://forums.matronics.com/viewtopic.php?p=500438#500438 ________________________________________________________________________________
Date: Jan 27, 2021
From: "Robert L. Nuckolls, III" <nuckolls.bob(at)aeroelectric.com>
Subject: Re: Coax Mixing RG-400 and RG58
At 04:39 PM 1/27/2021, you wrote: >Same impedance, so I know that *I'd* certainly >try it (using the proper connectors), before >ordering a 2' jumper or re-running the whole >thing.=C2 Worst=C2 case scenario would be ATC telling you it isn't working. Agreed. There are no performance risks for extending the original run of coax with a more modern material as long as the connectors are appropriate to the wire and in good shape. Bob . . . Un impeachable logic: George Carlin asked, "If black boxes survive crashes, why don't they make the whole airplane out of that stuff?" ________________________________________________________________________________
Date: Jan 27, 2021
From: "Robert L. Nuckolls, III" <nuckolls.bob(at)aeroelectric.com>
Subject: Re: Some Thoughts on Z101
> >Thoughts? Can you elaborate on your design goals? The original goal was to craft a tinker-toy/leggo style architecture offering a variety of mix/match features. Each feature enjoyed two feed paths. The presence of no feature was dependent on configuration of other features (except for the brownout booster which goes away if its target bus is not installed). All busses are hot any time the main bus is hot irrespective of other switch positions. Busses wrapped around the main bus may be selectively powered up independent of the main bus. A full-up system is managed with only 3 toggle switches. Re brownout booster: I'm pondering a plan-d . . . or are we up to 'e'? There are dozens of suitable step up devices on the market but I have no knowledge of their 'spool up' times nor can I personally vouch for their specifications without putting the device under test on the bench and doing some measurements. I think I have a way to craft a brown-out booster of KNOWN performance thus avoiding the risks for incorporating the Chinese unknowns. Watch this space. Bob . . . Un impeachable logic: George Carlin asked, "If black boxes survive crashes, why don't they make the whole airplane out of that stuff?" ________________________________________________________________________________
Date: Jan 27, 2021
From: "Robert L. Nuckolls, III" <nuckolls.bob(at)aeroelectric.com>
Subject: Re: Some Thoughts on Z101
> > >Attachments: > >http://forums.matronics.com//files/simplified_boost_topology_619.jpg A full schematic of the proposed boost system would make it easier to 'grok'. Bob . . . Un impeachable logic: George Carlin asked, "If black boxes survive crashes, why don't they make the whole airplane out of that stuff?" ________________________________________________________________________________
From: Sebastien <cluros(at)gmail.com>
Date: Jan 27, 2021
Subject: Re: Some Thoughts on Z101
Bob I think a clearance delivery bus with brownout protection is an important design goal for any IFR OBAM aircraft. The ability to get the clearance, program it, and then start the engine rather than doing it all with the engine running is almost a necessity in my mind. I may have enough equipment here that you could unload the testing portion onto me. If this interests you please get in touch and I will order the parts and assemble and test a system to your specs. On Wed, Jan 27, 2021 at 7:02 PM Robert L. Nuckolls, III < nuckolls.bob(at)aeroelectric.com> wrote: > > > Attachments: > > http://forums.matronics.com//files/simplified_boost_topology_619.jpg > > > A full schematic of the proposed boost > system would make it easier to 'grok'. > > Bob . . . > > Un impeachable logic: George Carlin asked, "If black boxes > survive crashes, why don't they make the whole airplane > out of that stuff?" > ________________________________________________________________________________
From: "Matthew S. Whiting" <m.whiting(at)frontier.com>
Subject: Re: Some Thoughts on Z101
Date: Jan 28, 2021
When I was flying IFR regularly in my younger days, I rarely found this to b e necessary and only at large airports like Philly, Boston, etc. At my home field, ELM, it would take the engine longer to warm up than it took me to g et my IFR clearance and taxi clearance. So, I fired up the engine and then u sed the warm-up time to contact clearance delivery and ground and begin taxi . At larger airports where you may have to wait 5-10 minutes for clearance, ha ving one radio on a separate bus makes sense. Much depends on where you fly more often. Matt Sent from my iPad > On Jan 27, 2021, at 10:27 PM, Sebastien wrote: > > =EF=BB > Bob I think a clearance delivery bus with brownout protection is an import ant design goal for any IFR OBAM aircraft. The ability to get the clearance, program it, and then start the engine rather than doing it all with the eng ine running is almost a necessity in my mind. > > I may have enough equipment here that you could unload the testing portion onto me. If this interests you please get in touch and I will order the par ts and assemble and test a system to your specs. > >> On Wed, Jan 27, 2021 at 7:02 PM Robert L. Nuckolls, III <nuckolls.bob@aer oelectric.com> wrote: >>> >>> >>> Attachments: >>> >>> http://forums.matronics.com//files/simplified_boost_topology_619.jpg >> >> A full schematic of the proposed boost >> system would make it easier to 'grok'. >> >> Bob . . . >> >> Un impeachable logic: George Carlin asked, "If black boxes >> survive crashes, why don't they make the whole airplane >> out of that stuff?" >> ========================== ========================== ========================== -========================= ========================== -========================= ========================== ========================== ==================== ________________________________________________________________________________
From: Sebastien <cluros(at)gmail.com>
Date: Jan 28, 2021
Subject: Re: Some Thoughts on Z101
Matthew I suspect your younger days were a long time ago :). Were you programming routes into a navigation system? Setting up SIDs? On Thu, Jan 28, 2021 at 5:32 AM Matthew S. Whiting wrote: > When I was flying IFR regularly in my younger days, I rarely found this t o > be necessary and only at large airports like Philly, Boston, etc. At my > home field, ELM, it would take the engine longer to warm up than it took me > to get my IFR clearance and taxi clearance. So, I fired up the engine an d > then used the warm-up time to contact clearance delivery and ground and > begin taxi. > > At larger airports where you may have to wait 5-10 minutes for clearance, > having one radio on a separate bus makes sense. Much depends on where yo u > fly more often. > > Matt > > Sent from my iPad > > On Jan 27, 2021, at 10:27 PM, Sebastien wrote: > > =EF=BB > Bob I think a clearance delivery bus with brownout protection is an > important design goal for any IFR OBAM aircraft. The ability to get the > clearance, program it, and then start the engine rather than doing it all > with the engine running is almost a necessity in my mind. > > I may have enough equipment here that you could unload the testing portio n > onto me. If this interests you please get in touch and I will order the > parts and assemble and test a system to your specs. > > On Wed, Jan 27, 2021 at 7:02 PM Robert L. Nuckolls, III < > nuckolls.bob(at)aeroelectric.com> wrote: > >> >> >> Attachments: >> >> http://forums.matronics.com//files/simplified_boost_topology_619.jpg >> >> >> A full schematic of the proposed boost >> system would make it easier to 'grok'. >> >> Bob . . . >> >> Un impeachable logic: George Carlin asked, "If black boxes >> survive crashes, why don't they make the whole airplane >> out of that stuff?" >> > ======================== =========== ============= ===================== > ======================== =========== ============= ===================== > ======================== > > ________________________________________________________________________________
From: "Matthew S. Whiting" <m.whiting(at)frontier.com>
Subject: Re: Some Thoughts on Z101
Date: Jan 28, 2021
I learned to fly in a 150 whose radio could either comm or nav, but not both at the same time. =F0=9F=98=82 Most of my instrument flying (other than training in a PA-180) was in my C-1 82 that had only a Garmin 150 GPS. So, all procedures were essentially flow n manually including SIDS and STARs. My 182 did not even have an A/P so I g ot fairly proficient at single pilot IFR hand flown. I plan to put a full Garmin digital panel in the S-21 I am currently buildin g. IFR with today=99s avionics is stupid easy. If you haven=99 t hand flown an NDB approach in moderate turbulence, you really aren=99 t an instrument pilot. =F0=9F=98=82. I actually had to do that on my instru ment checkride. Matt Sent from my iPad > On Jan 28, 2021, at 2:06 PM, Sebastien wrote: > > =EF=BB > Matthew I suspect your younger days were a long time ago :). Were you prog ramming routes into a navigation system? Setting up SIDs? > >> On Thu, Jan 28, 2021 at 5:32 AM Matthew S. Whiting wrote: >> When I was flying IFR regularly in my younger days, I rarely found this t o be necessary and only at large airports like Philly, Boston, etc. At my h ome field, ELM, it would take the engine longer to warm up than it took me t o get my IFR clearance and taxi clearance. So, I fired up the engine and th en used the warm-up time to contact clearance delivery and ground and begin t axi. >> >> At larger airports where you may have to wait 5-10 minutes for clearance, having one radio on a separate bus makes sense. Much depends on where you f ly more often. >> >> Matt >> >> Sent from my iPad >> >>>> On Jan 27, 2021, at 10:27 PM, Sebastien wrote: >>>> >>> =EF=BB >>> Bob I think a clearance delivery bus with brownout protection is an impo rtant design goal for any IFR OBAM aircraft. The ability to get the clearanc e, program it, and then start the engine rather than doing it all with the e ngine running is almost a necessity in my mind. >>> >>> I may have enough equipment here that you could unload the testing porti on onto me. If this interests you please get in touch and I will order the p arts and assemble and test a system to your specs. >>> >>>> On Wed, Jan 27, 2021 at 7:02 PM Robert L. Nuckolls, III <nuckolls.bob@a eroelectric.com> wrote: >>>>> >>>>> >>>>> Attachments: >>>>> >>>>> http://forums.matronics.com//files/simplified_boost_topology_619.jpg >>>> >>>> A full schematic of the proposed boost >>>> system would make it easier to 'grok'. >>>> >>>> Bob . . . >>>> >>>> Un impeachable logic: George Carlin asked, "If black boxes >>>> survive crashes, why don't they make the whole airplane >>>> out of that stuff?" >>>> >>> ========== ========== ==== ====== ========== ========= ________________________________________________________________________________
From: Sebastien <cluros(at)gmail.com>
Date: Jan 28, 2021
Subject: Re: Some Thoughts on Z101
If you put a Garmin navigator in it, you'll want that brownout booster. On Thu, Jan 28, 2021 at 11:45 AM Matthew S. Whiting wrote: > I learned to fly in a 150 whose radio could either comm or nav, but not > both at the same time. =F0=9F=98=82 > > Most of my instrument flying (other than training in a PA-180) was in my > C-182 that had only a Garmin 150 GPS. So, all procedures were essentiall y > flown manually including SIDS and STARs. My 182 did not even have an A/P > so I got fairly proficient at single pilot IFR hand flown. > > I plan to put a full Garmin digital panel in the S-21 I am currently > building. IFR with today=99s avionics is stupid easy. If you have n=99t hand > flown an NDB approach in moderate turbulence, you really aren=99t a n > instrument pilot. =F0=9F=98=82. I actually had to do that on my instrume nt checkride. > > Matt > > Sent from my iPad > > On Jan 28, 2021, at 2:06 PM, Sebastien wrote: > > =EF=BB > Matthew I suspect your younger days were a long time ago :). Were you > programming routes into a navigation system? Setting up SIDs? > > On Thu, Jan 28, 2021 at 5:32 AM Matthew S. Whiting > wrote: > >> When I was flying IFR regularly in my younger days, I rarely found this >> to be necessary and only at large airports like Philly, Boston, etc. At my >> home field, ELM, it would take the engine longer to warm up than it took me >> to get my IFR clearance and taxi clearance. So, I fired up the engine a nd >> then used the warm-up time to contact clearance delivery and ground and >> begin taxi. >> >> At larger airports where you may have to wait 5-10 minutes for clearance , >> having one radio on a separate bus makes sense. Much depends on where y ou >> fly more often. >> >> Matt >> >> Sent from my iPad >> >> On Jan 27, 2021, at 10:27 PM, Sebastien wrote: >> >> =EF=BB >> Bob I think a clearance delivery bus with brownout protection is an >> important design goal for any IFR OBAM aircraft. The ability to get the >> clearance, program it, and then start the engine rather than doing it al l >> with the engine running is almost a necessity in my mind. >> >> I may have enough equipment here that you could unload the testing >> portion onto me. If this interests you please get in touch and I will or der >> the parts and assemble and test a system to your specs. >> >> On Wed, Jan 27, 2021 at 7:02 PM Robert L. Nuckolls, III < >> nuckolls.bob(at)aeroelectric.com> wrote: >> >>> >>> >>> Attachments: >>> >>> http://forums.matronics.com//files/simplified_boost_topology_619.jpg >>> >>> >>> A full schematic of the proposed boost >>> system would make it easier to 'grok'. >>> >>> Bob . . . >>> >>> Un impeachable logic: George Carlin asked, "If black boxes >>> survive crashes, why don't they make the whole airplane >>> out of that stuff?" >>> >> ========== ========== === ======= ========== ======= == >> >> ________________________________________________________________________________
Subject: Re: Some Thoughts on Z101
From: Rick Beebe <rick(at)beebe.org>
Date: Jan 28, 2021
Yes. My G3X Touch screen survives engine start but the GTN 750 reboots. I need to figure out a brownout booster too. When I was getting my instrument rating I had a Piper Warrior that I upgraded with a GNS-430. I specifically removed the ADF from that plane so they wouldn't make me fly any NDB approaches on the checkride. :) --Rick On 1/28/2021 2:52 PM, Sebastien wrote: > If you put a Garmin navigator in it, you'll want that brownout booster. > > On Thu, Jan 28, 2021 at 11:45 AM Matthew S. Whiting > > wrote: > > > I plan to put a full Garmin digital panel in the S-21 I am > currently building. IFR with todays avionics is stupid easy. If > you havent hand flown an NDB approach in moderate turbulence, you > really arent an instrument pilot. . I actually had to do that > on my instrument checkride. > ________________________________________________________________________________
Subject: Re: Some Thoughts on Z101
From: Kelly McMullen <kellym(at)aviating.com>
Date: Jan 28, 2021
The two most common radios for training aircraft in the '70s, at least before Cessna bought ARC, were the Escort 110 from Narco, and the King KX145. Both used a single shared receiver for nav and com reception. The fancier radios like the KX170B were refered to something like a 1 and 1/2 because they had one transmitter and two receivers, so you could listen to com freq and keep nav signal at same time. And Matt, flying an NDB was no big deal if you had learned the tricks of the trade. Flying colored airways was a bigger deal because you were tracking often 50 nm off one beacon before getting the next. Back then I flew IFR with one digital nav com and a manually tuned ADF, no transponder, no autopilot and AN Gyros powered by venturis etc. So you had to do identifying turns for the controller if picked up anywhere but departing an airport. Controllers had to do more work too. No strip printers, you had to call Center and have them issue the clearance, write it on a strip, then issue to the aircraft. Moving maps and stored procedures do make it much easier. Thankfully the old AN approaches disappeared a few years before I started flying. Kelly On 1/28/2021 12:37 PM, Matthew S. Whiting wrote: > I learned to fly in a 150 whose radio could either comm or nav, but not > both at the same time. > > Most of my instrument flying (other than training in a PA-180) was in my > C-182 that had only a Garmin 150 GPS. So, all procedures were > essentially flown manually including SIDS and STARs. My 182 did not > even have an A/P so I got fairly proficient at single pilot IFR hand flown. > > I plan to put a full Garmin digital panel in the S-21 I am currently > building. IFR with todays avionics is stupid easy. If you havent > hand flown an NDB approach in moderate turbulence, you really arent an > instrument pilot. . I actually had to do that on my instrument checkride. > > Matt > > Sent from my iPad > >> On Jan 28, 2021, at 2:06 PM, Sebastien wrote: >> >> >> Matthew I suspect your younger days were a long time ago :). Were you >> programming routes into a navigation system? Setting up SIDs? >> >> On Thu, Jan 28, 2021 at 5:32 AM Matthew S. Whiting >> > wrote: >> >> When I was flying IFR regularly in my younger days, I rarely found >> this to be necessary and only at large airports like Philly, >> Boston, etc. At my home field, ELM, it would take the engine >> longer to warm up than it took me to get my IFR clearance and taxi >> clearance. So, I fired up the engine and then used the warm-up >> time to contact clearance delivery and ground and begin taxi. >> >> At larger airports where you may have to wait 5-10 minutes for >> clearance, having one radio on a separate bus makes sense. Much >> depends on where you fly more often. >> >> Matt >> >> Sent from my iPad >> >>> On Jan 27, 2021, at 10:27 PM, Sebastien >> > wrote: >>> >>> >>> Bob I think a clearance delivery bus with brownout protection is >>> an important design goal for any IFR OBAMaircraft. The ability >>> to get the clearance, program it, and then start the engine >>> rather than doing it all with the engine running is almost a >>> necessity in my mind. >>> >>> I may have enough equipment here that you could unload the >>> testing portion onto me. If this interests you please get in >>> touch and I will order the parts and assemble and test a system >>> to your specs. >>> >>> On Wed, Jan 27, 2021 at 7:02 PM Robert L. Nuckolls, III >>> >> > wrote: >>> >>>> >>>> >>>> Attachments: >>>> >>>> http://forums.matronics.com//files/simplified_boost_topology_619.jpg >>>> <http://forums.matronics.com//files/simplified_boost_topology_619.jpg> >>>> >>> >>> A full schematic of the proposed boost >>> system would make it easier to 'grok'. >>> >>> __ >>> >>> __ Bob . . . >>> >>> Un impeachable logic: George Carlin asked, "If black boxes >>> survive crashes, why don't they make the whole airplane >>> out of that stuff?" >>> >>> ========== ========== ========== ========== ========== >> ________________________________________________________________________________
Subject: Re: Some Thoughts on Z101
From: Charlie England <ceengland7(at)gmail.com>
Date: Jan 28, 2021
I posted here a few months ago about my brownout solution on my GRT EFIS. The GRT, like most flight critical electronics these days, has multiple diode-isolated inputs. I just tapped the power feed wire to the EFIS, used it to also power a small 4A boost converter (small enough to fit in a pill bottle) that was set to step up voltage to ~11.5V (minimum boost to reliably keep the EFIS online). The booster output feeds into one of the extra power inputs on the EFIS. During cranking, the booster will keep voltage into the spare power terminal at 12V; in normal operation, it's powered and its output is the same as bus voltage. Slight parasitic draw during operation, but extremely simple. One really cheap device & two wires. Charlie On 1/28/2021 3:04 PM, Rick Beebe wrote: > Yes. My G3X Touch screen survives engine start but the GTN 750 > reboots. I need to figure out a brownout booster too. > > When I was getting my instrument rating I had a Piper Warrior that I > upgraded with a GNS-430. I specifically removed the ADF from that > plane so they wouldn't make me fly any NDB approaches on the checkride. :) > > --Rick > > On 1/28/2021 2:52 PM, Sebastien wrote: >> If you put a Garmin navigator in it, you'll want that brownout booster. >> >> On Thu, Jan 28, 2021 at 11:45 AM Matthew S. Whiting >> > wrote: >> >> >> I plan to put a full Garmin digital panel in the S-21 I am >> currently building. IFR with todays avionics is stupid easy. >> If you havent hand flown an NDB approach in moderate turbulence, >> you really arent an instrument pilot. . I actually had to do >> that on my instrument checkride. >> > -- This email has been checked for viruses by Avast antivirus software. https://www.avast.com/antivirus ________________________________________________________________________________
Subject: Re: Some Thoughts on Z101
From: "Eric Page" <edpav8r(at)yahoo.com>
Date: Jan 28, 2021
Thanks, fellas. I knew my thinking was a bit muddy and this was the best place for effective filtering. I'll try to answer your questions as best I can. user9253 wrote: > Eric, what advantage does your proposal have over Z-101? A bit of simplification (one less AUX BUS feed path) and the Brown-Out Booster continuously powered. > Why eliminate the normal diode feed path to the aux bus? Not eliminated; replaced with the booster's internal rectifier. Electrons still come from the same place, but are routed differently. > You can rewire the brownout booster whether the diode is there or not. Indeed, but you would end up with duplicate normal feed paths: one through the diode bridge and one through the booster. > It is a bad idea to have two fuses in series. Z-101 does not fuse the aux bus. Yes, I agree. I forgot that the appliances on the AUX BUS would be individually fused. > Is a brown out booster really necessary? Doesn't your EFIS have an internal backup battery? Internal, no. Dynon sells a separate backup battery of undisclosed chemistry, which creates a testing/maintenance burden. I would like to wire the plane such that the EFIS and a few other items share a separate bus (AUX BUS?) that is redundantly powered. The booster solves the reboot-on-engine-start problem. With those two features, Dynon's $175 battery isn't needed. > Why even supply a whole bus with brownout protection? > Why is a separate aux bus needed? > Just connect one or two loads to a brownout booster using dual diodes. OK, I'm beginning to see the wisdom of your questions. I plan to power the EFIS, GPS antenna, comm radio, landing light and Hobbs (minimum/emergency VFR equipment) from the AUX BUS. Given that the EFIS, and perhaps the GPS antenna, are the only things I need to protect against brown-out, I could leave Z101 alone and just feed them from the AUX BUS through the booster, much as Charlie described above. ---------- nuckolls.bob(at)aeroelect wrote: > Can you elaborate on your design goals? - Redundant electrical supply for the engine (auto conversion with ECU and electronic ignition) - Failure of one alternator or regulator cannot cause a remote AOG (back-country airplane) - Redundant electrical supply for minimum/emergency VFR equipment (EFIS, GPS, comm, landing light, Hobbs) - Brown-out-proof EFIS installation > Re brownout booster: I'm pondering a plan-d . . . or are we up to 'e'? There are dozens of suitable step up devices on the market but I have no knowledge of their 'spool up' times nor can I personally vouch for their specifications without putting the device under test on the bench and doing some measurements. > > I think I have a way to craft a brown-out booster of KNOWN performance thus avoiding the risks for incorporating the Chinese unknowns. Watch this space. > > A full schematic of the proposed boost system would make it easier to 'grok'. A schematic for the booster I've been fiddling with is attached. It uses TI's LM3481: https://tinyurl.com/yy9twn9t+ It will grunt 7A all day. It takes about 12ms for the output to come up into a 2A load when it's powered up from cold, and if it's already powered, output never falls more than a few mV below 12V when input is cut from 14V to 10V. It seems to work well but it's never going to be a drop-in, jellybean solution; it's all SMD, including fine-pitch and leadless components. Cheers, Eric Read this topic online here: http://forums.matronics.com/viewtopic.php?p=500454#500454 Attachments: http://forums.matronics.com//files/brown_out_booster_rev_b_168.pdf ________________________________________________________________________________
Subject: Re: Some Thoughts on Z101
From: "prestonkavanagh" <preston.kavanagh(at)gmail.com>
Date: Jan 29, 2021
This has turned into a thread on alternative ways of providing a brownout bus. As discussed in the past, we don't necessarily need one. Accepting that my Garmin GPS-Comm might cutout on startup, and that there are situations where this would be inconvenient, in my Z101 design I'm including a booster (see attached photos). I built two small boxes and both work. Now I pick one and decide whether to brown-out protect a circuit or add a small brownout bus. Yes, I should add a diode. Other comments? Skip this complication entirely? -------- PBK3 PA-12, BD-4, RV6a, gliders Read this topic online here: http://forums.matronics.com/viewtopic.php?p=500465#500465 Attachments: http://forums.matronics.com//files/boost2_166.png http://forums.matronics.com//files/boost1_133.png ________________________________________________________________________________
From: Charlie England <ceengland7(at)gmail.com>
Date: Jan 29, 2021
Subject: Re: Some Thoughts on Z101
On Fri, Jan 29, 2021 at 4:25 PM prestonkavanagh wrote: > preston.kavanagh(at)gmail.com> > > This has turned into a thread on alternative ways of providing a brownout > bus. As discussed in the past, we don't necessarily need one. Accepting > that my Garmin GPS-Comm might cutout on startup, and that there are > situations where this would be inconvenient, in my Z101 design I'm > including a booster (see attached photos). I built two small boxes and > both work. Now I pick one and decide whether to brown-out protect a > circuit or add a small brownout bus. Yes, I should add a diode. Other > comments? Skip this complication entirely? > > -------- > PBK3 > PA-12, BD-4, RV6a, gliders > > Have you tested the 'dual' version for proper boost operation? The data sheet hints that it is a buck converter; not boost. As to whether to include brownout: If there's no delay, or the delay in bootup of your installed devices isn't inconvenient, then don't install one. I had hoped that a new battery would solve my issue, but it didn't. I installed one because my older EFIS equipment requires several minutes to fully boot, and often will 'lock up' during cranking, requiring me to shut off the entire airframe to enable a re-start of the EFIS equipment. Livable in winter, but in summer, it just wastes fuel to sit idling so long. (Almost all my flying is from uncontrolled fields, so clearance delivery isn't a factor for me.) Charlie ________________________________________________________________________________
From: Roger Evenson <revenson3(at)gmail.com>
Date: Jan 29, 2021
Subject: Poor Man's Battery Tester
I built Bob's "Poor Man's Battery Capacity Tester". Somehow I got the impression the relay would 'drop' out and thus turn off the lamp and clock when the battery reached about 10V. But after re-reading "Battery Replacement: A Plan for Throwing in the Towel", that expectation is not written there. My experience has been that it turns off when the battery reaches about 4V. Odyssey Load Test Procedure and Reconditioning Procedures recommend ending the test at 10V. I've done this, but it requires my presence when the battery nears 10V. Is there a way to modify this circuit so that it will 'turn off' when the battery reaches 10V? Or, is there a commercially available tester that would load a battery at 4 amps and record the time required until the battery reaches 10V? Roger. ________________________________________________________________________________
Subject: Re: bench power
From: "bobnoffs" <icubob(at)gmail.com>
Date: Jan 30, 2021
while waiting for the power supply i ordered i came across some tutorials [u tube] about taking an atx [the power supply in a computer] and converting it to a bench power. easy, and a fun project. i would take a guess that dell or H.P. or what ever computer mfgr.. has a power supply of 'higher standards' than a $65 bench power on ebay. anyway.........everyone has an old computer lying around and i will soon have another bench power made from an atx . 12 volts, that's it, although there are leads for 3.3 and 5.08 volts i have no need. at 12 amps it is plenty for me. Read this topic online here: http://forums.matronics.com/viewtopic.php?p=500469#500469 ________________________________________________________________________________
Subject: Re: Poor Man's Battery Tester
From: "user9253" <fransew(at)gmail.com>
Date: Jan 30, 2021
Bob's Poor Man's Battery Tester http://www.aeroelectric.com/articles/battest.pdf is supposed to shut off automatically at 10 volts. Did you change any parts values? Did you use a 10 volt zener? -------- Joe Gores Read this topic online here: http://forums.matronics.com/viewtopic.php?p=500470#500470 ________________________________________________________________________________
Subject: Re: Poor Man's Battery Tester
From: "user9253" <fransew(at)gmail.com>
Date: Jan 30, 2021
Trouble shooting the battery tester: Is the zener diode installed backwards? If one end of the zener diode is disconnected, does the relay drop out? If not, has the correct transistor been used? Has the transistor been connected correctly? -------- Joe Gores Read this topic online here: http://forums.matronics.com/viewtopic.php?p=500471#500471 ________________________________________________________________________________
Subject: Re: Poor Man's Battery Tester
From: "Eric Page" <edpav8r(at)yahoo.com>
Date: Jan 30, 2021
It sounds like your circuit may be shutting off at the natural dropout voltage of the relay, which suggests that full battery voltage is reaching the relay's coil. In addition to Joe's suggestions, check that the Zener diode is actually a 10V device, and not something near 3V, and that it hasn't failed (diodes can fail short circuit). Read this topic online here: http://forums.matronics.com/viewtopic.php?p=500472#500472 ________________________________________________________________________________
Subject: Re: Some Thoughts on Z101
From: "Eric Page" <edpav8r(at)yahoo.com>
Date: Jan 30, 2021
Ceengland wrote: > Have you tested the 'dual' version for proper boost operation? The data sheet hints that it is a buck converter; not boost. It's buck-boost; note the two inductors. The LM2577 does boost and the LM2596 does buck. https://www.ti.com/lit/ds/symlink/lm2577.pdf https://www.ti.com/lit/ds/symlink/lm2596.pdf Read this topic online here: http://forums.matronics.com/viewtopic.php?p=500474#500474 ________________________________________________________________________________
From: Steve Stearns <steve(at)tomasara.com>
Date: Jan 30, 2021
Subject: R.e. Poor Man's Battery Tester
You said, "My experience has been that it turns off when the battery reaches about 4V." I built one and it shuts off right at 11 volts which is what I was expecting. I think you have a wiring error or some other issue. Steve. Steve. ________________________________________________________________________________
From: Roger Evenson <revenson3(at)gmail.com>
Date: Jan 30, 2021
Subject: Re: Poor Man's Battery Tester
Bought new resistors and 10V zener. Rewired carefully on a test board. This time it kicked off, but at a higher voltage than I wanted (12V). The zener was a NTE5019A, 10.0V +/-5%, 1/2 watt. Isn't that more like +/-20%? Cost all of $0.79. I'll go back Monday, see if I can get a +/-1% zener, and ask about a higher quality brand. Thanks for your comments. One more question...does the wattage make a difference in this circuit? Roger. On Fri, Jan 29, 2021, 9:23 PM Roger Evenson wrote: > I built Bob's "Poor Man's Battery Capacity Tester". > > Somehow I got the impression the relay would 'drop' out and thus turn off > the lamp and clock when the battery reached about 10V. But after > re-reading "Battery Replacement: A Plan for Throwing in the Towel", that > expectation is not written there. My experience has been that it turns > off when the battery reaches about 4V. > > Odyssey Load Test Procedure and Reconditioning Procedures recommend ending > the test at 10V. I've done this, but it requires my presence when the > battery nears 10V. > > Is there a way to modify this circuit so that it will 'turn off' when the > battery reaches 10V? > > Or, is there a commercially available tester that would load a battery at > 4 amps and record the time required until the battery reaches 10V? > Roger. > ________________________________________________________________________________
Subject: Re: Poor Man's Battery Tester
From: "user9253" <fransew(at)gmail.com>
Date: Jan 30, 2021
The problem is not the tolerance of the zener. The resistors affect the dropout voltage too. Try a 9 volt zener. They are cheap. Buy a few different values to experiment with. Wattage does not affect the circuit very much. 1/2 watt is good. -------- Joe Gores Read this topic online here: http://forums.matronics.com/viewtopic.php?p=500477#500477 ________________________________________________________________________________
Subject: Re: Poor Man's Battery Tester
From: Charles Davis <charlesdavis(at)iuncapped.co.za>
Date: Jan 31, 2021
You may have better luck getting 2x5v 1% zeners than a 10v 1% - just wire them in series Charles On 31/01/21 03:10 am, Roger Evenson wrote: > Bought new resistors and 10V zener. Rewired carefully on a test board. > This time it kicked off, but at a higher voltage than I wanted (12V). > > The zener was a NTE5019A, 10.0V +/-5%, 1/2 watt. Isn't that more like > +/-20%? Cost all of $0.79. > > I'll go back Monday, see if I can get a +/-1% zener, and ask about a > higher quality brand. > > Thanks for your comments. One more question...does the wattage make a > difference in this circuit? > Roger. > ________________________________________________________________________________
Subject: Re: Poor Man's Battery Tester
From: Dick Tasker <dick(at)thetaskerfamily.com>
Date: Jan 31, 2021
Unfortunately they don't make 5V zeners - only 4.7V or 5.1V. The turnoff voltage is also affected by the actual dropout current of the relay as well. The less current it takes to hold in the relay, the lower the voltage it drops out at. And conversely, the higher the current it takes to hold the relay on, the higher the voltage it drops out at. The gain of the 2N3904 also affects the turnoff voltage as well as the actual resistor values. However, none of these should make a huge difference. If you have a variable power supply you can experiment and check things. If not, you will just have to try various different components. Do NOT change the top 470 ohm resistor very much. If you reduce it too much you will burn out the transistor or zener. With the circuit elements shown the nominal turnoff voltage should be ~10.5V. Finally, how do you know that it turned off at 12V? The only real way to know this is to watch it with a voltmeter WHEN the circuit turns off. If you go back later to check things after the circuit turned off, the battery voltage will have recovered somewhat and it is quite possible that it will show 12V on a voltmeter when it is sitting there with no load. Dick Tasker Charles Davis wrote: > > You may have better luck getting 2x5v 1% zeners than a 10v 1% - just wire them in series > > Charles > > > On 31/01/21 03:10 am, Roger Evenson wrote: >> Bought new resistors and 10V zener. Rewired carefully on a test board. This time it kicked off, but at a higher voltage than I wanted (12V). >> >> The zener was a NTE5019A, 10.0V +/-5%, 1/2 watt. Isn't that more like +/-20%? Cost all of $0.79. >> >> I'll go back Monday, see if I can get a +/-1% zener, and ask about a higher quality brand. >> >> Thanks for your comments. One more question...does the wattage make a difference in this circuit? >> Roger. >> > > ________________________________________________________________________________
Date: Jan 31, 2021
From: Rod Smith <rodsmith52(at)yahoo.com>
Subject: LED question
I am installing dual lightspeed ignitions in my Bearhawk. 5100H1LC LEDs are speced as panel mounts to indicate when one of the ignitions is not powered. The LEDs are powered off a pin on the ignition output connectors. The LEDS are described as 1.8V low current, the red lead is indicated as positive. To test them I used a new AA battery reading 1.6V. I did not get even a faint glow, so wondering if they are bad or if they need the full 1.8V or more to produce visible output. Rod ________________________________________________________________________________
From: Kent or Jackie Ashton <kjashton(at)vnet.net>
Subject: Re: LED question
Date: Jan 31, 2021
The spec says will operate off 5V to 12V without an external resistor". You just need more juice. See datasheet here https://www.digikey.com/en/products/detail/visual-communications-company-vcc/5100H1LC/59900 -Kent > On Jan 31, 2021, at 1:37 PM, Rod Smith wrote: > > I am installing dual lightspeed ignitions in my Bearhawk. 5100H1LC LEDs are speced as panel mounts to indicate when one of the ignitions is not powered. The LEDs are powered off a pin on the ignition output connectors. The LEDS are described as 1.8V low current, the red lead is indicated as positive. To test them I used a new AA battery reading 1.6V. I did not get even a faint glow, so wondering if they are bad or if they need the full 1.8V or more to produce visible output. > > Rod ________________________________________________________________________________
Subject: Re: LED question
From: "user9253" <fransew(at)gmail.com>
Date: Jan 31, 2021
Do NOT test an LED with a battery without a series resistor. Otherwise the LED will be destroyed. You did not destroy yours because the battery voltage was not high enough. But if the battery was 1.8 volts or more, the LED would be destroyed without a series resistor. There are online calculators to determine the resistor value to use. -------- Joe Gores Read this topic online here: http://forums.matronics.com/viewtopic.php?p=500483#500483 ________________________________________________________________________________
Subject: Re: LED question
From: "user9253" <fransew(at)GMAIL.COM>
Date: Jan 31, 2021
I was wrong and Kent is right. The 5100H1LC has a Built-in resistor chip. But if an LED does not have a built in resistor, then one must be added. -------- Joe Gores Read this topic online here: http://forums.matronics.com/viewtopic.php?p=500484#500484 ________________________________________________________________________________
Date: Jan 31, 2021
From: "Robert L. Nuckolls, III" <nuckolls.bob(at)aeroelectric.com>
Subject: Re: Poor Man's Battery Tester
At 10:24 AM 1/31/2021, you wrote: >Tasker > >Unfortunately they don't make 5V zeners - only 4.7V or 5.1V. > >The turnoff voltage is also affected by the >actual dropout current of the relay as >well.=C2 The less current it takes to hold in the >relay, the lower the voltage it drops out >at.=C2 And conversely, the higher the current it >takes to hold the relay on, the higher the voltage it drops out at. My apologies guys . . . I intended to update that article years ago for just problems you're wrestling with. I sat down today and finished an update that I started some time ago. https://tinyurl.com/y44wl64t The 'sloppy' zener is replaced with a precision adjustable zener. The test set's trip point is calibrated by one of two methods. Use precision, 1% resistors to bias up the reference port of the zener -OR- you can use 5% resistors and a potentiometer to fabricate an adjustable trip point feature. Either technique yields a predictable and stable set point for disconnecting the test load. Again, please forgive my tardiness. I'll also remind readers of a sophisticated battery test option in the form of West Mountain Radio's CBA series, computer driven battery testers. https://tinyurl.com/mas7ea9 The one I have right now is probably the 4th in a series that I've owned over the last 20 years, give or take. They are versatile and accurate. But for most of our brother's need to peek into Conditions for Continued Airworthiness, the Po' Man's battery-runner-downer is of excellent value. Bob . . . Un impeachable logic: George Carlin asked, "If black boxes survive crashes, why don't they make the whole airplane out of that stuff?" ________________________________________________________________________________
Date: Feb 01, 2021
From: Jeff Luckey <jluckey(at)pacbell.net>
Subject: Re: Poor Man's Battery Tester
Bob, Is that the correct symbol for the 1N4001?=C2- Is it a zener? -Jeff nuckolls.bob(at)aeroelectric.com> wrote: At 10:24 AM 1/31/2021, you wrote: --> AeroElectric-List messageposted by: Dick Tasker Unfortunately they don't make 5V zeners - only 4.7V or 5.1V. The turnoff voltage is also affected by the actual dropout current of there lay as well.=C3=82=C2- The less current it takes to hold in the relay, th elower the voltage it drops out at.=C3=82=C2- And conversely, the higher thecurrent it takes to hold the relay on, the higher the voltage it dropsou t at. =C2- My apologies guys . . . I intended to update that =C2- article years ago for just problems you're wrestling =C2- with. I sat down today and finished an update that I =C2- started some time ago. https://tinyurl.com/y44wl64t =C2- The 'sloppy' zener is replaced with a precision =C2- adjustable zener. The test set's trip point is =C2- calibrated by one of two methods. Use precision, =C2- 1% resistors to bias up the reference port of the =C2- zener -OR- you can use 5% resistors and a potentiometer =C2- to fabricate an adjustable trip point feature. =C2- Either technique yields a predictable and stable =C2- set point for disconnecting the test load. Again, =C2- please forgive my tardiness. =C2- I'll also remind readers of a sophisticated =C2- battery test option in the form of West Mountain =C2- Radio's CBA series, computer driven battery =C2- testers. https://tinyurl.com/mas7ea9 =C2- The one I have right now is probably the 4th =C2- in a series that I've owned over the last 20 =C2- years, give or take. They are versatile and =C2- accurate. =C2- But for most of our brother's need to peek =C2- into Conditions for Continued Airworthiness, =C2- the Po' Man's battery-runner-downer is of =C2- excellent value. =C2- Bob . . . =C2- Un impeachable logic: George Carlin asked, "If blackboxes =C2- survive crashes, why don't they make the whole airplane =C2- out of that stuff?" ________________________________________________________________________________
Date: Feb 01, 2021
From: Jeff Luckey <jluckey(at)pacbell.net>
Subject: Re: Poor Man's Battery Tester
and do you need the second 470 ohm resistor between the transistor base & the LM431? -Jeff ell.net> wrote: Bob, Is that the correct symbol for the 1N4001?=C2- Is it a zener? -Jeff nuckolls.bob(at)aeroelectric.com> wrote: At 10:24 AM 1/31/2021, you wrote: --> AeroElectric-List messageposted by: Dick Tasker Unfortunately they don't make 5V zeners - only 4.7V or 5.1V. The turnoff voltage is also affected by the actual dropout current of there lay as well.=C3=82=C2- The less current it takes to hold in the relay, th elower the voltage it drops out at.=C3=82=C2- And conversely, the higher thecurrent it takes to hold the relay on, the higher the voltage it dropsou t at. =C2- My apologies guys . . . I intended to update that =C2- article years ago for just problems you're wrestling =C2- with. I sat down today and finished an update that I =C2- started some time ago. https://tinyurl.com/y44wl64t =C2- The 'sloppy' zener is replaced with a precision =C2- adjustable zener. The test set's trip point is =C2- calibrated by one of two methods. Use precision, =C2- 1% resistors to bias up the reference port of the =C2- zener -OR- you can use 5% resistors and a potentiometer =C2- to fabricate an adjustable trip point feature. =C2- Either technique yields a predictable and stable =C2- set point for disconnecting the test load. Again, =C2- please forgive my tardiness. =C2- I'll also remind readers of a sophisticated =C2- battery test option in the form of West Mountain =C2- Radio's CBA series, computer driven battery =C2- testers. https://tinyurl.com/mas7ea9 =C2- The one I have right now is probably the 4th =C2- in a series that I've owned over the last 20 =C2- years, give or take. They are versatile and =C2- accurate. =C2- But for most of our brother's need to peek =C2- into Conditions for Continued Airworthiness, =C2- the Po' Man's battery-runner-downer is of =C2- excellent value. =C2- Bob . . . =C2- Un impeachable logic: George Carlin asked, "If blackboxes =C2- survive crashes, why don't they make the whole airplane =C2- out of that stuff?" ________________________________________________________________________________
Subject: Re: Poor Man's Battery Tester
From: C&K <yellowduckduo(at)gmail.com>
Date: Feb 01, 2021
I just use a 12 volt automotive inverter to drive 120 volt lamps as a load. The cigarette lighter inverters that I have automatically shut down right around 10 volts input. Close enough for me. Ken On 31/01/2021 11:09 PM, Robert L. Nuckolls, III wrote: > At 10:24 AM 1/31/2021, you wrote: >> >> >> Unfortunately they don't make 5V zeners - only 4.7V or 5.1V. >> >> The turnoff voltage is also affected by the actual dropout current of >> the relay as well. The less current it takes to hold in the relay, >> the lower the voltage it drops out at. And conversely, the higher >> the current it takes to hold the relay on, the higher the voltage it >> drops out at. > > My apologies guys . . . I intended to update that > article years ago for just problems you're wrestling > with. I sat down today and finished an update that I > started some time ago. > > https://tinyurl.com/y44wl64t > > The 'sloppy' zener is replaced with a precision > adjustable zener. The test set's trip point is > calibrated by one of two methods. Use precision, > 1% resistors to bias up the reference port of the > zener -OR- you can use 5% resistors and a potentiometer > to fabricate an adjustable trip point feature. > > Either technique yields a predictable and stable > set point for disconnecting the test load. Again, > please forgive my tardiness. > > I'll also remind readers of a sophisticated > battery test option in the form of West Mountain > Radio's CBA series, computer driven battery > testers. > > https://tinyurl.com/mas7ea9 > > The one I have right now is probably the 4th > in a series that I've owned over the last 20 > years, give or take. They are versatile and > accurate. > > But for most of our brother's need to peek > into Conditions for Continued Airworthiness, > the Po' Man's battery-runner-downer is of > excellent value. > > Bob . . . > > Un impeachable logic: George Carlin asked, "If black boxes > survive crashes, why don't they make the whole airplane > out of that stuff?" > ________________________________________________________________________________
Subject: Re: Poor Man's Battery Tester
From: "user9253" <fransew(at)gmail.com>
Date: Feb 01, 2021
Bob, clocks that run off from 120 VAC are rare. I suggest that the AC clock be replaced by a 1.5 volt battery operated clock. Some experimenters might prefer to power a Mini Quartz Clock Movement by the 12 volt battery that is being tested (with voltage reduced to 1.5) The clock and relay and circuit board could all be contained in one enclosure. -------- Joe Gores Read this topic online here: http://forums.matronics.com/viewtopic.php?p=500490#500490 ________________________________________________________________________________
Date: Feb 01, 2021
From: rd2 <rd2(at)dejazzd.com>
Subject: Re: Poor Man's Battery Tester
How about a (readily available) AC hour meter? Admittedly, it could give up to 1/10 (6 min) less reading. Rumen ----- Original Message ----- From: user9253 <fransew(at)gmail.com> Subject: AeroElectric-List: Re: Poor Man's Battery Tester Bob, clocks that run off from 120 VAC are rare. I suggest that the AC clock be replaced by a 1.5 volt battery operated clock. Some experimenters might prefer to power a Mini Quartz Clock Movement by the 12 volt battery that is being tested (with voltage reduced to 1.5) The clock and relay and circuit board could all be contained in one enclosure. -------- Joe Gores Read this topic online here: http://forums.matronics.com/viewtopic.php?p=500490#500490 ________________________________________________________________________________
From: Earl Schroeder <n233ee(at)gmail.com>
Subject: Re: Poor Man's Battery Tester
Date: Feb 01, 2021
I wonder if something like this would be useful? https://www.mpja.com/02-01-21.asp?r=326752&s=39 > On Jan 31, 2021, at 10:20 PM, Robert L. Nuckolls, III <nuckolls.bob@aeroel ectric.com> wrote: > > =EF=BB At 10:24 AM 1/31/2021, you wrote: y.com> >> >> Unfortunately they don't make 5V zeners - only 4.7V or 5.1V. >> >> The turnoff voltage is also affected by the actual dropout current of the relay as well.=C3=82 The less current it takes to hold in the relay, the l ower the voltage it drops out at.=C3=82 And conversely, the higher the curr ent it takes to hold the relay on, the higher the voltage it drops out at. > > My apologies guys . . . I intended to update that > article years ago for just problems you're wrestling > with. I sat down today and finished an update that I > started some time ago. > > https://tinyurl.com/y44wl64t > > The 'sloppy' zener is replaced with a precision > adjustable zener. The test set's trip point is > calibrated by one of two methods. Use precision, > 1% resistors to bias up the reference port of the > zener -OR- you can use 5% resistors and a potentiometer > to fabricate an adjustable trip point feature. > > Either technique yields a predictable and stable > set point for disconnecting the test load. Again, > please forgive my tardiness. > > I'll also remind readers of a sophisticated > battery test option in the form of West Mountain > Radio's CBA series, computer driven battery > testers. > > https://tinyurl.com/mas7ea9 > > The one I have right now is probably the 4th > in a series that I've owned over the last 20 > years, give or take. They are versatile and > accurate. > > But for most of our brother's need to peek > into Conditions for Continued Airworthiness, > the Po' Man's battery-runner-downer is of > excellent value. > > Bob . . . > > Un impeachable logic: George Carlin asked, "If black boxes > survive crashes, why don't they make the whole airplane > out of that stuff?" ________________________________________________________________________________
Subject: Re: Poor Man's Battery Tester
From: Charles Davis <charlesdavis(at)iuncapped.co.za>
Date: Feb 01, 2021
Bob Is it right that the clock only runs when the battery is charging ? Charles On 01/02/21 08:18 am, Jeff Luckey wrote: > Bob, > > Is that the correct symbol for the 1N4001?=C2- Is it a zener? > > > -Jeff > > wrote: > > > At 10:24 AM 1/31/2021, you wrote: >> >> >> Unfortunately they don't make 5V zeners - only 4.7V or 5.1V. >> >> The turnoff voltage is also affected by the actual dropout current of >> the relay as well.=C3=82=C2- The less current it takes to hold in th e relay, >> the lower the voltage it drops out at.=C3=82=C2- And conversely, the higher >> the current it takes to hold the relay on, the higher the voltage it >> drops out at. > > =C2- My apologies guys . . . I intended to update that > =C2- article years ago for just problems you're wrestling > =C2- with. I sat down today and finished an update that I > =C2- started some time ago. > > https://tinyurl.com/y44wl64t > > =C2- The 'sloppy' zener is replaced with a precision > =C2- adjustable zener. The test set's trip point is > =C2- calibrated by one of two methods. Use precision, > =C2- 1% resistors to bias up the reference port of the > =C2- zener -OR- you can use 5% resistors and a potentiometer > =C2- to fabricate an adjustable trip point feature. > > =C2- Either technique yields a predictable and stable > =C2- set point for disconnecting the test load. Again, > =C2- please forgive my tardiness. > > =C2- I'll also remind readers of a sophisticated > =C2- battery test option in the form of West Mountain > =C2- Radio's CBA series, computer driven battery > =C2- testers. > > https://tinyurl.com/mas7ea9 > > =C2- The one I have right now is probably the 4th > =C2- in a series that I've owned over the last 20 > =C2- years, give or take. They are versatile and > =C2- accurate. > > =C2- But for most of our brother's need to peek > =C2- into Conditions for Continued Airworthiness, > =C2- the Po' Man's battery-runner-downer is of > =C2- excellent value. > > =C2- Bob . . . > > =C2- Un impeachable logic: George Carlin asked, "If black boxes > =C2- survive crashes, why don't they make the whole airplane > =C2- out of that stuff?" > ________________________________________________________________________________
From: Roger & Jean <rnjcurtis(at)charter.net>
Subject: Re: Poor Man's Battery Tester
Date: Feb 01, 2021
Sm9lLCANCg0KQSBtZXJlIDUwIHllYXJzLCB3ZSBhbGwgaGFkIG9uZSBvZiB0aGVzZSBjbG9ja3Mg b24gb3VyIGJlZHNpZGUgbmlnaHQgc3RhbmQuIFdhbG1hcnQgc3RpbGwgc2VsbHMgdGhlbS4NCg0K Um9nZXINCg0KRnJvbTogdXNlcjkyNTMNClNlbnQ6IE1vbmRheSwgRmVicnVhcnkgMSwgMjAyMSA5 OjA1IEFNDQpUbzogYWVyb2VsZWN0cmljLWxpc3RAbWF0cm9uaWNzLmNvbQ0KU3ViamVjdDogQWVy b0VsZWN0cmljLUxpc3Q6IFJlOiBQb29yIE1hbidzIEJhdHRlcnkgVGVzdGVyDQoNCi0tPiBBZXJv RWxlY3RyaWMtTGlzdCBtZXNzYWdlIHBvc3RlZCBieTogInVzZXI5MjUzIiA8ZnJhbnNld0BnbWFp bC5jb20+DQoNCkJvYiwgY2xvY2tzIHRoYXQgcnVuIG9mZiBmcm9tIDEyMCBWQUMgYXJlIHJhcmUu IA0KSSBzdWdnZXN0IHRoYXQgdGhlIEFDIGNsb2NrIGJlIHJlcGxhY2VkIGJ5IGEgMS41IHZvbHQg YmF0dGVyeSBvcGVyYXRlZCBjbG9jay4NClNvbWUgZXhwZXJpbWVudGVycyBtaWdodCBwcmVmZXIg dG8gcG93ZXIgYSBNaW5pIFF1YXJ0eiBDbG9jayBNb3ZlbWVudA0KYnkgdGhlIDEyIHZvbHQgYmF0 dGVyeSB0aGF0IGlzIGJlaW5nIHRlc3RlZCAod2l0aCB2b2x0YWdlIHJlZHVjZWQgdG8gMS41KQ0K VGhlIGNsb2NrIGFuZCByZWxheSBhbmQgY2lyY3VpdCBib2FyZCBjb3VsZCBhbGwgYmUgY29udGFp bmVkIGluIG9uZSBlbmNsb3N1cmUuDQoNCi0tLS0tLS0tDQpKb2UgR29yZXMNCg0KDQoNCg0KUmVh ZCB0aGlzIHRvcGljIG9ubGluZSBoZXJlOg0KDQpodHRwOi8vZm9ydW1zLm1hdHJvbmljcy5jb20v dmlld3RvcGljLnBocD9wPTUwMDQ5MCM1MDA0OTANCg0KDQoNCg0KDQoNCl8tPT09PT09PT09PT09 PT09PT09PT09PT09PT09PT09PT09PT09PT09PT09PT09PT09PT09PT09PT09PT09DQpfLT0gICAg ICAgICAgLSBUaGUgQWVyb0VsZWN0cmljLUxpc3QgRW1haWwgRm9ydW0gLQ0KXy09IFVzZSB0aGUg TWF0cm9uaWNzIExpc3QgRmVhdHVyZXMgTmF2aWdhdG9yIHRvIGJyb3dzZQ0KXy09IHRoZSBtYW55 IExpc3QgdXRpbGl0aWVzIHN1Y2ggYXMgTGlzdCBVbi9TdWJzY3JpcHRpb24sDQpfLT0gQXJjaGl2 ZSBTZWFyY2ggJiBEb3dubG9hZCwgNy1EYXkgQnJvd3NlLCBDaGF0LCBGQVEsDQpfLT0gUGhvdG9z aGFyZSwgYW5kIG11Y2ggbXVjaCBtb3JlOg0KXy09DQpfLT0gICAtLT4gaHR0cDovL3d3dy5tYXRy b25pY3MuY29tL05hdmlnYXRvcj9BZXJvRWxlY3RyaWMtTGlzdA0KXy09DQpfLT09PT09PT09PT09 PT09PT09PT09PT09PT09PT09PT09PT09PT09PT09PT09PT09PT09PT09PT09PT09PQ0KXy09ICAg ICAgICAgICAgICAgLSBNQVRST05JQ1MgV0VCIEZPUlVNUyAtDQpfLT0gU2FtZSBncmVhdCBjb250 ZW50IGFsc28gYXZhaWxhYmxlIHZpYSB0aGUgV2ViIEZvcnVtcyENCl8tPQ0KXy09ICAgLS0+IGh0 dHA6Ly9mb3J1bXMubWF0cm9uaWNzLmNvbQ0KXy09DQpfLT09PT09PT09PT09PT09PT09PT09PT09 PT09PT09PT09PT09PT09PT09PT09PT09PT09PT09PT09PT09PQ0KXy09ICAgICAgICAgICAgICAt IE5FVyBNQVRST05JQ1MgTElTVCBXSUtJIC0NCl8tPSBBZGQgc29tZSBpbmZvIHRvIHRoZSBNYXRy b25pY3MgRW1haWwgTGlzdCBXaWtpIQ0KXy09ICAgLS0+IGh0dHA6Ly93aWtpLm1hdHJvbmljcy5j b20NCl8tPT09PT09PT09PT09PT09PT09PT09PT09PT09PT09PT09PT09PT09PT09PT09PT09PT09 PT09PT09PT09DQpfLT0gICAgICAgICAgICAgLSBMaXN0IENvbnRyaWJ1dGlvbiBXZWIgU2l0ZSAt DQpfLT0gIFRoYW5rIHlvdSBmb3IgeW91ciBnZW5lcm91cyBzdXBwb3J0IQ0KXy09ICAgICAgICAg ICAgICAgICAgICAgICAgICAgICAgLU1hdHQgRHJhbGxlLCBMaXN0IEFkbWluLg0KXy09ICAgLS0+ IGh0dHA6Ly93d3cubWF0cm9uaWNzLmNvbS9jb250cmlidXRpb24NCl8tPT09PT09PT09PT09PT09 PT09PT09PT09PT09PT09PT09PT09PT09PT09PT09PT09PT09PT09PT09PT09DQoNCg0KDQoNCg= ________________________________________________________________________________
Date: Feb 01, 2021
From: "Robert L. Nuckolls, III" <nuckolls.bob(at)aeroelectric.com>
Subject: Re: Poor Man's Battery Tester
At 08:55 AM 2/1/2021, you wrote: >Bob > >Is it right that the clock only runs when the battery is charging ? discharging Bob . . . Un impeachable logic: George Carlin asked, "If black boxes survive crashes, why don't they make the whole airplane out of that stuff?" ________________________________________________________________________________
Date: Feb 01, 2021
From: "Robert L. Nuckolls, III" <nuckolls.bob(at)aeroelectric.com>
Subject: Re: Poor Man's Battery Tester
At 08:49 AM 2/1/2021, you wrote: >I wonder if something like this would be useful? > >https://www.mpja.com/02-01-21.asp?r=326752&s=39 Absolutely. Good eye! Bob . . . Un impeachable logic: George Carlin asked, "If black boxes survive crashes, why don't they make the whole airplane out of that stuff?" ________________________________________________________________________________
Date: Feb 01, 2021
From: "Robert L. Nuckolls, III" <nuckolls.bob(at)aeroelectric.com>
Subject: Re: Poor Man's Battery Tester
At 08:29 AM 2/1/2021, you wrote: > >How about a (readily available) AC hour meter? Admittedly, it could >give up to 1/10 (6 min) less reading. >Rumen Sure . . . unless the meter is resetable to zero, you would have to calculate the discharge time but yes, as a timekeeping device it would be a good candidate. Bob . . . Un impeachable logic: George Carlin asked, "If black boxes survive crashes, why don't they make the whole airplane out of that stuff?" ________________________________________________________________________________
Date: Feb 01, 2021
From: "Robert L. Nuckolls, III" <nuckolls.bob(at)aeroelectric.com>
Subject: Re: Poor Man's Battery Tester
At 08:01 AM 2/1/2021, you wrote: > >Bob, clocks that run off from 120 VAC are rare. >I suggest that the AC clock be replaced by a 1.5 volt battery operated clock. >Some experimenters might prefer to power a Mini Quartz Clock Movement >by the 12 volt battery that is being tested (with voltage reduced to 1.5) >The clock and relay and circuit board could all be contained in one enclosure. How do you start/stop the thing without it going brain dead? Bob . . . Un impeachable logic: George Carlin asked, "If black boxes survive crashes, why don't they make the whole airplane out of that stuff?" ________________________________________________________________________________
Date: Feb 01, 2021
From: "Robert L. Nuckolls, III" <nuckolls.bob(at)aeroelectric.com>
Subject: Re: Poor Man's Battery Tester
At 05:25 AM 2/1/2021, you wrote: > >I just use a 12 volt automotive inverter to drive 120 volt lamps as >a load. The cigarette lighter inverters that I have automatically >shut down right around 10 volts input. Close enough for me. >Ken Good put. Keep in mind that the precision with which this device works is something like drawing with a piece of chalk. It is quite precise with respect to measuring end-of-life. No mater WHAT reading you get on your NEW battery, that time value represents 100% of as-new capacity. The point at which it falls to 80% is readily observed. Fur for our purposes, qualifying against battery only endurance goals is quite good too. Knowing the exact ah or watt-hours is irrelevant. Bob . . . Un impeachable logic: George Carlin asked, "If black boxes survive crashes, why don't they make the whole airplane out of that stuff?" ________________________________________________________________________________
Date: Feb 01, 2021
From: "Robert L. Nuckolls, III" <nuckolls.bob(at)aeroelectric.com>
Subject: Re: Poor Man's Battery Tester
At 12:27 AM 2/1/2021, you wrote: >and do you need the second 470 ohm resistor between the transistor >base & the LM431? Yes. Pulling down on that base is a 'hard' conduction path. Maximum rated current for the LM431 would be exceeded. Bob . . . Un impeachable logic: George Carlin asked, "If black boxes survive crashes, why don't they make the whole airplane out of that stuff?" ________________________________________________________________________________
Date: Feb 01, 2021
From: "Robert L. Nuckolls, III" <nuckolls.bob(at)aeroelectric.com>
Subject: Re: Poor Man's Battery Tester
At 12:18 AM 2/1/2021, you wrote: >Bob, > >Is that the correct symbol for the 1N4001? Is it a zener? Good catch. no it's a plain vanilla rectifier. Got it marked up on the original. Anyone else fine a 'toe stubber'? Bob . . . Un impeachable logic: George Carlin asked, "If black boxes survive crashes, why don't they make the whole airplane out of that stuff?" ________________________________________________________________________________
Date: Feb 01, 2021
From: "Robert L. Nuckolls, III" <nuckolls.bob(at)aeroelectric.com>
Subject: Re: Poor Man's Battery Tester
At 08:49 AM 2/1/2021, you wrote: >I wonder if something like this would be useful? > >https://www.mpja.com/02-01-21.asp?r=326752&s=39 Absolutely. Good eye! OOPS! Need to control the clock too. I'm leaving for Wichita in a few minutes. I'll do some 'asphalt engineering' on it to see if there's a way to integrate this product into the project. It could save a lot of fuss and bother even when burdened with shipping costs. Bob . . . Un impeachable logic: George Carlin asked, "If black boxes survive crashes, why don't they make the whole airplane out of that stuff?" ________________________________________________________________________________
Subject: Re: Poor Man's Battery Tester
From: "user9253" <fransew(at)gmail.com>
Date: Feb 01, 2021
> How do you start/stop the thing without it going brain dead? You must be thinking of a digital clock. Quartz clock movements come with hands and are powered by a AA battery. -------- Joe Gores Read this topic online here: http://forums.matronics.com/viewtopic.php?p=500504#500504 ________________________________________________________________________________
Subject: Re: Poor Man's Battery Tester
From: "user9253" <fransew(at)gmail.com>
Date: Feb 01, 2021
> OOPS! Need to control the clock too. I'm leaving for Wichita in a few > minutes. I'll do some 'asphalt engineering' on it to see if there's a way to > integrate this product into the project. Just power the quartz clock (it has hands) in parallel with the load. Of course drop the voltage to 1.5 -------- Joe Gores Read this topic online here: http://forums.matronics.com/viewtopic.php?p=500505#500505 ________________________________________________________________________________
Subject: Re: Poor Man's Battery Tester
From: "user9253" <fransew(at)gmail.com>
Date: Feb 01, 2021
> Joe, > A mere 50 years, we all had one of these clocks on our bedside night stand. Walmart still sells them. Roger You must be old like me. :-) When I wired my house back in the 1970s, I put a clock outlet on the kitchen wall. -------- Joe Gores Read this topic online here: http://forums.matronics.com/viewtopic.php?p=500506#500506 ________________________________________________________________________________
From: Robert Sultzbach <endspeed(at)yahoo.com>
Subject: Re: Poor Man's Battery Tester
Date: Feb 01, 2021
To make matters worse Joe, I would bet it was an analogue clock at that! The young kids who grew up with digital sometimes cannot read analogue clocks! They also cannot read cursive but thats a story for another day! Bob > On Feb 1, 2021, at 12:02, user9253 wrote: > > > >> Joe, >> A mere 50 years, we all had one of these clocks on our bedside night stand. Walmart still sells them. Roger > > You must be old like me. :-) When I wired my house back in the 1970s, I put a clock outlet on the kitchen wall. > > -------- > Joe Gores > > > > > Read this topic online here: > > http://forums.matronics.com/viewtopic.php?p=500506#500506 > > > > > > > > > ________________________________________________________________________________
Subject: Re: Alternator rectifier bad?
From: "pjc" <peter_campo(at)yahoo.com>
Date: Feb 01, 2021
nuckolls.bob(at)aeroelect wrote: > At 10:06 PM 1/13/2021, you wrote: > > > > > I have an automotive alternator (14184, external ford regulator) that has > > given good service in my RV6. Recently I have noticed fluctuating voltage > > displayed (digitally) on the USB adapter plugged into a 12V power socket > > (cigarette lighter style). > > Fluctuating voltage from an alternator is almost > never caused by a failure in the power-output > components (stator wiring and rectifier); it's > almost always caused by some anomaly in the > excitation/regulation side of the house. Regulator, > field supply power path, brushes, etc. > > > > Investigating further I see about 2V (RMS) on the bus with my cheapie > > nultimeter when the alternator is running and measure 14V (fluctuating > > +2/-0.5) with the meter on DC. > > The output from an alternator is trashy. 3-phase, rectified > power has a built-in peak to peak ripple on the order of > 13%. So a 14vdc machine can be expected to come with ~2 > volts peak-to-peak ripple. There are test tools intended to > diagnose an abnormal increase in this value as a result > of rectifier failure. An open or shorted diode in a > rectifier will produce a marked increase in ripple > voltage. > > Multimeters have varying ability to quantify this > voltage. There's pk-pk, average, rms, etc. Then > there are vagaries in the AC to DC conversion > techniques that the meter ultimately tries to > interpret and display. Some are better at it than > others but unless you have a BENCHMARK ac voltage > reading for a known good alternator, interpreting > the readings you've observed is problematic. > > > > I imagine this isnt doing any good for my Odyssey PC680. > > The battery couldn't care less. > > > > Im thinking I have a blown diode in the rectifier. > > Unlikely. > > > > Any chance the regulator could be to blame? Any further diagnostics > > called for before buying a replacement (rebuilt) alternator at the > > auto parts place ($35) ? Any place to find a replacement rectifier > > to install in the otherwise still functioning alternator? > > You bet! "Ford" regulators (an those with similar > architecture) are 'hyper sensitive' to increases > in resistance in the field-supply/voltage-sense > pathway. > > See the narrative in this document > > http://www.aeroelectric.com/articles/Alternators/Know_Your_Charging_System.pdf > > That starts on page 8 under, "Battery Master Switch and Battery > Master Relay" > > This phenomenon described has often been called > "the galloping/dancing ammeter syndrome". In your case, > it may be the "galloping voltmeter syndrome". > Given the age of your airplane and the 'ford' > regulator, there's a high probability that > your difficulty has roots in this same cause. > (do a Google on "dancing ammeter" and "galloping > ammeter" for a look back in time for this > condition. > > Start by fabricating a jumper wire to run > from your regulator A and B terminals directly > to the alternator B terminal thus bypassing ALL > at-risk wiring. Fire the engine up and check > voltage NOTE: THE ALTERNATOR COMES ON LINE > IMMEDIATELY AS FOR AS LONG AS THE ENGINE IS > RUNNING. > > If the voltage becomes stable, then consider > renewing ALL components between the bus and > the regulator A and B terminals. Replacing > one item might 'fix' the problem but renewing > all the components comes closer to effecting > a fix good for another 10 years. > > To rule out a regulator failure, you can > use a known good regulator to fabricate > the test-tool shown and wire it to the back > of your alternator for a test fun. > > This is a problem that has plagued a constellation > of older, single engine aircraft. We've had lengthy > discussions on the List and elsewhere on the 'net. > > > > As to causes, I am guilty of powering the field on and off a couple of times while the engine was running. This was in effort to diagnose a headphone noise (probably originating from the electro-mechanical regulator switching). I have subsequently been told powering the field on/off while running is a a bad idea. > > Contrary to popular myths, that has no deleterious > effect on the system or its components. > > > Bob . . . > > Un impeachable logic: George Carlin asked, "If black boxes > survive crashes, why don't they make the whole airplane > out of that stuff?" Follow-up: 1). No discrepancies (high resistances) found at Field switch or fuse (though AGC fuse and holder contacts cleaned). 2). No discrepancies at the alternator field and B connections or with the capacitor (condenser) mounted there. 3). Little improvement with further flight time (resulting in higher battery SOC) - this behavior started after a period of no use during annual inspection. 4). Significant improvement from unplugging and reseating the connector at the voltage regulator. Next time the cowl is off I will perform a more comprehensive contact cleaning (tough to do properly through the oil door). So now I am back to a simple dancing ammeter (with regular period between 1 and 2 Hz) that has been the norm for several years. The AC component as measured with my cheapie Digital voltmeter is under 0.5V and my panel lights are no-longer flickering. Im chalking this one up to a poor connection at the voltage regulator and returning the replacement alternator. Thanks very much for the lessons !! Peter Read this topic online here: http://forums.matronics.com/viewtopic.php?p=500512#500512 ________________________________________________________________________________
From: <mike(at)vision499.com>
Subject: EFIS Erratic Temp Readings
Date: Feb 01, 2021
Hello I have an EFIS that has erratic temperature readings when the engine is running. Manufacturer has suggested that EMI interference introduced by the ignition system may be the culprit and has suggested shielding the leads with tin plated copper braid that is grounded. I am unable to locate copper braid near me and will have to wait for a delivery from Aircraft Spruce. Will wrapping the leads with aluminum foil replicate the shielding effect for test purposes. The EGT leads (clamp on probe type) are already covered with a braided covering, which I presume is Stainless Steel for physical protection, will this act as an EMI shield and if yes how do I ground it? Copper Braid is sold in 1/8, 1/4, and 3/8 nominal flat width, any guesses as to what size will accommodate k-type thermocouple leads. Thanks ________________________________________________________________________________
Subject: Re: EFIS Erratic Temp Readings
From: Kelly McMullen <kellym(at)aviating.com>
Date: Feb 01, 2021
While I suppose it is possible that EMI could cause erratic readings, I tend to think the rate of erratic reads would be too fast for you to detect. A very high percentage of erratic readings are cause by inadequate connectors between the probe and the aircraft harness. Spade connectors or ring connectors are common. They can be made to work correctly for a period of time, but typically will degrade to inadequate conductivity. The only connector I know that has worked 100% of the time for me is the Electronics Internation OLC-2 connector(OlC-1 worked too, but was harder to use.). It can be obtained in multiples of 5 from EI for the bargain price of $1 ea. https://iflyei.com/product/olc-2/ On 2/1/2021 7:19 PM, mike(at)vision499.com wrote: > Hello > > I have an EFIS that has erratic temperature readings when the engine is > running. > > Manufacturer has suggested that EMI interference introduced by the > ignition system may be the culprit and has suggested shielding the leads > with tin plated copper braid that is grounded. I am unable to locate > copper braid near me and will have to wait for a delivery from Aircraft > Spruce. Will wrapping the leads with aluminum foil replicate the > shielding effect for test purposes. > > The EGT leads (clamp on probe type) are already covered with a braided > covering, which I presume is Stainless Steel for physical protection, > will this act as an EMI shield and if yes how do I ground it? > > Copper Braid is sold in 1/8, 1/4, and 3/8 nominal flat width, any > guesses as to what size will accommodate k-type thermocouple leads. > > Thanks > ________________________________________________________________________________
From: David Saylor <saylor.dave(at)gmail.com>
Date: Feb 01, 2021
Subject: Re: EFIS Erratic Temp Readings
I can't tell you how effective foil would be but maybe the group can help troubleshoot. Can you describe how it changes? Quickly, with large displacements, on/off, gradually, etc? Is it around the range you would expect? Is it a single cylinder measurement or similar behavior at different cylinders? --Dave On Mon, Feb 1, 2021 at 6:27 PM wrote: > Hello > > > I have an EFIS that has erratic temperature readings when the engine is > running. > > > Manufacturer has suggested that EMI interference introduced by the > ignition system may be the culprit and has suggested shielding the leads > with tin plated copper braid that is grounded. I am unable to locate copper > braid near me and will have to wait for a delivery from Aircraft Spruce. > Will wrapping the leads with aluminum foil replicate the shielding effect > for test purposes. > > > The EGT leads (clamp on probe type) are already covered with a braided > covering, which I presume is Stainless Steel for physical protection, will > this act as an EMI shield and if yes how do I ground it? > > > Copper Braid is sold in 1/8, 1/4, and 3/8 nominal flat width, any guesses > as to what size will accommodate k-type thermocouple leads. > > > Thanks > > ________________________________________________________________________________
From: Roger Evenson <revenson3(at)gmail.com>
Date: Feb 01, 2021
Subject: Poor Man's Battery Tester
The problem I had is solved to my satisfaction. For now, I'm sticking with the original schematic, except I've substituted a lower value zener diode. I'm now using a 8.2V, 1/4 watt, 5% zener in place of the 10V. I've tested it 3 times total on 2 different batteries. On my circuit I'm getting the 'kickout' at 10V. Scary how close it is (9.95V, 9.97V, and 10V). Thanks for all the help. This is a great forum. Roger, Tucson, AZ ________________________________________________________________________________
Date: Feb 01, 2021
From: "Robert L. Nuckolls, III" <nuckolls.bob(at)aeroelectric.com>
Subject: Re: Poor Man's Battery Tester
At 10:31 AM 2/1/2021, you wrote: > > > > How do you start/stop the thing without it going brain dead? > >You must be thinking of a digital clock. Quartz clock movements come with >hands and are powered by a AA battery. Aha! I had one of those once. Yeah, that's an option. Bob . . . Un impeachable logic: George Carlin asked, "If black boxes survive crashes, why don't they make the whole airplane out of that stuff?" ________________________________________________________________________________
Date: Feb 01, 2021
From: "Robert L. Nuckolls, III" <nuckolls.bob(at)aeroelectric.com>
Subject: trim those replies
Folks, when you reply to a post, please try to delete any portions of previous posts that do not support/advance the significance of your your comments/ideas. Matt archives all our postings but when excess baggage is not trimmed off a reply, the same words get archived over and over again . . . stuffing the library without adding value and making searches of relevant text more difficult. Thanks! Bob . . . Un impeachable logic: George Carlin asked, "If black boxes survive crashes, why don't they make the whole airplane out of that stuff?" ________________________________________________________________________________
From: <mikepienaar09(at)gmail.com>
Subject: EFIS Erratic Temp Readings
Date: Feb 01, 2021
If I only have sensor connected temperature reading is different for each =9Cscan=9D and varies by up to 150 degrees between max and min. If I connect more sensors readings become more and more erratic. Even if no sensors are connected =9Cghost=9D readings appear for cylinders not connected with the same reading for EGT and CHT. Ghost readings vary from between -50 to 800 degrees. Hope that helps From: owner-aeroelectric-list-server(at)matronics.com On Behalf Of David Saylor Sent: February 1, 2021 7:10 PM Subject: Re: AeroElectric-List: EFIS Erratic Temp Readings I can't tell you how effective foil would be but maybe the group can help troubleshoot. Can you describe how it changes? Quickly, with large displacements, on/off, gradually, etc? Is it around the range you would expect? Is it a single cylinder measurement or similar behavior at different cylinders? --Dave On Mon, Feb 1, 2021 at 6:27 PM > wrote: Hello I have an EFIS that has erratic temperature readings when the engine is running. Manufacturer has suggested that EMI interference introduced by the ignition system may be the culprit and has suggested shielding the leads with tin plated copper braid that is grounded. I am unable to locate copper braid near me and will have to wait for a delivery from Aircraft Spruce. Will wrapping the leads with aluminum foil replicate the shielding effect for test purposes. The EGT leads (clamp on probe type) are already covered with a braided covering, which I presume is Stainless Steel for physical protection, will this act as an EMI shield and if yes how do I ground it? Copper Braid is sold in 1/8, 1/4, and 3/8 nominal flat width, any guesses as to what size will accommodate k-type thermocouple leads. Thanks ________________________________________________________________________________
Subject: Re: EFIS Erratic Temp Readings
From: Kelly McMullen <kellym(at)aviating.com>
Date: Feb 02, 2021
Understand that both EGT and CHT are thermocouples that measure small changes in resistance. If you have nothing connected the resistance can float. If you have less than solid, airtight soldered, crimped or screw connection, the resistance will float. I'd still put my money on connections that are not airtight. I fought the same issue thinking I had bad probes until I switched from spade connectors to the OLC-2 connectors. Temps are now rock solid. Same for fuel flow sensor. In fact on fuel flow the connection improved enough I had to lower the K factor between .5-1%. You can get all the connectors needed for $30 for a 6 cylinder or $20 for a 4 cylinder. AFAIK EI changed all of their instruments to the OLC-2 connectors instead of spade connectors. I seriously doubt you will see much improvement with shielding the whole system. On 2/1/2021 10:51 PM, mikepienaar09(at)gmail.com wrote: > If I only have sensor connected temperature reading is different for > each scan and varies by up to 150 degrees between max and min. > > If I connect more sensors readings become more and more erratic. Even if > no sensors are connected ghost readings appear for cylinders not > connected with the same reading for EGT and CHT. Ghost readings vary > from between -50 to 800 degrees. > > Hope that helps > > *From:*owner-aeroelectric-list-server(at)matronics.com > *On Behalf Of *David Saylor > *Sent:* February 1, 2021 7:10 PM > *To:* aeroelectric-list > *Subject:* Re: AeroElectric-List: EFIS Erratic Temp Readings > > I can't tell you how effective foil would be but maybe thegroup can > help troubleshoot. Can you describe how it changes? Quickly, with > large displacements, on/off, gradually, etc? Is it around the range you > would expect? Is it a single cylinder measurement or similar behavior > at different cylinders? > > --Dave > > On Mon, Feb 1, 2021 at 6:27 PM > wrote: > > Hello > > I have an EFIS that has erratic temperature readings when the engine > is running. > > Manufacturer has suggested that EMI interference introduced by the > ignition system may be the culprit and has suggested shielding the > leads with tin plated copper braid that is grounded. I am unable to > locate copper braid near me and will have to wait for a delivery > from Aircraft Spruce. Will wrapping the leads with aluminum foil > replicate the shielding effect for test purposes. > > The EGT leads (clamp on probe type) are already covered with a > braided covering, which I presume is Stainless Steel for physical > protection, will this act as an EMI shield and if yes how do I > ground it? > > Copper Braid is sold in 1/8, 1/4, and 3/8 nominal flat width, any > guesses as to what size will accommodate k-type thermocouple leads. > > Thanks > ________________________________________________________________________________
Subject: Re: Poor Man's Battery Tester
From: "cjfortin" <cjfortin(at)juno.com>
Date: Feb 02, 2021
On the updated file shouldn't the one wall power lead go the first set of normally open contacts on the relay? Read this topic online here: http://forums.matronics.com/viewtopic.php?p=500529#500529 ________________________________________________________________________________
Date: Feb 02, 2021
From: "Robert L. Nuckolls, III" <nuckolls.bob(at)aeroelectric.com>
Subject: Galloping ammeter
>Follow-up: > > 1). No discrepancies (high resistances) > found at Field switch or fuse (though AGC fuse and holder contacts cleaned). > 2). No discrepancies at the alternator > field and B connections or with the capacitor (condenser) mounted there. > 3). Little improvement with further flight > time (resulting in higher battery SOC) - this > behavior started after a period of no use during annual inspection. The resistance values I cited are measured in tens of milliohms and cannot be observed with the legacy volt-ohmmeter. > 4). Significant improvement from > unplugging and reseating the connector at the > voltage regulator. Next time the cowl is off I > will perform a more comprehensive contact > cleaning (tough to do properly through the oil door). Aha! this is suggestive of exactly the phenomenon I was describing. The act of re-seating a connection effected a CHANGE in resistance at that joint on the order of a milliohm or two. > >So now I am back to a simple dancing ammeter >(with regular period between 1 and 2 Hz) that >has been the norm for several years. The =9CAC >component=9D as measured with my cheapie Digital >voltmeter is under 0.5V and my panel lights are no-longer flickering. > >I=99m chalking this one up to a poor connection >at the voltage regulator and returning the replacement alternator. Good show. Try the experiment I suggested where you wire a regulator right to the back of the alternator thus eliminating all other ship's wiring . . . see if the ammeter stabilizes. I don't think I explained the 'resistance' thing clearly enough. We're talking about the resistance of wiring COMBINED with the resistance of EVERY metal-metal joint in the field supply pathway. Total pathway resistance from bus through circuit breakers, connectors, switches and intermediate accessories like ov sensor all add up to much less than an ohm. When the aircraft is new, all those joints are fresh and unaffected by the effects of environment and age. The attached drawing is an excerpt from a Cessna drawing for alternator installation. I've counted all the metal-to-metal joints which ADD to the resistance of the wire (22AWG at 16 milliOhms per foot) . . . all of which are in series with the FIELD SUPPLY path of a 3-terminal voltage regulator. This same pathway doubles as the VOLTAGE SENSE lead for the regulator. As long as that total resistance is low (factory fresh), all is right with the universe. 30 year old airplanes are another matter. The "galloping ammeter" phenomenon manifests when total path resistance rises above a threshold for regulation stability. We might be talking about a few hundred milliohms . . . a characteristic you cannot detect/measure with your ohmmeter. In some cases, replacing the split rocker switch 'cured' the problem . . . but it is only one of many contributors to rise in path resistance. In a relatively short period of time compared to airplane age, the problem returns because rise in path resistance has multiple contributors. This is why I suggested the short-lead, test-regulator experiment. It gets bypasses all ship's wiring and accessories. It serves to confirm that root cause galloping ammeter is too high resistance in the field-supply/voltage-sense pathway. Bob . . . Un impeachable logic: George Carlin asked, "If black boxes survive crashes, why don't they make the whole airplane out of that stuff?" ________________________________________________________________________________
Date: Feb 02, 2021
From: "Robert L. Nuckolls, III" <nuckolls.bob(at)aeroelectric.com>
Subject: Re: Poor Man's Battery Tester
At 09:49 AM 2/2/2021, you wrote: > >On the updated file shouldn't the one wall power lead go the first >set of normally open contacts on the relay? > Good eye . . . yes. Thank you. I've posted the corrected drawing at: https://tinyurl.com/y5urfyrr I took a closer look at the COTS battery LV shut-off controller. It has a lot of promise. I note that they do not bring the normally closed contact of the relay out to a terminal. Depending on their board layout, we have a 50-50 chance that the after-test-charging mode can be achieved by tacking the charger(+) feeder to the bottom of the board. I've ordered some samples of this product. I'll publish a page 2 to the Poor Man's Battery Tester when I've had a chance to evaluate the critter more closely. Bob . . . Un impeachable logic: George Carlin asked, "If black boxes survive crashes, why don't they make the whole airplane out of that stuff?" ________________________________________________________________________________
Date: Feb 02, 2021
From: "Robert L. Nuckolls, III" <nuckolls.bob(at)aeroelectric.com>
Subject: Re: EFIS Erratic Temp Readings
At 08:07 AM 2/2/2021, you wrote: > >Understand that both EGT and CHT are thermocouples that measure >small changes in resistance. Actually very TINY changes in VOLTAGE generated by the Seebeck effect . . . small temperature dependent voltages generated at the junction of dissimilar metals. >I had bad probes until I switched from spade connectors to the OLC-2 >connectors. >Temps are now rock solid. Joint quality in thermocouple feeds are important given the very tiny voltages involved. >I seriously doubt you will see much improvement with shielding the >whole system. Agreed, It is exceedingly rare that adding shields fixes any problem in an instrumentation system. Bob . . . Un impeachable logic: George Carlin asked, "If black boxes survive crashes, why don't they make the whole airplane out of that stuff?" ________________________________________________________________________________
Date: Feb 02, 2021
From: "Robert L. Nuckolls, III" <nuckolls.bob(at)aeroelectric.com>
Subject: EFIS Erratic Temp Readings
At 11:51 PM 2/1/2021, you wrote: >If I only have sensor connected temperature reading is different for >each scan and varies by up to 150 degrees between max and min. Whoops . . . missed this out the gate. If the readings are THAT erratic you may have something else going on. Is this a new condition or did it crop up after a period of normal ops? Are the readings believable when the engine is not running? Does turning the alternator on/off make a difference? Are your thermocouples on the engine electrically isolated . . . i.e. not grounded to the engine? Does the EFIS installation manual speak to the need for 'isolated' thermocouples? The gross excursions of reading sound more like a ground loop issue as opposed to connector quality. Bob . . . Un impeachable logic: George Carlin asked, "If black boxes survive crashes, why don't they make the whole airplane out of that stuff?" ________________________________________________________________________________
Subject: Re: EFIS Erratic Temp Readings
From: Dick Tasker <dick(at)thetaskerfamily.com>
Date: Feb 02, 2021
Actually, the voltage is not generated at the junction. It is effectively generated along the whole length of a conductor due to the temperature difference between the two ends. In fact, the only reason we get any output to measure is that the two wires of a thermocouple are chosen to have different Seebeck coefficients. See this article for more info tinyurl.com/3njog7qv Nonetheless, the junction, as well all connections from the point where the temperature is to be measured to the device doing the measuring, is definitely important since a bad connection there will probably cause significant errors in the generated output. Dick Tasker Robert L. Nuckolls, III wrote: > At 08:07 AM 2/2/2021, you wrote: >> >> Understand that both EGT and CHT are thermocouples that measure small changes in resistance. > > Actually very TINY changes in VOLTAGE generated > by the Seebeck effect . . . small temperature > dependent voltages generated at the junction > of dissimilar metals. > >> I had bad probes until I switched from spade connectors to the OLC-2 connectors. >> Temps are now rock solid. > > Joint quality in thermocouple feeds > are important given the very tiny > voltages involved. > >> I seriously doubt you will see much improvement with shielding the whole system. > > Agreed, It is exceedingly rare that adding shields > fixes any problem in an instrumentation > system. > > > Bob . . . > > Un impeachable logic: George Carlin asked, "If black boxes > survive crashes, why don't they make the whole airplane > out of that stuff?" > ________________________________________________________________________________
From: <mikepienaar09(at)gmail.com>
Subject: EFIS Erratic Temp Readings
Date: Feb 02, 2021
From: owner-aeroelectric-list-server(at)matronics.com On Behalf Of Robert L. Nuckolls, III Sent: February 2, 2021 8:48 AM Subject: RE: AeroElectric-List: EFIS Erratic Temp Readings At 11:51 PM 2/1/2021, you wrote: If I only have sensor connected temperature reading is different for each scan and varies by up to 150 degrees between max and min. Whoops . . . missed this out the gate. If the readings are THAT erratic you may have something else going on. Is this a new condition or did it crop up after a period of normal ops? Aircraft has never flown, only noticed this while testing engine. Are the readings believable when the engine is not running? Does turning the alternator on/off make a difference? Readings seem stable and accurate when engine not running, will test this again. Will run to-morrow and then disconnect alternator Are your thermocouples on the engine electrically isolated . . . i.e. not grounded to the engine? Not sure what this means I have K Type bayonet EGT sensors that clamp onto Exhaust and 1 yellow and 1 red wire that connects directly to MGL RDAC unit CHT sensors are under plug with same wire setup Does the EFIS installation manual speak to the need for 'isolated' thermocouples? Cannot find a reference for "isolated " thermocouples The gross excursions of reading sound more like a ground loop issue as opposed to connector quality. I have taken a ground from the EFIS directly to the engine block. The engine mounting bolt is grounded to battery and to the starter motor I have also tried it with the RDAC ground attached next to the EGT sensor but this made no difference None of the leads are joined, they go directly from the sensor to the MGL RDAC unit Mike Bob . . . Un impeachable logic: George Carlin asked, "If black boxes survive crashes, why don't they make the whole airplane out of that stuff?" ________________________________________________________________________________
Subject: Re: EFIS Erratic Temp Readings
From: Charlie England <ceengland7(at)gmail.com>
Date: Feb 02, 2021
snipped > > Cannot find a reference for isolated thermocouples > snipped > > None of the leads are joined, they go directly from the sensor to the > MGL RDAC unit > > Mike > Ask MGL which style (grounded or ungrounded) they require for their RDAC. Ground issues might not be limited to the sensors; poor/inconsistent ground reference could be related to the RDAC, or the EFIS, or ???. Charlie -- This email has been checked for viruses by Avast antivirus software. https://www.avast.com/antivirus ________________________________________________________________________________
Subject: Re: Some Thoughts on Z101
From: "dj_theis" <djtheis58(at)gmail.com>
Date: Feb 02, 2021
> Re brownout booster: I'm pondering a plan-d . . . > or are we up to 'e'? There are dozens of suitable > step up devices on the market but I have no knowledge > of their 'spool up' times nor can I personally > vouch for their specifications without putting > the device under test on the bench and doing > some measurements. I purchased a couple of these a while back from Amazon. https://www.amazon.com/ANMBEST-LTC1871-Converter-Adjustable-Voltmeter/dp/B08CMKGRZQ/ref=sr_1_1?dchild=1&keywords=ANMBEST+2PCS+LTC1871&qid=1612318310&s=electronics&sr=1-1 Adjustable output and alleged 6amp continuous output. I ran a quick test with a bench supply (using a GRT Sport EX EFIS as the only load). The GRT only draws about 0.5 amps so it's not much of a load but likely all I might need. I captured two waveforms. The first with simply turning off my very dated "BK Precision, bench supply. Turning off the supply left the output voltage to decay with the internal caps bleeding down. Hardly a simulation of an engine starting. The second might be slightly closer to a cranking environment, where I rapidly turned the voltage control knob down. For what it's worth. I like the thought of using one that has a known design and components, as Eric is suggesting. I'll continue to test this unit but who knows how long it will last. Dan Theis -------- Scratch building Sonex #1362 Read this topic online here: http://forums.matronics.com/viewtopic.php?p=500540#500540 Attachments: http://forums.matronics.com//files/voltage_control_change_in_bench_supply_with_grt_sport_ex_192.pdf http://forums.matronics.com//files/bench_supply_power_down_with_grt_sport_ex_205.pdf ________________________________________________________________________________
Date: Feb 02, 2021
From: "Robert L. Nuckolls, III" <nuckolls.bob(at)aeroelectric.com>
Subject: Re: EFIS Erratic Temp Readings
At 02:12 PM 2/2/2021, you wrote: > > >Actually, the voltage is not generated at the junction. It is >effectively generated along the whole length of a conductor due to >the temperature difference between the two ends. In fact, the only >reason we get any output to measure is that the two wires of a >thermocouple are chosen to have different Seebeck coefficients. Actually no. There are TWO junctions. One is attached to the temperature of interest, the second resides at some REFERENCE temperature. WAaayyy back when, multiple thermocouples scattered about a test airplane at Cessna were paired with 'reference' thermocouples immersed in an distilled water ice bath (0 degrees C). The real temperature of interest is deduced by comparing the test thermocouple with the reference thermocouple. Views -A- and -B- in this excerpt from the 'Connection. http://www.aeroelectric.com/articles/Thermocouple%20Junctions.pdf The reference junction doesn't have to be an ice bath, it can be part of the indicating instrument wherein 'reference' is cabin environment temperature . . . there will be compensating thermistors in the instrument to improve accuracy over a range of cabin environments. Non-electronic thermocouple instruments of old actually measured that difference voltage with meter movements that read full scale with tens of millivolts applied. In these cases, there was considerable current flowing in the thermocouple leads compared to electronic displays. Hence, thermocouple wire lengths and wire gages were critical to accurate measurement. Electronic displays don't need to draw current from the thermocouples hence, wire length and gage don't matter . . . there are no voltage drops in the lead wires. JOINTS in those runs are critical in that they need to occur in pairs so that potentially parasitic junctions cancel each other out. See chapter 10 in the 'Connection. Bob . . . Un impeachable logic: George Carlin asked, "If black boxes survive crashes, why don't they make the whole airplane out of that stuff?" ________________________________________________________________________________
Subject: Re: EFIS Erratic Temp Readings
From: Dick Tasker <dick(at)thetaskerfamily.com>
Date: Feb 03, 2021
I know there are two junctions and am quite well aware how thermocouples work and how to use them - one of the many jobs I had as an Electrical Engineer was designing thermocouple amplifiers to measure temperature in industrial settings. This was before the nifty ICs that have internal reference junction compensation so we used a temperature sensitive resistor network to compensate for the reference junction temperature. My point was more of a technical clarification. The voltage that we measure is NOT generated at a junction (or two junctions). It is generated along the whole length of the wire based on the temperature gradient along the wire in question. The only reason we get anything to measure is that the two wires used in the thermocouple have different coefficients. Not that all this is terribly pertinent in actual applications, but one should know how electrical phenomena actually work. Dick Tasker Robert L. Nuckolls, III wrote: > At 02:12 PM 2/2/2021, you wrote: >> >> Actually, the voltage is not generated at the junction. It is effectively generated along the whole length of a conductor due to the temperature difference between the two ends. In fact, the only >> reason we get any output to measure is that the two wires of a thermocouple are chosen to have different Seebeck coefficients. > > Actually no. There are TWO junctions. One is > attached to the temperature of interest, the > second resides at some REFERENCE temperature. > WAaayyy back when, multiple thermocouples scattered > about a test airplane at Cessna were paired with > 'reference' thermocouples immersed in an distilled > water ice bath (0 degrees C). > > The real temperature of interest is deduced by > comparing the test thermocouple with the reference > thermocouple. Views -A- and -B- in this excerpt > from the 'Connection. > > http://www.aeroelectric.com/articles/Thermocouple%20Junctions.pdf > > The reference junction doesn't have to be an ice bath, > it can be part of the indicating instrument wherein > 'reference' is cabin environment temperature . . . there > will be compensating thermistors in the instrument to > improve accuracy over a range of cabin environments. > > Non-electronic thermocouple instruments of old actually > measured that difference voltage with meter movements that read > full scale with tens of millivolts applied. In these > cases, there was considerable current flowing in the > thermocouple leads compared to electronic displays. > Hence, thermocouple wire lengths and wire gages were > critical to accurate measurement. > > Electronic displays don't need to draw current from > the thermocouples hence, wire length and gage > don't matter . . . there are no voltage drops in the > lead wires. JOINTS in those runs are critical > in that they need to occur in pairs so that potentially > parasitic junctions cancel each other out. See chapter > 10 in the 'Connection. > > > Bob . . . > > Un impeachable logic: George Carlin asked, "If black boxes > survive crashes, why don't they make the whole airplane > out of that stuff?" > ________________________________________________________________________________
Date: Feb 03, 2021
From: "Robert L. Nuckolls, III" <nuckolls.bob(at)aeroelectric.com>
Subject: EFIS Erratic Temp Readings
> Is this a new condition or did it crop up after a period > of normal ops? > >Aircraft has never flown, only noticed this while testing engine. Good data point. > Are the readings believable when the engine is not > running? Does turning the alternator on/off make > a difference? > >Readings seem stable and accurate when engine not running, will test >this again. > >Will run to-morrow and then disconnect alternator Also, while exploring effects of alternator operation, you might dis-mount temperature sensors from engine and let them hang free not touching engine. > Are your thermocouples on the engine electrically > isolated . . . i.e. not grounded to the engine? > > Not sure what this means > > I have K Type bayonet EGT sensors that clamp onto > Exhaust and 1 yellow and 1 red wire that connects directly to MGL RDAC unit > > CHT sensors are under plug with same wire setup The reason for this question is founded in characteristics of legacy thermocouples designed to drive stand-alone instruments. Given that there were no wires at the instrument ends attached to ground or other ship's wiring, the engine thermocouples could be quite simply constructed with the junctions literally 'grounded' to the sensor shell. Such sensors might not play well in the modern EFIS sandbox. I rather suspect that everyone builds isolated (insulated) thermocouples such that no electrical continuity exists between the thermocouple conductors and the sensor enclosure. These would be functionally friendly in any engine management system. > The gross excursions of reading sound more like > a ground loop issue as opposed to connector quality. > > I have taken a ground from the EFIS directly to the > engine block. The engine mounting bolt is grounded to battery and > to the starter motor > > I have also tried it with the RDAC ground attached > next to the EGT sensor but this made no difference When you installed this experimental ground, were there any OTHER grounds in operation? I.e. DC power ground? To absolutely eliminate the possibility of this being a ground loop problem. ALL grounds would need to go ONLY to crankcase. However, if your thermocouples are of the insulated variety, ground loop(s) are almost assuredly not the problem. Have you conversed with the manufacturer about this? > > >None of the leads are joined, they go directly from the sensor to >the MGL RDAC unit Okay. Bob . . . Un impeachable logic: George Carlin asked, "If black boxes survive crashes, why don't they make the whole airplane out of that stuff?" ________________________________________________________________________________
Subject: Re: Some Thoughts on Z101
From: "Eric Page" <edpav8r(at)yahoo.com>
Date: Feb 03, 2021
nuckolls.bob(at)aeroelect wrote: > Can you elaborate on your design goals? - Redundant electrical supply for the engine (auto conversion with ECU and electronic ignition) - Failure of one alternator or regulator cannot cause AOG - Redundant electrical supply for minimum/emergency VFR equipment (EFIS, GPS, comm, landing light, Hobbs) - Brown-out-proof EFIS installation > Re brownout booster: I'm pondering a plan-d . . . or are we up to 'e'? There are dozens of suitable step up devices on the market but I have no knowledge of their 'spool up' times nor can I personally vouch for their specifications without putting the device under test on the bench and doing some measurements. > > I think I have a way to craft a brown-out booster of KNOWN performance thus avoiding the risks for incorporating the Chinese unknowns. Watch this space. > > A full schematic of the proposed boost system would make it easier to 'grok'. A schematic for the booster I've been fiddling with is attached. It uses TI's LM3481: https://tinyurl.com/yy9twn9t+ It will grunt 7A all day. It takes about 12ms for the output to come up into a 2A load when it's powered up from cold, and if it's already powered, output never falls more than a few mV below 12V when input is cut from 14V to 10V. It seems to work well but it's never going to be a drop-in, jellybean solution; it's all SMD, including fine-pitch and leadless components. Thanks, Eric Read this topic online here: http://forums.matronics.com/viewtopic.php?p=500549#500549 Attachments: http://forums.matronics.com//files/brown_out_booster_rev_b_520.pdf ________________________________________________________________________________
Date: Feb 03, 2021
From: "Robert L. Nuckolls, III" <nuckolls.bob(at)aeroelectric.com>
Subject: Re: EFIS Erratic Temp Readings
> It is generated along the whole length of the wire based on the > temperature gradient along the wire in question. The only reason > we get anything to measure is that the two wires used in the > thermocouple have different coefficients. > >Not that all this is terribly pertinent in actual applications, but >one should know how electrical phenomena actually work. That's the assertion I was having trouble with. There are thermocouple installations in petroleum refineries that are hundreds of feet long and traverse a huge range of temperatures . . . if 'gradient along the length' is in play, how can such installations offer meaningful readings from 'the far end'? I would draw your attention to View -B- of the illustration I posted. It's entirely valid to EXTEND a thermocouple any necessary distance with two copper wires . . . as long as you KNOW the temperature of the environment where the transition from t/c wire to copper takes place. That joint becomes the reference junction. I've not encountered a situation where length or surrounding environment of extension conductors have an effect on calibration of a 'zero current' thermocouple installation. Bob . . . Un impeachable logic: George Carlin asked, "If black boxes survive crashes, why don't they make the whole airplane out of that stuff?" ________________________________________________________________________________
Subject: Re: EFIS Erratic Temp Readings
From: Dick Tasker <dick(at)thetaskerfamily.com>
Date: Feb 03, 2021
Robert L. Nuckolls, III wrote: > That's the assertion I was having trouble with. > There are thermocouple installations in petroleum > refineries that are hundreds of feet long and > traverse a huge range of temperatures . . . if > 'gradient along the length' is in play, how > can such installations offer meaningful readings > from 'the far end'? Think about it. The thermocouple that is hundreds of feet long that goes through lots of temperature transitions from one end to the other generates voltage all along its length - sometimes positive and sometimes negative, depending on the temperature gradient along that particular section. The only thing that matters is the total temperature difference between the two ends. All other intermediate transitions cancel out. It's like putting lots of batteries in series - some pointing one way and others pointing the other. The net result is just the sum of all the batteries - positive and negative adding and subtracting respectively. > I would draw your attention to View -B- of the > illustration I posted. It's entirely valid to > EXTEND a thermocouple any necessary distance > with two copper wires . . . as long as you KNOW > the temperature of the environment where the > transition from t/c wire to copper takes place. > That joint becomes the reference junction. You are correct and this works because you have extended both wires with the same material. You don't have to use copper, although that is normal because it is relatively cheap and available. You could extend them with gold wire if you wanted and you would get the same result. The key is that both extension wires are the same material and generate the same voltage from end to end so any temperature difference between the extension point and the meter (electronic display) in question is irrelevant (the voltagesgeneratedin each copper wire cancel out). > I've not encountered a situation where length > or surrounding environment of extension conductors > have an effect on calibration of a 'zero current' > thermocouple installation. Correct. As I suggested earlier, see this and related articles for more info tinyurl.com/3njog7qv Dick ________________________________________________________________________________
Subject: Re: Poor Man's Battery Tester
From: "user9253" <fransew(at)gmail.com>
Date: Feb 03, 2021
XH-M609 Battery Low Voltage Disconnect Module is available on eBay for as little as $5. Let me know if there is an error in my circuit. -------- Joe Gores Read this topic online here: http://forums.matronics.com/viewtopic.php?p=500555#500555 Attachments: http://forums.matronics.com//files/battery_load_tester_110.png ________________________________________________________________________________
Date: Feb 03, 2021
From: "Robert L. Nuckolls, III" <nuckolls.bob(at)aeroelectric.com>
Subject: Re: EFIS Erratic Temp Readings
>Think about it. The thermocouple that is hundreds of feet long that >goes through lots of temperature transitions from one end to the >other generates voltage all along its length - sometimes positive >and sometimes negative, depending on the temperature gradient along >that particular section. The only thing that matters is the total >temperature difference between the two ends. All other intermediate >transitions cancel out. It's like putting lots of batteries in >series - some pointing one way and others pointing the other. The >net result is just the sum of all the batteries - positive and >negative adding and subtracting respectively. Suppose I brought 10' of chromel wire together at a sense junction with 100' of alumel wire. Assume further that the far ends of both wires terminate at a temperature compensated thermocouple meter. Suppose further than the 100' strand was coiled up and immersed in 100C water while the 10' strand remained at room temperature. Is it your understanding that serious errors in temperature reading would be generated by gross differences in temperature dependent voltage sources within those two wires? I.e. thermocouple class voltages (microvolts) are generated by warming two identical molecules of a conductor? What is this phenomenon called? Can you point me to literature that explains it? Bob . . . Un impeachable logic: George Carlin asked, "If black boxes survive crashes, why don't they make the whole airplane out of that stuff?" ________________________________________________________________________________
Subject: Re: EFIS Erratic Temp Readings
From: "user9253" <fransew(at)gmail.com>
Date: Feb 03, 2021
One time at work I measured millivolts at the end of a two wire fire alarm cable that was buried underground. Water had gotten into the cable and turned it into a battery. -------- Joe Gores Read this topic online here: http://forums.matronics.com/viewtopic.php?p=500557#500557 ________________________________________________________________________________
Subject: Re: EFIS Erratic Temp Readings
From: Dick Tasker <dick(at)thetaskerfamily.com>
Date: Feb 03, 2021
Robert L. Nuckolls, III wrote: >> Think about it. The thermocouple that is hundreds of feet long that goes through lots of temperature transitions from one end to the other generates voltage all along its length - sometimes >> positive and sometimes negative, depending on the temperature gradient along that particular section. The only thing that matters is the total temperature difference between the two ends. All >> other intermediate transitions cancel out. It's like putting lots of batteries in series - some pointing one way and others pointing the other. The net result is just the sum of all the batteries >> - positive and negative adding and subtracting respectively. > > Suppose I brought 10' of chromel wire together at a > sense junction with 100' of alumel wire. Assume further > that the far ends of both wires terminate at a > temperature compensated thermocouple meter. > > Suppose further than the 100' strand was coiled up and > immersed in 100C water while the 10' strand remained > at room temperature. > > Is it your understanding that serious errors in > temperature reading would be generated by gross > differences in temperature dependent voltage > sources within those two wires? I.e. thermocouple > class voltages (microvolts) are generated by > warming two identical molecules of a conductor? No, assuming the ends of each wire are at room temperature on the meter and the temperature to be measured at the other end. The only thing that matters to your measurement is the temperature difference between the two ends of the thermocouple. What happens in between cancels out. The Alumel will generate an internal voltage where it goes into the 100C water, nothing along the 100' immersed in water and generate the opposite voltage where it comes out. This does of course assume that the alumel and chromel wires are homogeneous throughout their length. Manufacturers go to great lengths to make sure this is so. If not, all bets are off. > > What is this phenomenon called? Can you point > me to literature that explains it? It is called the Seebeck effect. I previously included a link that explains the effect: tinyurl.com/3njog7qv Also refer to the links in that article for more information. For practical purposes you can certainly continue to say that the voltage is generated at each junction and that nothing happens in between. All I am doing is explaining what actually happens. Dick ________________________________________________________________________________
Date: Feb 05, 2021
From: "Robert L. Nuckolls, III" <nuckolls.bob(at)aeroelectric.com>
Subject: Re: EFIS Erratic Temp Readings
>> What is this phenomenon called? Can you point >> me to literature that explains it? > >It is called the Seebeck effect. I previously included a link that >explains the effect: tinyurl.com/3njog7qv Also refer to the links in >that article for more information. > >For practical purposes you can certainly continue to say that the >voltage is generated at each junction and that nothing happens in >between. All I am doing is explaining what actually happens. I had to dig out references I've not consulted for oh . . . say . . . 40 years? Even my revered CRC Handbook of Chemistry and Physics doesn't dig this deep into the phenomenon. But I did find some recitations on the phenomenon Dick brought to the conversation. Thermocouples are but one instance where the characteristics of materials push electrons around. These are studies in electro-motive physics. There are many instances where 'differences' in two materials encourage motion of electrons under certain conditions. MECHANICALLY induced electro-motion: Shuffle your rubber soled shoes on a carpet in dry weather and you build a potential on your body that can produce visible sparks when discharged to some metal object or perhaps another person. Leather soled shoes, yeah but not so much. Mechanically modulated magnetic fields in alternators and generators produce very significant electron motion that keeps our instrument panels lit up. CHEMICALLY induced electro-motion: Conductive plates pasted with two different compounds of lead submerged in mixture of sulfuric acid will offers a potential for flow of electrons . . . what's more it's reversible. But if the plates are identical, no joy. Pieces of zinc and copper pushed into a lemon produces a measurable voltage between the two metals; but use two copper pieces and the effect goes away. THERMALLY induced electro-motion: put any two dissimilar metals in intimate contact with each other and they will produce a voltage based on temperature. I had not picked up on the phenomenon Dick cited because it's a layer down from the practical physics that applies to our craft. About 1821 this observant guy Seebeck discovered the thermo-electric effects in a loop of two metals wherein their joinings (junctions) were at different temperatures. The energy flow generated in that loop of wire was directly related to the magnitude of temperature difference between the two junctions. If the alloys were carefully controlled and tiny voltages accurately measured that energy would represent real temperatures. Hence the foundation of the thermocouple driven instruments spanning over a century of aviation practice. ELECTRICALLY induced thermo-motion: A few years later in 1834, another sharp fellow Peltier observed that the thermo-electric generation was reversible. I.e. if you replaced the thermocouple measuring instrument with a current source, those same two junctions would assume temperatures above and below ambient. Effects difficult to observe due to the very tiny effects. But they were real and could be detected. It wasn't until years later in 1854 that another sharp fellow Thomson identified thermo-electric effects within conductors that explained how Seebeck and Peltier's discoveries worked. The longitudinal thermo-electric property within conductors was named after Thomson. The Seebeck, Peltier and Thomson effects combine to complete a story of how thermocouples function to monitor engines and thermoelectric coolers refrigerate beer. At the ripe age of 22, Thompson was awarded a professorship in natural philosophy (physics). Thompson was only 30 years old when he identified and quantified this longitudinal behavior of thermodynamics in a single conductor. Thompson was better known by his British title of station: Lord Kelvin. Dick cited the next level down in the physics of Seebeck and Peltier effects; accurate but not generally included in practical explanations for the two technologies. It's kinda like dissecting the physics of lubricity and viscosity when considering suitability to task for lubricating oils in engines. His only error was to identify it as the 'Seebeck effect' as opposed to the 'Thomson effect'. It was an interesting excursion to a time when I probably should have been paying more attention in class! Thanks for the trip Dick! Bob . . . ----------------------------- Bob Nuckolls AeroElectric Connection P.O. Box 130 Medicine Lodge, KS 67104-0130 Mobile: 316-209-7528 Web: http://aeroelectric.com ------------------------------ >Dick > > Bob . . . Un impeachable logic: George Carlin asked, "If black boxes survive crashes, why don't they make the whole airplane out of that stuff?" ________________________________________________________________________________
Date: Feb 06, 2021
From: "Robert L. Nuckolls, III" <nuckolls.bob(at)aeroelectric.com>
Subject: Re: Poor Man's Battery Tester
At 03:41 PM 2/3/2021, you wrote: > >XH-M609 Battery Low Voltage Disconnect Module is available on eBay >for as little as $5. >Let me know if there is an error in my circuit. > >-------- >Joe Gores Looks good to me Joe. You could also use two FORWARD biased diodes to get your ~1.5 Volt supply for the clock. Let us know how this shakes out . . . Bob . . . Un impeachable logic: George Carlin asked, "If black boxes survive crashes, why don't they make the whole airplane out of that stuff?" ________________________________________________________________________________
Date: Feb 06, 2021
From: "Robert L. Nuckolls, III" <nuckolls.bob(at)aeroelectric.com>
Subject: EFIS Erratic Temp Readings
At 11:51 PM 2/1/2021, you wrote: >If I only have sensor connected temperature >reading is different for each =9Cscan=9D and >varies by up to 150 degrees between max and min. > >If I connect more sensors readings become more >and more erratic. Even if no sensors are >connected =9Cghost=9D readings appear for >cylinders not connected with the same reading >for EGT and CHT. Ghost readings vary from between -50 to 800 degrees. > > >Hope that helps Mike, I was reviewing this thread and another thought occurs. While testing with 'no sensors' what was the condition of the open inputs? Did you put shorts across them or were they simply disconnected? Bob . . . Un impeachable logic: George Carlin asked, "If black boxes survive crashes, why don't they make the whole airplane out of that stuff?" ________________________________________________________________________________
From: <mikepienaar09(at)gmail.com>
Subject: EFIS Erratic Temp Readings
Date: Feb 06, 2021
From: owner-aeroelectric-list-server(at)matronics.com On Behalf Of Robert L. Nuckolls, III Sent: February 6, 2021 9:36 AM Subject: RE: AeroElectric-List: EFIS Erratic Temp Readings At 11:51 PM 2/1/2021, you wrote: Mike, I was reviewing this thread and another thought occurs. While testing with 'no sensors' what was the condition of the open inputs? Did you put shorts across them or were they simply disconnected? No, they were not shorted, simply disconnected. Will try with them shorted next week. I would never have thought of shorting them I did remove the alternator and ran the engine with no alternator and that made no difference, Readings were still erratic Thanks Mike Bob . . . Un impeachable logic: George Carlin asked, "If black boxes survive crashes, why don't they make the whole airplane out of that stuff?" ________________________________________________________________________________
Date: Feb 06, 2021
From: "Robert L. Nuckolls, III" <nuckolls.bob(at)aeroelectric.com>
Subject: EFIS Erratic Temp Readings
> No, they were not shorted, simply disconnected. > Will try with them shorted next week. > > I would never have thought of shorting them It used to be pretty much standard that the input to a thermocouple driven display device was pretty low impedance. Modern electronics allows us to resolve those tiny voltages with displays having very high input impedances. I have data acquisition equipment that requires unused channels to be 'tied down', i.e. shorted out. Otherwise they 'wander' in response to stimuli in the environment. Your EFIS may be similarly designed. > > > I did remove the alternator and ran the engine with > no alternator and that made no difference, > > Readings were still erratic Hmmmm . . . so probably no value in exploring noise on the DC power line. See what the thing does with all inputs 'locked down' . . . then if stable . . . try adding one t/c at a time. Bob . . . Un impeachable logic: George Carlin asked, "If black boxes survive crashes, why don't they make the whole airplane out of that stuff?" ________________________________________________________________________________
Subject: Re: EFIS Erratic Temp Readings
From: Charlie England <ceengland7(at)gmail.com>
Date: Feb 06, 2021
snipped > > > Did you put shorts across them or were they > simply disconnected? > > No, they were not shorted, simply disconnected. Will > try with them shorted next week. > > I would never have thought of shorting them > > I did remove the alternator and ran the engine with no > alternator and that made no difference, > > Readings were still erratic > > Thanks > > Mike > Hi Mike, Can you remind us whether the engine is 'standard' mags & carb (or Bendix style injection), or if it has some combination of electronic ignition/fuel injection? Thanks, Charlie -- This email has been checked for viruses by Avast antivirus software. https://www.avast.com/antivirus ________________________________________________________________________________
From: Daryl Thompson <flyer532(at)gmail.com>
Date: Feb 08, 2021
Subject: Molex vs bus bar
Need some thoughts from you guys. I am installing 8 Mitchell products cluster gauges in a 1978 PA32-300 (fuel qty, oil press, oil tem, etc). I am wondering if there are any disadvantages to using a molex 8 pin connector with one side having the 8 power wires connected to it and the other side with the single 20 AWG power wire to one pin location and then use jumping wires to connect in series the other remaining pins so as to power the entire set of 8 gauge clusters. Each cluster draws 1/10th amp and 20 AWG wire is attached to the existing 5A Inst Gauge C/B. The other alternative would be to use a bus bar but I am very limited in locating a good place to attach a bar. Could I also do the same for the grounding wires, i.e. 8 ground wires to one side of the molex connector and other side with a single grounding wire with jumped pins. The single ground wire (size?) would be grounded to the frame. The entire installation will be receiving a 337 field approval. Thoughts? ________________________________________________________________________________
Subject: Re: Molex vs bus bar
From: Charlie England <ceengland7(at)gmail.com>
Date: Feb 08, 2021
On 2/8/2021 8:48 AM, Daryl Thompson wrote: > Need some thoughts from you guys. I am installing 8 Mitchell products > cluster gauges in a 1978PA32-300 (fuel qty, oil press, oil tem, etc). > I am wondering if there are any disadvantages to using a molex 8 pin > connector with one side having the 8 power wires connected to it and > the other side with the single 20 AWG power wire to one pin location > and then use jumping wires to connect in series the other > remainingpins so as to power the entire set of 8 gauge clusters. Each > cluster draws 1/10th amp and 20 AWG wire is attached to the existing > 5A Inst Gauge C/B. The other alternative would be to use a bus bar > but I am very limited in locating a good place to attach a bar. Could > I also do the same for the grounding wires, i.e. 8 ground wires to one > side of the molex connector and other side with a single grounding > wire with jumped pins. The single ground wire (size?) would be > grounded to the frame. The entire installation will be receiving a 337 > field approval. Thoughts? The architecture sounds reasonable to me, but my personal choice of hardware would not be molex. I was an electronics tech in several past lives, and saw a *lot* of reliability issues with them, in a lot of different environments. In addition, jumping that many pins would be as big a pain as just jumping from instrument to instrument in the 'traditional' fashion of wiring instruments. My choice of hardware would be Dsub. The airframe side of the connector could be a molded solder cup style, allowing you to bridge an entire line of pins with a single bared wire for B+, and the same on the other line for ground. A 25 pin connector would likely take less real estate than a 16 pin molex. The instrument side of the connector could be the removable pin type of connector, making it easy to add additional power wires for each instrument. Similar idea to the 'panel ground bus' detailed in the AEC drawings, but with one row being the supply instead of ground. FWIW, Charlie -- This email has been checked for viruses by Avast antivirus software. https://www.avast.com/antivirus ________________________________________________________________________________
From: Daryl Thompson <flyer532(at)gmail.com>
Date: Feb 08, 2021
Subject: Re: Molex vs bus bar
Awesome idea! Thank you sir! Any recommendations on the best place to buy the d subs? On Mon, Feb 8, 2021 at 11:27 AM Charlie England wrote: > ceengland7(at)gmail.com> > > On 2/8/2021 8:48 AM, Daryl Thompson wrote: > > Need some thoughts from you guys. I am installing 8 Mitchell products > > cluster gauges in a 1978 PA32-300 (fuel qty, oil press, oil tem, etc). > > I am wondering if there are any disadvantages to using a molex 8 pin > > connector with one side having the 8 power wires connected to it and > > the other side with the single 20 AWG power wire to one pin location > > and then use jumping wires to connect in series the other > > remaining pins so as to power the entire set of 8 gauge clusters. Each > > cluster draws 1/10th amp and 20 AWG wire is attached to the existing > > 5A Inst Gauge C/B. The other alternative would be to use a bus bar > > but I am very limited in locating a good place to attach a bar. Could > > I also do the same for the grounding wires, i.e. 8 ground wires to one > > side of the molex connector and other side with a single grounding > > wire with jumped pins. The single ground wire (size?) would be > > grounded to the frame. The entire installation will be receiving a 337 > > field approval. Thoughts? > The architecture sounds reasonable to me, but my personal choice of > hardware would not be molex. I was an electronics tech in several past > lives, and saw a *lot* of reliability issues with them, in a lot of > different environments. In addition, jumping that many pins would be as > big a pain as just jumping from instrument to instrument in the > 'traditional' fashion of wiring instruments. > > My choice of hardware would be Dsub. The airframe side of the connector > could be a molded solder cup style, allowing you to bridge an entire > line of pins with a single bared wire for B+, and the same on the other > line for ground. A 25 pin connector would likely take less real estate > than a 16 pin molex. The instrument side of the connector could be the > removable pin type of connector, making it easy to add additional power > wires for each instrument. Similar idea to the 'panel ground bus' > detailed in the AEC drawings, but with one row being the supply instead > of ground. > > FWIW, > > Charlie > > -- > This email has been checked for viruses by Avast antivirus software. > https://www.avast.com/antivirus > > ________________________________________________________________________________
From: Charlie England <ceengland7(at)gmail.com>
Date: Feb 08, 2021
Subject: Re: Molex vs bus bar
They're sold by just about every vendor that handles electronic components, so it's hard to recommend just one. For the individual crimp pin type, B&C would be a great choice. https://bandc.com/product-category/electrical-supplies/d-sub-audio-connectors/ For solder style (and crimp, as well), just about any of the 'usual suspects'. Newark, Mouser, Digi-Key, Allied, etc. The highest quality individual pins are like the ones B&C sells; known as 'machined pins', that have gold plated contact surfaces. Some (not all) of the solder type connectors will have gold plated contact surfaces, as well. Just search their sites for 'Dsub' and 'DB25' (for a 25 pin version). Charlie On Mon, Feb 8, 2021 at 11:06 AM Daryl Thompson wrote: > Awesome idea! Thank you sir! Any recommendations on the best place to buy > the d subs? > > On Mon, Feb 8, 2021 at 11:27 AM Charlie England > wrote: > >> ceengland7(at)gmail.com> >> >> On 2/8/2021 8:48 AM, Daryl Thompson wrote: >> > Need some thoughts from you guys. I am installing 8 Mitchell products >> > cluster gauges in a 1978 PA32-300 (fuel qty, oil press, oil tem, etc). >> > I am wondering if there are any disadvantages to using a molex 8 pin >> > connector with one side having the 8 power wires connected to it and >> > the other side with the single 20 AWG power wire to one pin location >> > and then use jumping wires to connect in series the other >> > remaining pins so as to power the entire set of 8 gauge clusters. Each >> > cluster draws 1/10th amp and 20 AWG wire is attached to the existing >> > 5A Inst Gauge C/B. The other alternative would be to use a bus bar >> > but I am very limited in locating a good place to attach a bar. Could >> > I also do the same for the grounding wires, i.e. 8 ground wires to one >> > side of the molex connector and other side with a single grounding >> > wire with jumped pins. The single ground wire (size?) would be >> > grounded to the frame. The entire installation will be receiving a 337 >> > field approval. Thoughts? >> The architecture sounds reasonable to me, but my personal choice of >> hardware would not be molex. I was an electronics tech in several past >> lives, and saw a *lot* of reliability issues with them, in a lot of >> different environments. In addition, jumping that many pins would be as >> big a pain as just jumping from instrument to instrument in the >> 'traditional' fashion of wiring instruments. >> >> My choice of hardware would be Dsub. The airframe side of the connector >> could be a molded solder cup style, allowing you to bridge an entire >> line of pins with a single bared wire for B+, and the same on the other >> line for ground. A 25 pin connector would likely take less real estate >> than a 16 pin molex. The instrument side of the connector could be the >> removable pin type of connector, making it easy to add additional power >> wires for each instrument. Similar idea to the 'panel ground bus' >> detailed in the AEC drawings, but with one row being the supply instead >> of ground. >> >> FWIW, >> >> Charlie >> >> -- >> This email has been checked for viruses by Avast antivirus software. >> https://www.avast.com/antivirus >> >> ========== >> - >> Electric-List" rel="noreferrer" target="_blank"> >> http://www.matronics.com/Navigator?AeroElectric-List >> ========== >> FORUMS - >> eferrer" target="_blank">http://forums.matronics.com >> ========== >> WIKI - >> errer" target="_blank">http://wiki.matronics.com >> ========== >> b Site - >> -Matt Dralle, List Admin. >> rel="noreferrer" target="_blank">http://www.matronics.com/contribution >> ========== >> >> >> >> ________________________________________________________________________________
From: Daryl Thompson <flyer532(at)gmail.com>
Date: Feb 08, 2021
Subject: Re: Molex vs bus bar
Ok thanks Charlie On Mon, Feb 8, 2021 at 1:11 PM Charlie England wrote: > They're sold by just about every vendor that handles electronic > components, so it's hard to recommend just one. For the individual crimp > pin type, B&C would be a great choice. > > https://bandc.com/product-category/electrical-supplies/d-sub-audio-connectors/ > > For solder style (and crimp, as well), just about any of the 'usual > suspects'. Newark, Mouser, Digi-Key, Allied, etc. The highest quality > individual pins are like the ones B&C sells; known as 'machined pins', that > have gold plated contact surfaces. Some (not all) of the solder type > connectors will have gold plated contact surfaces, as well. Just search > their sites for 'Dsub' and 'DB25' (for a 25 pin version). > > Charlie > > > On Mon, Feb 8, 2021 at 11:06 AM Daryl Thompson wrote: > >> Awesome idea! Thank you sir! Any recommendations on the best place to >> buy the d subs? >> >> On Mon, Feb 8, 2021 at 11:27 AM Charlie England >> wrote: >> >>> ceengland7(at)gmail.com> >>> >>> On 2/8/2021 8:48 AM, Daryl Thompson wrote: >>> > Need some thoughts from you guys. I am installing 8 Mitchell products >>> > cluster gauges in a 1978 PA32-300 (fuel qty, oil press, oil tem, etc). >>> > I am wondering if there are any disadvantages to using a molex 8 pin >>> > connector with one side having the 8 power wires connected to it and >>> > the other side with the single 20 AWG power wire to one pin location >>> > and then use jumping wires to connect in series the other >>> > remaining pins so as to power the entire set of 8 gauge clusters. Each >>> > cluster draws 1/10th amp and 20 AWG wire is attached to the existing >>> > 5A Inst Gauge C/B. The other alternative would be to use a bus bar >>> > but I am very limited in locating a good place to attach a bar. Could >>> > I also do the same for the grounding wires, i.e. 8 ground wires to one >>> > side of the molex connector and other side with a single grounding >>> > wire with jumped pins. The single ground wire (size?) would be >>> > grounded to the frame. The entire installation will be receiving a 337 >>> > field approval. Thoughts? >>> The architecture sounds reasonable to me, but my personal choice of >>> hardware would not be molex. I was an electronics tech in several past >>> lives, and saw a *lot* of reliability issues with them, in a lot of >>> different environments. In addition, jumping that many pins would be as >>> big a pain as just jumping from instrument to instrument in the >>> 'traditional' fashion of wiring instruments. >>> >>> My choice of hardware would be Dsub. The airframe side of the connector >>> could be a molded solder cup style, allowing you to bridge an entire >>> line of pins with a single bared wire for B+, and the same on the other >>> line for ground. A 25 pin connector would likely take less real estate >>> than a 16 pin molex. The instrument side of the connector could be the >>> removable pin type of connector, making it easy to add additional power >>> wires for each instrument. Similar idea to the 'panel ground bus' >>> detailed in the AEC drawings, but with one row being the supply instead >>> of ground. >>> >>> FWIW, >>> >>> Charlie >>> >>> -- >>> This email has been checked for viruses by Avast antivirus software. >>> https://www.avast.com/antivirus >>> >>> ========== >>> - >>> Electric-List" rel="noreferrer" target="_blank"> >>> http://www.matronics.com/Navigator?AeroElectric-List >>> ========== >>> FORUMS - >>> eferrer" target="_blank">http://forums.matronics.com >>> ========== >>> WIKI - >>> errer" target="_blank">http://wiki.matronics.com >>> ========== >>> b Site - >>> -Matt Dralle, List Admin. >>> rel="noreferrer" target="_blank">http://www.matronics.com/contribution >>> ========== >>> >>> >>> >>> ________________________________________________________________________________
From: <mikepienaar09(at)gmail.com>
Subject: EFIS Erratic Temp Readings
Date: Feb 08, 2021
Hi Mike, Can you remind us whether the engine is 'standard' mags & carb (or Bendix style injection), or if it has some combination of electronic ignition/fuel injection? It is a LOM engine with magneto's only and mechanical fuel injection. No electronics on the engine at all Mike Thanks, Charlie Virus-free. www.avast.com ________________________________________________________________________________
Subject: Re: Molex vs bus bar
From: "user9253" <fransew(at)gmail.com>
Date: Feb 08, 2021
Steinair also sells D-Sub pins. -------- Joe Gores Read this topic online here: http://forums.matronics.com/viewtopic.php?p=500607#500607 ________________________________________________________________________________
Subject: Antenna Grounding
From: "Eric Page" <edpav8r(at)yahoo.com>
Date: Feb 08, 2021
I think I know the answer to this question, but I'd like confirmation: does an antenna ground plane have to be electrically bonded to the airframe? The aircraft is a Kitfox, which has a steel tube fuselage. In an effort to keep the outside of the plane as clean as possible, I'm trying to put as many antennas as I can inside the fabric envelope. For example, the comm antenna... https://www.aircraftspruce.com/catalog/avpages/antennasystems.php ...is mounted inside the vertical stab. Other Kitfox builders have reported good performance with this antenna in the tail, despite a couple of steel tubes in the vicinity. Since the transponder and ADS-B antennas... https://www.aircraftspruce.com/catalog/avpages/ted_transponder.php --and-- https://www.aircraftspruce.com/catalog/avpages/monopole11-13561.php ...are so short, they'll easily fit inside the wings. I can fabricate a ~3" diameter ground plane from aluminum sheet for each of them, but there's no way to electrically ground those plates to the airframe if they're suspended inside the wings. Is that a problem, or is a good bond from coax shield to ground plane all that's needed? Thanks. Read this topic online here: http://forums.matronics.com/viewtopic.php?p=500608#500608 ________________________________________________________________________________
Subject: Re: Antenna Grounding
From: "user9253" <fransew(at)gmail.com>
Date: Feb 08, 2021
No, the ground plane does not have to be grounded to the airframe. Com antennas should be vertically polarized. The ANTENNA VHF-5 will work in a wing, but not very well. It might work well vertically in the tail. How about this antenna for $45? https://www.aircraftspruce.com/catalog/avpages/stainlessantenna.php You could make your own antenna. http://www.aeroelectric.com/articles/Antennas/DIY_Comm_Ant/DIY_Comm.html But for $45, it will be easier to buy one. Mount it vertically inside of the fuselage. It can either point up or down. Make a ground plane out of 4 wires going out in 4 directions. Keep the first half of the antenna away from airframe tubes. Use a SWR meter to check how well the antenna works. What engine are you using in your Kitfox? -------- Joe Gores Read this topic online here: http://forums.matronics.com/viewtopic.php?p=500610#500610 ________________________________________________________________________________
Subject: Re: Antenna Grounding
From: "user9253" <fransew(at)gmail.com>
Date: Feb 08, 2021
Did you mean 3 inch radius for the transponder antenna? 3 inch diameter is too small. The radius should be at least as long as the antenna. -------- Joe Gores Read this topic online here: http://forums.matronics.com/viewtopic.php?p=500611#500611 ________________________________________________________________________________
Subject: Re: Antenna Grounding
From: "user9253" <fransew(at)gmail.com>
Date: Feb 09, 2021
Bob N describes how to make a com antenna in his book on page 13-10. http://www.aeroelectric.com/Books/Connection/13_Antenna.pdf -------- Joe Gores Read this topic online here: http://forums.matronics.com/viewtopic.php?p=500614#500614 ________________________________________________________________________________
Subject: Re: Antenna Grounding
From: "Eric Page" <edpav8r(at)yahoo.com>
Date: Feb 09, 2021
user9253 wrote: > No, the ground plane does not have to be grounded to the airframe. OK, that's what I thought. > Com antennas should be vertically polarized. The ANTENNA VHF-5 will work in a wing, but not very well. It might work well vertically in the tail. Indeed. I installed the VHF-5 vertically, inside the tail. > Did you mean 3 inch radius for the transponder antenna? 3 inch diameter is too small. Whoops! Yes, I meant radius. Thanks, Joe. Read this topic online here: http://forums.matronics.com/viewtopic.php?p=500617#500617 ________________________________________________________________________________
Date: Feb 10, 2021
From: "Robert L. Nuckolls, III" <nuckolls.bob(at)aeroelectric.com>
Subject: Re: Antenna Grounding
>...are so short, they'll easily fit inside the wings. I can >fabricate a ~3" diameter ground plane from aluminum sheet for each >of them, but there's no way to electrically ground those plates to >the airframe if they're suspended inside the wings. Is that a >problem, or is a good bond from coax shield to ground plane all >that's needed? Thanks. no ground to aircraft needed for the UHF antennas. However, their RADIUS is equal to the seated height of the antenna so closer to 6" diameter for ADSB and 5.75" for the xponder antenna. Orient them so that the antennas extend down from the ground plane. It's hard to argue with customer satisfaction for any particular antenna configuration . . . but I'd really like to see a sweep of the antenna mounted in the vertical fin. At the least, proximity of metal to the antenna might well call for physical 'shortening' of the antenna. Bob . . . Un impeachable logic: George Carlin asked, "If black boxes survive crashes, why don't they make the whole airplane out of that stuff?" ________________________________________________________________________________
Date: Feb 10, 2021
From: "Robert L. Nuckolls, III" <nuckolls.bob(at)aeroelectric.com>
Subject: Kitfox vertical fin antenna opportunities
I'm hearing stories that builders are experiencing 'satisfactory' performance with vhf comm antennas embedded in the structure of a Kitfox vertical fin. I'd like to better understand the physics of this location a bit better. I've attached a photo of a Kitfox vertical fin sans fabric. Would you folks with access to a 'naked' structure tell me what dimensions apply to the items labeled in the attached photograph? Thanks! P.S. Names and email addresses of individuals flying Kitfox, embedded off-the-shelf antennas for VHF comm would be useful too. Bob . . . Un impeachable logic: George Carlin asked, "If black boxes survive crashes, why don't they make the whole airplane out of that stuff?" ________________________________________________________________________________
Subject: Re: Antenna Grounding
From: "Eric Page" <edpav8r(at)yahoo.com>
Date: Feb 11, 2021
nuckolls.bob(at)aeroelect wrote: > ...RADIUS is equal to the seated height of the > antenna so closer to 6" diameter for ADSB and > 5.75" for the xponder antenna Yeah, the 3" diameter was a brain fart on my part. I meant radius. > ...I'd really like to see a sweep of the > antenna mounted in the vertical fin. At the least, > proximity of metal to the antenna might well > call for physical 'shortening' of the antenna. I have a poor-man's VNA, so I'll see what I can do. I ordered some RG-400 last night, so it'll be a week or two before I can mock up a test. If I can make the VNA show something that looks sensible, I'll post it here. According to the manufacturer's data, this antenna cannot be shortened. Read this topic online here: http://forums.matronics.com/viewtopic.php?p=500643#500643 ________________________________________________________________________________
Subject: Re: Kitfox vertical fin antenna opportunities
From: "Eric Page" <edpav8r(at)yahoo.com>
Date: Feb 11, 2021
nuckolls.bob(at)aeroelect wrote: > Would you folks with access to a 'naked' > structure tell me what dimensions apply to > the items labeled in the attached photograph? A couple of notes... This installation would only be possible in a Kitfox with the optional airfoiled tail, which adds wood ribs to the steel structure. Otherwise the vertical stab and rudder are flat sided and there is no room to pass an antenna vertically within the stab. When installed, the ribs are only there to shape the fabric and are not structural. I bought my project from its first builder, and the tail ribs were already installed. It was very messily done, but the ribs are firmly attached so I did not remove/replace them. Their location is specified in the build manual, but accuracy obviously depends on the builder. These measurements are from my example: A: 6-1/2" B: 6-3/8" C: 6-1/8" D: 6-3/8" E: 19" (sits on 3/8" dia steel tube) F: 22-1/2" G: 27" (widest rib; sits on 7/8" dia steel tube) H: 6-1/4" The ribs taper from 3-3/4" wide at the bottom of "A" to 1-7/8" wide at the top of "H". I'm attaching some photos of how I did my installation. I fabricated a wood block with an angled end that would direct the antenna with equal spacing between the forward and aft tubes in the stab. The wood block has a slot in the bottom end to receive the antenna, which is secured with a fillet of caulk. Each rib has a 1" slot, through which the antenna elements pass, again secured with a small dab of caulk to prevent chafing. The center feed point falls in the space labeled "B" in your photo, so the coax will be supported by Adel clamps on diagonal steel member "F". The feed point is secured against vibration by a vertical strip of wood bonded to the ribs above and below, and attached to the feed point with a dab of caulk. The bottom element of the antenna will be secured by pinching between two small wood blocks, with their inner surfaces lined with a thin rubber sheet for grip. This will be done after the horizontal stab is installed, as the antenna passes through the open center section of the stab. > Names and email addresses of individuals > flying Kitfox, embedded off-the-shelf antennas for VHF > comm would be useful too. I can't help with that, unfortunately. I roughly copied the installation done by another builder who used this antenna. He reported on the TeamKitfox forum that he hears aircraft in the pattern at an airport 70 miles away from his, has good comms with ATC and has had no complaints about his transmissions. As I posted in the other thread, I'll try to get a VNA sweep once I have the coax installed and the weather improves so I can roll the fuselage outside. It's snowing sideways for the next several days. Read this topic online here: http://forums.matronics.com/viewtopic.php?p=500644#500644 Attachments: http://forums.matronics.com//files/img_1267_125.jpg http://forums.matronics.com//files/img_1099_561.jpg http://forums.matronics.com//files/img_0966_272.jpg http://forums.matronics.com//files/img_0965_118.jpg http://forums.matronics.com//files/img_0963_207.jpg ________________________________________________________________________________
Subject: Re: Some Thoughts on Z101
From: "Eric Page" <edpav8r(at)yahoo.com>
Date: Feb 11, 2021
Bob -- was there more to be said on this topic? I answered your questions above, but the thread wandered then slipped below the noise floor... Read this topic online here: http://forums.matronics.com/viewtopic.php?p=500645#500645 ________________________________________________________________________________
Subject: Re: Kitfox vertical fin antenna opportunities
From: "user9253" <fransew(at)gmail.com>
Date: Feb 11, 2021
It took me awhile to realize that the vertical black object going through the middle of the ribs is actually the com antenna and not structural steel. Is that a black plastic box at the bottom? What is inside of it? And what is the purpose of the triangular part that you are holding? Thanks Theoretically the steel tubes will block radio waves fore and aft. But one can not argue with the success of others. The installation looks good and I think that you will be happy with the antenna. -------- Joe Gores Read this topic online here: http://forums.matronics.com/viewtopic.php?p=500649#500649 ________________________________________________________________________________
From: Janet Amtmann <jgamtmann2(at)gmail.com>
Date: Feb 11, 2021
Subject: I forgot how to list on the list
How do I list on the list. I do get the list in my emails. Jurgen Amtmann ________________________________________________________________________________
Date: Feb 12, 2021
From: "Robert L. Nuckolls, III" <nuckolls.bob(at)aeroelectric.com>
Subject: Re: I forgot how to list on the list
At 07:42 PM 2/11/2021, you wrote: >How do I list on the=C2 list.=C2 I do get the list in my emails. > >Jurgen Amtmann Just send your posting to "aeroelectric-list(at)matronics.com" with a new subject line. Everything sent to this address is echoed to all List members. Bob . . . Un impeachable logic: George Carlin asked, "If black boxes survive crashes, why don't they make the whole airplane out of that stuff?" ________________________________________________________________________________
Subject: Van's Aircraft Statement
From: "user9253" <fransew(at)gmail.com>
Date: Feb 12, 2021
Here is a quote from Van's Aircraft FLIGHT TRAINING SUPPLEMENT: https://www.vansaircraft.com/wp-content/uploads/2020/10/RV-12iS-FTS-4.pdf page 6-6 > The Rotax 912 iS is equipped with two voltage regulators, one for each generator on the engine. These regulators reject any unused electrical power as heat. Allowing the power usage to fall below 7 amps for extended periods of time may damage the regulators, especially on hot days. That can not be true can it? Suppose that 21 amps are available but not used by the aircraft electrical system. 21 amps x 14 volts = 300 watts of heat that must be dissipated by the voltage regulators. Don't modern permanent magnet voltage regulators work by shutting off the output during part of the AC cycle using SCRs that are in series with the output? -------- Joe Gores Read this topic online here: http://forums.matronics.com/viewtopic.php?p=500656#500656 ________________________________________________________________________________
Subject: Re: Van's Aircraft Statement
From: C&K <yellowduckduo(at)gmail.com>
Date: Feb 12, 2021
Pretty sure I've read all of the current rotax manuals and bulletins for that engine and there is no such information or caution anywhere. (and it doesn't make sense anyway for something this large as Joe has indicated). The temperature limit on the regulators is 80*C and rotax insists that it be installed in the engine compartment. While there is an optional additional heat sink available for regulator B it's still an order of magnitude too small to handle that kind of heat. Ken On 12/02/2021 11:43 AM, user9253 wrote: > > Here is a quote from Van's Aircraft FLIGHT TRAINING SUPPLEMENT: > https://www.vansaircraft.com/wp-content/uploads/2020/10/RV-12iS-FTS-4.pdf page 6-6 > >> The Rotax 912 iS is equipped with two voltage regulators, one for each generator on the engine. These regulators reject any unused electrical power as heat. Allowing the power usage to fall below 7 amps for extended periods of time may damage the regulators, especially on hot days. > That can not be true can it? Suppose that 21 amps are available but not > used by the aircraft electrical system. 21 amps x 14 volts = 300 watts > of heat that must be dissipated by the voltage regulators. > Don't modern permanent magnet voltage regulators work by shutting off the > output during part of the AC cycle using SCRs that are in series with the output? > > -------- > Joe Gores > > > Read this topic online here: > > http://forums.matronics.com/viewtopic.php?p=500656#500656 > > ________________________________________________________________________________
Subject: Re: Kitfox vertical fin antenna opportunities
From: "Eric Page" <edpav8r(at)yahoo.com>
Date: Feb 12, 2021
user9253 wrote: > It took me awhile to realize that the vertical black object going through the > middle of the ribs is actually the com antenna and not structural steel. Is that > a black plastic box at the bottom? What is inside of it? And what is the > purpose of the triangular part that you are holding? Thanks > Theoretically the steel tubes will block radio waves fore and aft. But one can > not argue with the success of others. The installation looks good and I think > that you will be happy with the antenna. Yes, the antenna is sort of a fabric tape with, I presume, copper foil elements bonded to it, then encased with a layer of fiberglass and resin. It's about 1" wide and perhaps 0.05" thick. The "box" and triangular object are the same item: a small block of wood (painted black). It's half of the clamp for the bottom end of the antenna. The antenna's feed point is a ~1" cube in the center of the tape, located just behind the diagonal steel tube. The BNC is just visible in the "Finished Installation" photo. Read this topic online here: http://forums.matronics.com/viewtopic.php?p=500658#500658 ________________________________________________________________________________
Subject: Re: Van's Aircraft Statement
From: Charlie England <ceengland7(at)gmail.com>
Date: Feb 12, 2021
On 2/12/2021 10:43 AM, user9253 wrote: > > Here is a quote from Van's Aircraft FLIGHT TRAINING SUPPLEMENT: > https://www.vansaircraft.com/wp-content/uploads/2020/10/RV-12iS-FTS-4.pdf page 6-6 > >> The Rotax 912 iS is equipped with two voltage regulators, one for each generator on the engine. These regulators reject any unused electrical power as heat. Allowing the power usage to fall below 7 amps for extended periods of time may damage the regulators, especially on hot days. > That can not be true can it? Suppose that 21 amps are available but not > used by the aircraft electrical system. 21 amps x 14 volts = 300 watts > of heat that must be dissipated by the voltage regulators. > Don't modern permanent magnet voltage regulators work by shutting off the > output during part of the AC cycle using SCRs that are in series with the output? > > -------- > Joe Gores Easy enough to test empirically, if you have the system. IIRC, early rectifier-regulators really were 'shunt' style regulators, that would align with the warning from Van's. Newer regulator designs do seem to use 'switcher' style voltage control, which should function like typical semiconductors and get hotter as more current flows to the load. No idea what's used in the iS, but you'd hope it would be the more modern type. Charlie -- This email has been checked for viruses by Avast antivirus software. https://www.avast.com/antivirus ________________________________________________________________________________
From: Janet Amtmann <jgamtmann2(at)gmail.com>
Date: Feb 12, 2021
Subject: Repair of Apollo GX65
Does anyone know of someone that can repair an old Apollo GX 65 GPS/COM radio? I bought it new many years ago and installed it in my RV6. On power-up it did not work. No xmit or receive. The GPS window lights up and cycles thru the initial tests, so the radio is getting power properly. I get no satisfaction from Garmin who bought the line and then deleted it from stock. I did a bench test and it is not transmitting or receiving. The trays will not accommodate a different radio, I would have to rebuild the whole instrument panel, but I'm leery of purchasing a used one (they are available) and might get one with the same or other problem. At the moment we are flying with a handheld, but that's a PITA. Any information would be welcome. Jurgen Amtmann ________________________________________________________________________________
Subject: Re: Revmaster R2300 Ignition
From: "BRS" <brs(at)19301.net>
Date: Feb 13, 2021
I just found this old thread. Got to researching the topic as I've a R-2300 that is currently being built. After looking this all over, here is what I'm thinking. Do you see any issues with doing it this way? Wire the top plugs to a single double throw switch and the bottom plugs to another double throw switch. Thus flipping one switch will will isolate either the top or bottom plugs. Then it's just a matter of glancing at the multi-prob EGT to see if a coil is failing or not. A key switch could still be used for starting and possibly other functons. -------- -brs (Sportsman) Read this topic online here: http://forums.matronics.com/viewtopic.php?p=500674#500674 ________________________________________________________________________________
Subject: A 3 phase AC (Alternating Current) question
From: "donjohnston" <don@velocity-xl.com>
Date: Feb 14, 2021
Looking for some solutions on a project. I have a lathe/mill combo. I'm going to be upgrading it by replacing the motors and controlling them with VFD's. Space is tight and it occurred to me that if I could use one VFD to control the motors (never at the same time) that would simplify the installation. It would also save money since I would only need one VFD. :D But now the question is how to switch the VFD between the two motors? My first thought was a 3P2T switch. But I have not been able to find one rated for 230VAC and 1HP. It was suggested to use a contactor. But then I would need a low voltage source for the contactor with a switch for that power source and now it's no longer simplifying things. Any ideas? Read this topic online here: http://forums.matronics.com/viewtopic.php?p=500688#500688 ________________________________________________________________________________
Subject: Re: A 3 phase AC (Alternating Current) question
From: Charlie England <ceengland7(at)gmail.com>
Date: Feb 14, 2021
On 2/14/2021 2:00 PM, donjohnston wrote: > > Looking for some solutions on a project. > > I have a lathe/mill combo. I'm going to be upgrading it by replacing the motors and controlling them with VFD's. Space is tight and it occurred to me that if I could use one VFD to control the motors (never at the same time) that would simplify the installation. It would also save money since I would only need one VFD. :D > > But now the question is how to switch the VFD between the two motors? > > My first thought was a 3P2T switch. But I have not been able to find one rated for 230VAC and 1HP. It was suggested to use a contactor. But then I would need a low voltage source for the contactor with a switch for that power source and now it's no longer simplifying things. > > Any ideas? Contactors with 120VAC coils are used in thousands of devices. 1 HP should be easy to find; here's one rated at 30A per contact with a 120V coil: https://www.galco.com/buy/Deltrol-Controls/21011-84?source=googleshopping The vendor's spec sheet says '30A @ 30V DC', so it should handle 30A AC with no problem at all. I started by searching for '3pdt 30a 110v relay' Charlie -- This email has been checked for viruses by Avast antivirus software. https://www.avast.com/antivirus ________________________________________________________________________________
Date: Feb 14, 2021
From: "Robert L. Nuckolls, III" <nuckolls.bob(at)aeroelectric.com>
Subject: Re: A 3 phase AC (Alternating Current) question
At 02:00 PM 2/14/2021, you wrote: > >Looking for some solutions on a project. > >I have a lathe/mill combo. I'm going to be upgrading it by >replacing the motors and controlling them with VFD's. Space is >tight and it occurred to me that if I could use one VFD to control >the motors (never at the same time) that would simplify the >installation. It would also save money since I would only need one VFD. :D > >But now the question is how to switch the VFD between the two motors? > >My first thought was a 3P2T switch. But I have not been able to >find one rated for 230VAC and 1HP. It was suggested to use a >contactor. But then I would need a low voltage source for the >contactor with a switch for that power source and now it's no longer >simplifying things. How often do you switch? Perhaps a simple drop-cord off each motor with a twist-lock plug could be used to mate the in-service motor with its power source. Bob . . . Un impeachable logic: George Carlin asked, "If black boxes survive crashes, why don't they make the whole airplane out of that stuff?" ________________________________________________________________________________
Subject: Re: A 3 phase AC (Alternating Current) question
From: "donjohnston" <don@velocity-xl.com>
Date: Feb 14, 2021
> How often do you switch? Perhaps a simple > drop-cord off each motor with a twist-lock > plug could be used to mate the in-service > motor with its power source. Unfortunately, often enough that I'd rather not be swapping cables. That's just as bad as having to move the belts to change speeds. Read this topic online here: http://forums.matronics.com/viewtopic.php?p=500691#500691 ________________________________________________________________________________
Date: Feb 14, 2021
From: "Robert L. Nuckolls, III" <nuckolls.bob(at)aeroelectric.com>
Subject: Re: A 3 phase AC (Alternating Current) question
At 04:03 PM 2/14/2021, you wrote: > > > > How often do you switch? Perhaps a simple > > drop-cord off each motor with a twist-lock > > plug could be used to mate the in-service > > motor with its power source. > > >Unfortunately, often enough that I'd rather not be swapping >cables. That's just as bad as having to move the belts to change speeds. Okay, then use a 2PDT switch or relay to switch two of the three leads between motors. Leave the 3rd wire always connected between VFD and both motors. No current can flow to the 'unpowered' motor unless at least TWO wires are connected. So leaving the third wire always connected will have no effect on system operations. Bob . . . Un impeachable logic: George Carlin asked, "If black boxes survive crashes, why don't they make the whole airplane out of that stuff?" ________________________________________________________________________________
Subject: Re: A 3 phase AC (Alternating Current) question
From: "user9253" <fransew(at)gmail.com>
Date: Feb 14, 2021
How about one of these switches: https://www.mouser.com/Electromechanical/Switches/Toggle-Switches/_/N-5g2jZ1yzvvqxZgjdhub?P=1z0z296Z1z0z20xZ1z0z2qcZ1z0yqrwZ1y8pmr4Z1y8ziurZ1y8ziusZ1z0z63x&Rl=5g2jZer5eZ1z0x8gwZ1z0x87nSGT&Ns=Current%20Rating|0 I assume that your VFD has an on-off switch controlling input power. Turn the input power off before switching the output power from one motor to the other. Then the switch will not even know that an inductive load is connected. If you operate a switch with no load on it, then it will handle a lot more current without damage. Even if a switch carries more current than its rating, it will not self destruct. But its life might be shortened, for example from 100,000 cycles to 5,000 cycles. A 3 phase, 240 volt motor will draw less than 4 amps according to this table: https://twcontrols.com/lessons/motors-horsepower-and-amps-of-230vac-and-460vac-3-phase-60hz-electric-motors -------- Joe Gores Read this topic online here: http://forums.matronics.com/viewtopic.php?p=500693#500693 ________________________________________________________________________________
Subject: Re: A 3 phase AC (Alternating Current) question
From: "donjohnston" <don@velocity-xl.com>
Date: Feb 14, 2021
Would I use one leg of the 230v supply to power the coil? (I haven't used a relay like this before) Read this topic online here: http://forums.matronics.com/viewtopic.php?p=500692#500692 ________________________________________________________________________________
Date: Feb 14, 2021
From: "Robert L. Nuckolls, III" <nuckolls.bob(at)aeroelectric.com>
Subject: Re: A 3 phase AC (Alternating Current) question
At 04:03 PM 2/14/2021, you wrote: > > > > How often do you switch? Perhaps a simple > > drop-cord off each motor with a twist-lock > > plug could be used to mate the in-service > > motor with its power source. > > >Unfortunately, often enough that I'd rather not be swapping >cables. That's just as bad as having to move the belts to change speeds. Plan C: Use two of these puppies https://tinyurl.com/y2dya9lc to effect your 2PDT switching. Only one contactor energized at a time. Then you can switch between motors with an itty-bitty toggle switch located remotely from the high-voltage-fat- wires. You'll also need a 24vac source to power the contactor coils https://tinyurl.com/yytcqyuy Bob . . . Un impeachable logic: George Carlin asked, "If black boxes survive crashes, why don't they make the whole airplane out of that stuff?" ________________________________________________________________________________
Subject: Re: A 3 phase AC (Alternating Current) question
From: Charlie England <ceengland7(at)gmail.com>
Date: Feb 14, 2021
On 2/14/2021 4:08 PM, donjohnston wrote: > > Would I use one leg of the 230v supply to power the coil? > > (I haven't used a relay like this before) Yes. You do need a neutral, but anything wired to modern 'code' will have 3 phases, neutral and ground anyway. The switch is between one leg & the relay coil; the other coil terminal goes to neutral. Switch on would feed one output; switch off would feed the other. You'd still need some method of shutting down the whole system, of course. Do check on whether you have 'delta' or 'Y' configuration. With 'Y', you can pick any leg for the coil; with 'delta', you need to pick one of the 120 V legs; the 3rd leg will measure ~180 V to neutral. Charlie -- This email has been checked for viruses by Avast antivirus software. https://www.avast.com/antivirus ________________________________________________________________________________
Date: Feb 15, 2021
From: Ernest Christley <echristley(at)att.net>
Subject: Re: A 3 phase AC (Alternating Current) question
connect the switch to relays?=C2- But, now you're talking six relays, so more complication. y-xl.com> wrote: m> Looking for some solutions on a project. I have a lathe/mill combo.=C2- I'm going to be upgrading it by replacing the motors and controlling them with VFD's.=C2- Space is tight and it occ urred to me that if I could use one VFD to control the motors (never at the same time) that would simplify the installation.=C2- It would also save money since I would only need one VFD. :D But now the question is how to switch the VFD between the two motors? My first thought was a 3P2T switch.=C2- But I have not been able to find one rated for 230VAC and 1HP.=C2- It was suggested to use a contactor.=C2 - But then I would need a low voltage source for the contactor with a swi tch for that power source and now it's no longer simplifying things. Any ideas? Read this topic online here: http://forums.matronics.com/viewtopic.php?p=500688#500688 - S - WIKI - - =C2- =C2- =C2- =C2- =C2- -Matt Dralle, List Admin. ________________________________________________________________________________
Date: Feb 15, 2021
From: Ernest Christley <echristley(at)att.net>
Subject: Re: A 3 phase AC (Alternating Current) question
Plan D:=C2- If you want to keep it really simple.=C2- I just switched my mill to use an old treadmill motor.=C2- It required some machining to make thing fit up, but two treadmill motors could use the same controller w ith a DPDT switch.=C2- Another DPDT before that one could also give you r everse (but, you'll want to pin the flywheel if you do that).=C2- I've go t a video up showing what I did. https://rumble.com/vdm28n-powering-a-rong-fu-mill-with-a-treadmill-motor.ht ml?mref=fui8j&mc=c5hbl wrote: At 04:03 PM 2/14/2021, you wrote: --> AeroElectric-List messageposted by: "donjohnston" <don@velocity-xl.com> > How often do you switch? Perhaps a simple > drop-cord off each motor with a twist-lock > plug could be used to mate the in-service > motor with its power source. Unfortunately, often enough that I'd rather not be swapping cables.=C2-Th at's just as bad as having to move the belts to changespeeds. =C2-Plan C: =C2-Use two of these puppieshttps://tinyurl.com/y2dya9lc =C2-to effect your 2PDT switching. Only one contactor =C2-energized at a time. =C2-Then you can switch between motors with an itty-bitty =C2-toggle switch located remotely from the high-voltage-fat- =C2-wires. =C2- You'll also need a 24vac source to power the =C2- contactor coils =C2-https://tinyurl.com/yytcqyuy =C2- Bob . . . =C2- Un impeachable logic: George Carlin asked, "If blackboxes =C2- survive crashes, why don't they make the whole airplane =C2- out of that stuff?" ________________________________________________________________________________
From: Rickifly <gambler7425(at)gmail.com>
Date: Feb 14, 2021
Subject: DC Wiring Diagrams for SPA Panther LS
Hello everyone, I created DC wiring diagrams for my Panther LS in Solidworks. EAA offers this 2D/3D cad program free for members. It is very easy to use and fairly intuitive. I based my electrical design on the Aeroelectric drawing files and B&C electric products. I don't mind sending these drawings out to use as a starting point in your own electrical designs. DM me for your preferred drawing format. .pdfs, .slddrw or .dwg formats. Notes: Based on the Aeroelectric Connection Handbook and drawing Z09A. The ignition and fuel systems are for a Corvair with Dual Ignition. The voltage Regulator is the new B&C Electric AVC-1. It features three functions in one and is Aircraft purpose built v the JohnDeere or Ford types of VR's. Scheme uses GRT Avionics as primary EFIS and MGL Avionics as a backup. Basic Day/Night VFR scheme. The lighting is FLYLEDS. The drawings are my draft design. I have not wired my airplane yet. They are not 100% the final scheme. Here are my concerns: 1. I chose to utilize relays to operate many of the devices that are on toggle switches (as long as the relay didn't draw more current than the actual devices). I can operate the devices by toggle switch without the intermediate relays, but wanted to see what everyone thought about how the S704-1 Relay is used in my schemes. 2. Inputs to the EIS device where the 4.8V excitation outputs are used to drive the sensors (oil, temp. pressure) and the way they are wired electrically for each circuit. Specifically how the Resistors are in series and parallel. 3. Use of flyback diodes on the relay coils and contactors, and the unidirectional diodes on the LED lighting circuits. Are they wired in the correct direction (anode and cathode)? 4. Headset mic and headphone jacks. Curious if I have these wired correctly. Thanks and many kind regards, Rick Gamble SPA Panther LS - under construction ________________________________________________________________________________
From: "Matthew S. Whiting" <m.whiting(at)frontier.com>
Subject: Re: DC Wiring Diagrams for SPA Panther LS
Date: Feb 14, 2021
Did you use the SolidWorks Electrical package or just the base mechanical pa ckage? I tried to install it a year or so ago and was able to install the b ase package just fine, but I could not get the electrical package to install . Sent from my iPad > On Feb 14, 2021, at 8:53 PM, Rickifly wrote: > > =EF=BB > Hello everyone, > > I created DC wiring diagrams for my Panther LS in Solidworks. > EAA offers this 2D/3D cad program free for members. It is very easy to use and fairly intuitive. > > I based my electrical design on the Aeroelectric drawing files and B&C ele ctric products. I don't mind sending these drawings out to use as a starting point in your own electrical designs. DM me for your preferred drawing form at. .pdfs, .slddrw or .dwg formats. > > Notes: > Based on the Aeroelectric Connection Handbook and drawing Z09A. > The ignition and fuel systems are for a Corvair with Dual Ignition. > The voltage Regulator is the new B&C Electric AVC-1. It features three fun ctions in one and is Aircraft purpose built v the JohnDeere or Ford types of VR's. > Scheme uses GRT Avionics as primary EFIS and MGL Avionics as a backup. > Basic Day/Night VFR scheme. The lighting is FLYLEDS. > The drawings are my draft design. I have not wired my airplane yet. They a re not 100% the final scheme. > > Here are my concerns: > 1. I chose to utilize relays to operate many of the devices that are on to ggle switches (as long as the relay didn't draw more current than the actual devices). I can operate the devices by toggle switch without the intermedia te relays, but wanted to see what everyone thought about how the S704-1 Rela y is used in my schemes. > 2. Inputs to the EIS device where the 4.8V excitation outputs are used to d rive the sensors (oil, temp. pressure) and the way they are wired electrical ly for each circuit. Specifically how the Resistors are in series and parall el. > 3. Use of flyback diodes on the relay coils and contactors, and the unidir ectional diodes on the LED lighting circuits. Are they wired in the correct direction (anode and cathode)? > 4. Headset mic and headphone jacks. Curious if I have these wired correctl y. > > Thanks and many kind regards, > Rick Gamble > SPA Panther LS - under construction > > > > > ________________________________________________________________________________
From: Rickifly <gambler7425(at)gmail.com>
Date: Feb 14, 2021
Subject: Re: DC Wiring Diagrams for SPA Panther LS
I am using the basic student edition of Solidworks. I may invest in the full version if I keep modeling in 3D. It is very easy to learn and now I want a new graphics card and more processors, for those realistic looking models. I completed my first parts in 3D for the airplane: the Panel and the GRT Sport EFIS. And was able to send the Avionics Panel with cutouts modeled part drawing to EMachineShop for a quote. They accept Solidworks modeled parts without converting them to a step file or machine file. The electrical package would be great to have as an add-in. The automated features are very cool. For this little airplane I prob could finish point to points in short order... The Solidworks 2021 version seems better than previous years BTW. Rick On Sun, Feb 14, 2021 at 8:48 PM Matthew S. Whiting wrote: > Did you use the SolidWorks Electrical package or just the base mechanical > package? I tried to install it a year or so ago and was able to install > the base package just fine, but I could not get the electrical package to > install. > > Sent from my iPad > > On Feb 14, 2021, at 8:53 PM, Rickifly wrote: > > =EF=BB > > Hello everyone, > > I created DC wiring diagrams for my Panther LS in Solidworks. > EAA offers this 2D/3D cad program free for members. It is very easy to us e > and fairly intuitive. > > I based my electrical design on the Aeroelectric drawing files and B&C > electric products. I don't mind sending these drawings out to use as a > starting point in your own electrical designs. DM me for your preferred > drawing format. .pdfs, .slddrw or .dwg formats. > > Notes: > Based on the Aeroelectric Connection Handbook and drawing Z09A. > The ignition and fuel systems are for a Corvair with Dual Ignition. > The voltage Regulator is the new B&C Electric AVC-1. It features three > functions in one and is Aircraft purpose built v the JohnDeere or Ford > types of VR's. > Scheme uses GRT Avionics as primary EFIS and MGL Avionics as a backup. > Basic Day/Night VFR scheme. The lighting is FLYLEDS. > The drawings are my draft design. I have not wired my airplane yet. They > are not 100% the final scheme. > > Here are my concerns: > 1. I chose to utilize relays to operate many of the devices that are on > toggle switches (as long as the relay didn't draw more current than the > actual devices). I can operate the devices by toggle switch without the > intermediate relays, but wanted to see what everyone thought about how th e > S704-1 Relay is used in my schemes. > 2. Inputs to the EIS device where the 4.8V excitation outputs are used to > drive the sensors (oil, temp. pressure) and the way they are wired > electrically for each circuit. Specifically how the Resistors are in seri es > and parallel. > 3. Use of flyback diodes on the relay coils and contactors, and the > unidirectional diodes on the LED lighting circuits. Are they wired in th e > correct direction (anode and cathode)? > 4. Headset mic and headphone jacks. Curious if I have these wired > correctly. > > Thanks and many kind regards, > Rick Gamble > SPA Panther LS - under construction > > > > > ________________________________________________________________________________
Date: Feb 15, 2021
From: Jeff Luckey <jluckey(at)pacbell.net>
Subject: Re: DC Wiring Diagrams for SPA Panther LS
Hey Rick, Nice drawings! Regarding Relays: Relays can be used for all sorts of things.=C2- Two of the most common us es are:1. To use a small current to control a big current2. To implement sp ecial switching logic (like your trim control) It looks like several of the things you are controlling with relays are pre tty low current (less than 10 amp) so putting a relay in the circuit is sup erfluous. Those relays just add complexity, parts count, number of terminat ions, weight, etc without adding any real value.=C2- You'd be better-off using switches directly for the fuel pumps & auto pilot and probably the av ionics relay (also consider that a single avionics switch is a possible sin gle point of failure).=C2-=C2- That's what I saw with a quick glance at your drawings.=C2- I did not stu dy them. -Jeff ail.com> wrote: I am using the basic student edition of Solidworks. I may invest in the fu ll version if I keep modeling in 3D. It is very easy to learn and now I wan t a new graphics card and more processors, for those realistic looking mode ls. I completed my first parts in 3D for the airplane: the Panel and the GR T Sport EFIS.=C2- And was able to send the Avionics Panel with cutouts mo deled part drawing to EMachineShop for a quote.=C2- They accept Solidwork s modeled parts without converting them to a step file or machine file. =C2 -The electrical package would be great to have as an add-in.=C2- The au tomated features are very cool.=C2- For this little airplane I prob could finish point to points in short order...The Solidworks 2021 version seems better than previous years BTW.Rick=C2- On Sun, Feb 14, 2021 at 8:48 PM Matthew S. Whiting wrote: Did you use the SolidWorks Electrical package or just the base mechanical p ackage?=C2- I tried to install it a year or so ago and was able to instal l the base package just fine, but I could not get the electrical package to install. Sent from my iPad On Feb 14, 2021, at 8:53 PM, Rickifly wrote: =EF=BB Hello everyone,=C2- I created DC wiringdiagrams for my Panther LS in Solidworks.=C2-EAA offer s this 2D/3D cad programfree for members. It is very easy to use and fairly intuitive. I based my electrical design on the Aeroelectricdrawing files and B&C elect ric products. I don't mind sending these drawings out to use as a starting point in yourown electrical designs. DM me for your preferred drawing forma t. .pdfs, .slddrwor .dwg formats. Notes: Based on the Aeroelectric Connection Handbook anddrawing Z09A.=C2- The ignition and fuel systems are for a Corvairwith Dual Ignition. The voltage Regulator is the new B&C ElectricAVC-1. It features three funct ions in one and is Aircraft purpose built v theJohnDeere or Ford types of V R's. Scheme uses GRT Avionics as primary EFIS and MGLAvionics as a backup. Basic Day/Night VFR scheme. The lighting isFLYLEDS. The drawings are my draft design. I have not wiredmy airplane yet. They are not 100% the final scheme. Here are=C2-my concerns:1. I chose to utilize relays to operate many of t hedevices that are on toggle switches (as long as the relay didn't=C2-dra w more current than the actual=C2-devices). I can operate the devices by toggleswitch without the intermediate relays, but wanted to see what everyo ne thought about how the S704-1 Relay is used in my schemes.=C2- 2. Inputs to the EIS device where the 4.8V excitation=C2-outputs are used to drive the sensors (oil, temp. pressure) and the way they are wired elec trically for each circuit. Specifically how the Resistors are in series and parallel.=C2-=C2-3. Use of flyback diodes on the relay coils and conta ctors, and the unidirectional diodes on the LED lighting circuits.=C2- Ar e they wired in the correct direction (anode and cathode)?4. Headset mic an d headphone jacks. Curious if I have these wired correctly.=C2- Thanks and many kind regards,Rick GambleSPA Panther LS - under construction ________________________________________________________________________________
Date: Feb 14, 2021
From: "Robert L. Nuckolls, III" <nuckolls.bob(at)aeroelectric.com>
Subject: Re: A 3 phase AC (Alternating Current) question
At 04:08 PM 2/14/2021, you wrote: > >Would I use one leg of the 230v supply to power the coil? > >(I haven't used a relay like this before) > > See attached . . . Bob . . . Un impeachable logic: George Carlin asked, "If black boxes survive crashes, why don't they make the whole airplane out of that stuff?" ________________________________________________________________________________
Date: Feb 15, 2021
From: "Robert L. Nuckolls, III" <nuckolls.bob(at)aeroelectric.com>
Subject: Re: DC Wiring Diagrams for SPA Panther LS
Here are my concerns: 1. I chose to utilize relays to operate many of the devices that are on toggle switches (as long as the relay didn't draw more current than the actual devices). I can operate the devices by toggle switch without the intermediate relays, but wanted to see what everyone thought about how the S704-1 Relay is used in my schemes. What advantage is secured by doing this? It adds weight and complexity. Why an 'avionics switch'? It too adds complexity and offers single point of failure for all avionics. A Boeing 787 doesn't have one . . . they were slightly useful in 1965 but totally worthless by 1980. 2. Inputs to the EIS device where the 4.8V excitation outputs are used to drive the sensors (oil, temp. pressure) and the way they are wired electrically for each circuit. Specifically how the Resistors are in series and parallel. Are these resistors required per manufacturer's installation instructions? There are better ways to 'package' these. Threaded fastener terminal strips are kind of a last resort. Bob . . . Un impeachable logic: George Carlin asked, "If black boxes survive crashes, why don't they make the whole airplane out of that stuff?" ________________________________________________________________________________
Subject: Re: DC Wiring Diagrams for SPA Panther LS
From: "user9253" <fransew(at)gmail.com>
Date: Feb 15, 2021
The symbol for a 1N4005 has a straight line. The symbol on your drawing is for a zener diode. The 1N4005 diode in series with the low voltage warning light is backwards and will prevent illumination. What is the purpose of those diodes in series with warning lights? If those warning lights are LEDs, then they need series resistors unless they are already inside of the warning lights. Like others have said, get rid of the avionics switch and relay. They serve no useful purpose. Connect avionics to the main power bus. Remove that fuse between the battery contactor and the main power bus. If it blows, everything goes dark. There is a reason that fuse is not on Bob's drawings. The same applies to the battery bus fuse. Either eliminate it or increase its size to 60 amps. The electrical system needs to be protected from a shorted alternator "B" lead. That fuse should be located at the battery contactor end of the B lead. Consider what will happen if the battery contactor fails open shortly after takeoff. You will not know it because everything on the main bus keeps working, powered by the alternator. The engine keeps running off from the battery until the battery goes dead. Then it quits, but you won't know why. To prevent that scenario, connect the secondary fuel pump and secondary ignition to the main power bus. Consider using Bob's Z-101 http://www.aeroelectric.com/PPS/Adobe_Architecture_Pdfs/Z101B.pdf Notice that in Z-101B, neither the main power bus nor the battery bus are protected by a fuse. The engine bus in Z-101B has two power sources: the battery and the alternator. It probably doesn't matter, but the part numbers for the fuel-level-sender resistors indicate 475ohms, not 470. If you use a terminal strip, do not depend on the screw to make the electrical connection. Put the 2 or 3 wires into one ring terminal and solder or crimp. Then put the ring terminal under the screw. A loose screw will not affect the connection. Use blue Loctite on screw threads. -------- Joe Gores Read this topic online here: http://forums.matronics.com/viewtopic.php?p=500706#500706 ________________________________________________________________________________
Date: Feb 15, 2021
From: Rod Smith <rodsmith52(at)yahoo.com>
Subject: Solidworks Electrical
I am also using the EAA student version of soldiworks. A year ago I couldn't download the electrical part of the program. A couple months ago I was able to. If having problems I suggest uninstalling the program and any solidworks files and do a complete new installation. The access code you get from EAA is good for all parts of the program. I had to contact EAA to find out that you only had to enter that code once, not for each sub program. ________________________________________________________________________________
From: Rickifly <gambler7425(at)gmail.com>
Date: Feb 15, 2021
Subject: Re: DC Wiring Diagrams for SPA Panther LS
To all, Thank you for your feedback! 1. I am pleased I asked about the relays in the design and am determined on removing many of them as suggested. (I was zealous to design with relays everywhere!) 2. Bob, are you saying the Avionics master switch is a feature that adds a point of failure and not normally used? 3. The resistors are per the manufacturer's requirements. How can they be packaged other than a screw terminal with ring tongue terminals on the resistor ends? The resistors must be included in most other designs I assume however there are not many installation examples upon researching the internet. Best Regards, Rick On Mon, Feb 15, 2021 at 12:21 AM Robert L. Nuckolls, III < nuckolls.bob(at)aeroelectric.com> wrote: > Here are my concerns: > 1. I chose to utilize relays to operate many of the devices that are on > toggle switches (as long as the relay didn't draw more current than the > actual devices). I can operate the devices by toggle switch without the > intermediate relays, but wanted to see what everyone thought about how the > S704-1 Relay is used in my schemes. > > > What advantage is secured by doing this? It > adds weight and complexity. Why an 'avionics > switch'? It too adds complexity and offers > single point of failure for all avionics. > A Boeing 787 doesn't have one . . . they > were slightly useful in 1965 but totally > worthless by 1980. > > 2. Inputs to the EIS device where the 4.8V excitation outputs are used to > drive the sensors (oil, temp. pressure) and the way they are wired > electrically for each circuit. Specifically how the Resistors are in series > and parallel. > > Are these resistors required per manufacturer's installation > instructions? There are better ways to 'package' these. > Threaded fastener terminal strips are kind of a > last resort. > > > Bob . . . > > Un impeachable logic: George Carlin asked, "If black boxes > survive crashes, why don't they make the whole airplane > out of that stuff?" > ________________________________________________________________________________
Subject: Re: Revmaster R2300 Ignition
From: "dj_theis" <djtheis58(at)gmail.com>
Date: Feb 15, 2021
> I just found this old thread. Got to researching the topic as I've a R-2300 that is currently being built. After looking this all over, here is what I'm thinking. Do you see any issues with doing it this way? > > Wire the top plugs to a single double throw switch and the bottom plugs to another double throw switch. Thus flipping one switch will will isolate either the top or bottom plugs. Then it's just a matter of glancing at the multi-prob EGT to see if a coil is failing or not. BRS, I don't see any issues with your proposal and in fact, is similar to how I've decided to run my system. I use two toggle switches as you suggest (S700-2-5), on the main panel area for normal startup and runup. These are shown as S2 and S3 on the attached drawing. I added two additional switches (S700-2-7) for further testing each of the 4 separate ignition circuits. These switches are separated on my panel and are not expected to be used in normal operation. The test switches are S10 and S11 on the drawing. My thought was to have the commonly used switches emulate more conventional startup, runup, and shutdown operation. My added S10 and S11 switches are superfluous and I might not have included them if I had thought of using the EGT feedback. My panel is pretty much completed at this point. I like Bob's suggestion of a "gang control mechanism" as well, (earlier in this thread) but my normal switching area was limited. One of the well known limitations of the Sonex Legacy design, "not much panel space." Dan Theis -------- Scratch building Sonex #1362 Read this topic online here: http://forums.matronics.com/viewtopic.php?p=500708#500708 Attachments: http://forums.matronics.com//files/z16m_revmaster_2300_r9b_pg_3_631.pdf ________________________________________________________________________________
Subject: Re: DC Wiring Diagrams for
SPA Panther LS
From: Dick Tasker <dick(at)thetaskerfamily.com>
Date: Feb 15, 2021
I had my panel fabricated by Bill M. of Up North Aviation. He did a great job for a very reasonable cost. If you have a design and can supply it in DXF format he can use that directly to cut the panel. My panel is for an RV9A. I started with an Affordable Panel when I first built the plane but they went out of business as far as I can find so when I upgraded my panel with more modern electronics, I had to find someone to cut a new panel from scratch. I found Bill and couldn't have been happier! Easy to work with and a panel cut perfectly. His web site has comments about costs and what he can and cannot do. His price for a complete design and cutout is pretty cheap but if you provide the DXF of your design, it is even less (quite a bit less if my panel was a good example).I strongly suggest you contact him for a quote. I suspect you will be pleasantly surprised. The only thing he does not do is countersinks - but if you have built a plane to the point you need a panel, I am sure you can do those... * Up North Aviation* CNC Instrument Panel Fabrication WEB www.upnorthaviation.com <http://www.upnorthaviation.com/> FaceBook www.facebook.com/upnorthaviation <http://www.facebook.com/upnorthaviation> E-Mail upnorthaviation(at)gmail.com Dick Tasker Rickifly wrote: > I am using the basic student edition of Solidworks. I may invest in the full version if I keep modeling in 3D. It is very easy to learn and now I want a new graphics card and more processors, for > those realistic looking models. I completed my first parts in 3D for the airplane: the Panel and the GRT Sport EFIS. And was able to send the Avionics Panel with cutouts modeled part drawing to > EMachineShop for a quote. They accept Solidworks modeled parts without converting them to a step file or machine file. > ________________________________________________________________________________
From: Rickifly <gambler7425(at)gmail.com>
Date: Feb 15, 2021
Subject: Re: DC Wiring Diagrams
for SPA Panther LS Rickifly [image: Attachments]10:55 AM (3 minutes ago) to aeroelectric-list Hi Mr Tasker and Group, Thank you for the feedback of a panel fabricator! I will give them a call. This is the Panel, and the quote from EMachineShop was $142.87 delivered (attached). Reminder: I am a novice and there are many better ways that I have discovered since beginning drafting like adding more relationships and constraints etc. The drawings are defined accordingly, I just know my drawings will be better as I go Also available by DM request is the Panel Part in 2D and 3D and GRT 10.1 Sport EFIS Part modeled in Solidworks for fun. Many Regards, Rick On Mon, Feb 15, 2021 at 10:26 AM Dick Tasker wrote: > I had my panel fabricated by Bill M. of Up North Aviation. He did a great > job for a very reasonable cost. If you have a design and can supply it in > DXF format he can use that directly to cut the panel. My panel is for an > RV9A. I started with an Affordable Panel when I first built the plane but > they went out of business as far as I can find so when I upgraded my panel > with more modern electronics, I had to find someone to cut a new panel from > scratch. > > I found Bill and couldn't have been happier! Easy to work with and a > panel cut perfectly. His web site has comments about costs and what he can > and cannot do. His price for a complete design and cutout is pretty cheap > but if you provide the DXF of your design, it is even less (quite a bit > less if my panel was a good example). I strongly suggest you contact him > for a quote. I suspect you will be pleasantly surprised. The only thing > he does not do is countersinks - but if you have built a plane to the point > you need a panel, I am sure you can do those... > > * Up North Aviation* > CNC Instrument Panel Fabrication > WEB www.upnorthaviation.com > FaceBook www.facebook.com/upnorthaviation > E-Mail upnorthaviation(at)gmail.com > > Dick Tasker > > > Rickifly wrote: > > I am using the basic student edition of Solidworks. I may invest in the > full version if I keep modeling in 3D. It is very easy to learn and now I > want a new graphics card and more processors, for those realistic looking > models. I completed my first parts in 3D for the airplane: the Panel and > the GRT Sport EFIS. And was able to send the Avionics Panel with cutouts > modeled part drawing to EMachineShop for a quote. They accept Solidworks > modeled parts without converting them to a step file or machine file. > > ________________________________________________________________________________
From: Rickifly <gambler7425(at)gmail.com>
Date: Feb 15, 2021
Subject: Re: DC Wiring Diagrams for SPA Panther LS
Dear Joe and Group, Again, excellent feedback I hoped for! I indicated my responses beginning with the >. Please advise any more thoughts you may have about this design. The symbol for a 1N4005 has a straight line. The symbol on your drawing is for a zener diode. > You are correct I drew a Zener with the angled tips. The 1N4005 diode in series with the low voltage warning light is backwards and will prevent illumination. > I thought it was backward, it may have been overthought by me! I looked at the diode connections as the anode side connected to the bus and the cathode side connected to the device side and this output is internal to the device and acting like an internal ground to complete the circuit. The action I will take is to delete these diodes as this will be a far more dependable design. What is the purpose of those diodes in series with warning lights? > I was thinking of isolating any 12VDC at effectively 5VDC outputs. If those warning lights are LEDs, then they need series resistors unless they are already inside of the warning lights. >The Warning lights are not LED. They are incandescent from B&C. Do I still need the series resistors? For the voltage regulator warning lamp diode I agree it's backward, but I intend on removing this diode. >The AVC-1 manual says the following on page A-2: Note when active this output pulls to ground; when not active, it is pulled up from a diode-isolated internal pullup to 8.5V. It will support up to 100ma continuous load and is intended to ground an annunciator light (incan or LED), or be used as an output to feed an aircraft EFIS or other similar display. >For the GRT, the Installation manual says on page 21: A warning output is provided on the D-sub connector to drive an external warning light. This output provides a path to ground when active, thus the indicator should be wired with one of its terminals to aircraft power while the other is wired to this output. The maximum current that can be controlled by this output is 0.2 amps. >For the EIS output warning light, the manual says the following on page 13: The output is an OPEN/GROUND type output. This means that when this output is off (the alarm is not active), this line is equivalent to an OPEN circuit. When this output is on, this output is switched to GROUND. Thus, the external warning light (or annunciator) is connected to this output and +12 Volts. The maximum current this output can control is 0.11 Amperes, or 110 mA Like others have said, get rid of the avionics switch and relay. They serve no useful purpose. > In total agreement. I want to have an Avionics switch but understand its not required and serves no purpose in the normal operation. I recall seeing many designs and am not opposed to removing the switch, but want to be sure. Connect avionics to the main power bus. > Much appreciated. Acknowledged and wilco. Remove that fuse between the battery contactor and the main power bus. If it blows, everything goes dark. There is a reason that fuse is not on Bob's drawings. The same applies to the battery bus fuse. Either eliminate it or increase its size to 60 amps. >I put those fuses in based on the recommended design available from B&C for the AVC-1 voltage regulator. They are slow-blow fuses with steep curves for inrush and thermal overloads. I prefer not to have these in the system, but now need to decide if I should seek feedback from B&C on their design? I attached the AVC-1 wiring diagram from B&C for your review. The electrical system needs to be protected from a shorted alternator "B" lead. That fuse should be located at the battery contactor end of the B lead. > Yes you are correct about the B lead. I have a 20A rated permanent magnet AC generator. Not a real "alternator" with greater ratings. It connects directly into the VR unlike the example in Z101B. How should I take care of this scenario? Consider what will happen if the battery contactor fails open shortly after takeoff. You will not know it because everything on the main bus keeps working, powered by the alternator. The engine keeps running off from the battery until the battery goes dead. Then it quits, but you won't know why. To prevent that scenario, connect the secondary fuel pump and secondary ignition to the main power bus. >Thank you! What a great catch. I follow your logic and intend on connecting the Secondaries to the main bus. Curious if you recommend mounting the contactor with the cap down as suggested by White-Rodgers Contactors? Consider using Bob's Z-101 http://www.aeroelectric.com/PPS/Adobe_Architecture_Pdfs/Z101B.pdf Notice that in Z-101B, neither the main power bus nor the battery bus are protected by a fuse. >The question I have that arises is if the fusible links in the circuit are any different than providing a slow-blow fuse? The fuses are very specific to this design with the AVC-1 VR. I have attached the MIDI fuse curves and information. I would greatly appreciate any more feedback you may have after review. The engine bus in Z-101B has two power sources: the battery and the alternator. >I am trying to work through your suggestion to use Z01B as a design basis for the Batt and Batt/Alt sources for the main bus. The way my circuit works is it has two sources I think. To confirm: My diagram shows 1st the master turns on the battery contactor and brings in the battery to the main bus, then the Batt/Alt switch point brings in the alternator field so to speak and then two sources are on the main bus. I probably just need to look a little harder so bare with me. Thanks! It probably doesn't matter, but the part numbers for the fuel-level-sender resistors indicate 475ohms, not 470. > You're correct I bought 475ohms and need to see if I can source the 470ohm resistors. I wish I could just apply a correction factor for the input... If you use a terminal strip, do not depend on the screw to make the electrical connection. Put the 2 or 3 wires into one ring terminal and solder or crimp. Then put the ring terminal under the screw. A loose screw will not affect the connection. Use blue Loctite on screw threads. > Very good advice. I am grateful. Is there a better way to install the resistors? On Mon, Feb 15, 2021 at 9:31 AM user9253 wrote: > > The symbol for a 1N4005 has a straight line. The symbol on your drawing is > for a zener diode. > The 1N4005 diode in series with the low voltage warning light is backwards > and will prevent illumination. > What is the purpose of those diodes in series with warning lights? > If those warning lights are LEDs, then they need series resistors unless > they are already inside of the warning lights. > Like others have said, get rid of the avionics switch and relay. They > serve no useful purpose. > Connect avionics to the main power bus. > Remove that fuse between the battery contactor and the main power bus. > If it blows, everything goes dark. There is a reason that fuse is not on > Bob's drawings. > The same applies to the battery bus fuse. Either eliminate it or increase > its size to 60 amps. > The electrical system needs to be protected from a shorted alternator "B" > lead. > That fuse should be located at the battery contactor end of the B lead. > Consider what will happen if the battery contactor fails open shortly > after takeoff. > You will not know it because everything on the main bus keeps working, > powered by the alternator. > The engine keeps running off from the battery until the battery goes > dead. Then it quits, but you won't know why. > To prevent that scenario, connect the secondary fuel pump and secondary > ignition to the main power bus. > Consider using Bob's Z-101 > http://www.aeroelectric.com/PPS/Adobe_Architecture_Pdfs/Z101B.pdf > Notice that in Z-101B, neither the main power bus nor the battery bus are > protected by a fuse. > The engine bus in Z-101B has two power sources: the battery and the > alternator. > It probably doesn't matter, but the part numbers for the fuel-level-sender > resistors indicate 475ohms, not 470. > If you use a terminal strip, do not depend on the screw to make the > electrical connection. > Put the 2 or 3 wires into one ring terminal and solder or crimp. Then put > the ring terminal under the screw. > A loose screw will not affect the connection. Use blue Loctite on screw > threads. > > -------- > Joe Gores > > > Read this topic online here: > > http://forums.matronics.com/viewtopic.php?p=500706#500706 > > ________________________________________________________________________________
From: Rickifly <gambler7425(at)gmail.com>
Date: Feb 15, 2021
Subject: Re: DC Wiring Diagrams for SPA Panther LS
Dear Joe and Group, Again, excellent feedback I hoped for! I indicated my responses beginning with the >. Please advise any more thoughts you may have about this design. The symbol for a 1N4005 has a straight line. The symbol on your drawing is for a zener diode. > You are correct I drew a Zener with the angled tips. The 1N4005 diode in series with the low voltage warning light is backwards and will prevent illumination. > I thought it was backward, it may have been overthought by me! I looked at the diode connections as the anode side connected to the bus and the cathode side connected to the device side and this output is internal to the device and acting like an internal ground to complete the circuit. The action I will take is to delete these diodes as this will be a far more dependable design. What is the purpose of those diodes in series with warning lights? > I was thinking of isolating any 12VDC at effectively 5VDC outputs. If those warning lights are LEDs, then they need series resistors unless they are already inside of the warning lights. >The Warning lights are not LED. They are incandescent from B&C. Do I still need the series resistors? For the voltage regulator warning lamp diode I agree it's backward, but I intend on removing this diode. >The AVC-1 manual says the following on page A-2: Note when active this output pulls to ground; when not active, it is pulled up from a diode-isolated internal pullup to 8.5V. It will support up to 100ma continuous load and is intended to ground an annunciator light (incan or LED), or be used as an output to feed an aircraft EFIS or other similar display. >For the GRT, the Installation manual says on page 21: A warning output is provided on the D-sub connector to drive an external warning light. This output provides a path to ground when active, thus the indicator should be wired with one of its terminals to aircraft power while the other is wired to this output. The maximum current that can be controlled by this output is 0.2 amps. >For the EIS output warning light, the manual says the following on page 13: The output is an OPEN/GROUND type output. This means that when this output is off (the alarm is not active), this line is equivalent to an OPEN circuit. When this output is on, this output is switched to GROUND. Thus, the external warning light (or annunciator) is connected to this output and +12 Volts. The maximum current this output can control is 0.11 Amperes, or 110 mA Like others have said, get rid of the avionics switch and relay. They serve no useful purpose. > In total agreement. I want to have an Avionics switch but understand its not required and serves no purpose in the normal operation. I recall seeing many designs and am not opposed to removing the switch, but want to be sure. Connect avionics to the main power bus. > Much appreciated. Acknowledged and wilco. Remove that fuse between the battery contactor and the main power bus. If it blows, everything goes dark. There is a reason that fuse is not on Bob's drawings. The same applies to the battery bus fuse. Either eliminate it or increase its size to 60 amps. >I put those fuses in based on the recommended design available from B&C for the AVC-1 voltage regulator. They are slow-blow fuses with steep curves for inrush and thermal overloads. I prefer not to have these in the system, but now need to decide if I should seek feedback from B&C on their design? I attached the AVC-1 wiring diagram from B&C for your review. The electrical system needs to be protected from a shorted alternator "B" lead. That fuse should be located at the battery contactor end of the B lead. > Yes you are correct about the B lead. I have a 20A rated permanent magnet AC generator. Not a real "alternator" with greater ratings. It connects directly into the VR unlike the example in Z101B. How should I take care of this scenario? Consider what will happen if the battery contactor fails open shortly after takeoff. You will not know it because everything on the main bus keeps working, powered by the alternator. The engine keeps running off from the battery until the battery goes dead. Then it quits, but you won't know why. To prevent that scenario, connect the secondary fuel pump and secondary ignition to the main power bus. >Thank you! What a great catch. I follow your logic and intend on connecting the Secondaries to the main bus. Curious if you recommend mounting the contactor with the cap down as suggested by White-Rodgers Contactors? Consider using Bob's Z-101 http://www.aeroelectric.com/PPS/Adobe_Architecture_Pdfs/Z101B.pdf Notice that in Z-101B, neither the main power bus nor the battery bus are protected by a fuse. >The question I have that arises is if the fusible links in the circuit are any different than providing a slow-blow fuse? The fuses are very specific to this design with the AVC-1 VR. The MIDI fuse curves and information are attached. I would greatly appreciate any more feedback you may have after review. The engine bus in Z-101B has two power sources: the battery and the alternator. >I am trying to work through your suggestion to use Z01B as a design basis for the Batt and Batt/Alt sources for the main bus. The way my circuit works is it has two sources I think. To confirm: My diagram shows 1st the master turns on the battery contactor and brings in the battery to the main bus, then the Batt/Alt switch point brings in the alternator field so to speak and then two sources are on the main bus. I probably just need to look a little harder so bare with me. Thanks! It probably doesn't matter, but the part numbers for the fuel-level-sender resistors indicate 475ohms, not 470. > You're correct I bought 475ohms and need to see if I can source the 470ohm resistors. I wish I could just apply a correction factor for the input... If you use a terminal strip, do not depend on the screw to make the electrical connection. Put the 2 or 3 wires into one ring terminal and solder or crimp. Then put the ring terminal under the screw. A loose screw will not affect the connection. Use blue Loctite on screw threads. > Very good advice. I am grateful. Is there a better way to install the resistors? Regards Rick On Mon, Feb 15, 2021 at 9:31 AM user9253 wrote: > > The symbol for a 1N4005 has a straight line. The symbol on your drawing is > for a zener diode. > The 1N4005 diode in series with the low voltage warning light is backwards > and will prevent illumination. > What is the purpose of those diodes in series with warning lights? > If those warning lights are LEDs, then they need series resistors unless > they are already inside of the warning lights. > Like others have said, get rid of the avionics switch and relay. They > serve no useful purpose. > Connect avionics to the main power bus. > Remove that fuse between the battery contactor and the main power bus. > If it blows, everything goes dark. There is a reason that fuse is not on > Bob's drawings. > The same applies to the battery bus fuse. Either eliminate it or increase > its size to 60 amps. > The electrical system needs to be protected from a shorted alternator "B" > lead. > That fuse should be located at the battery contactor end of the B lead. > Consider what will happen if the battery contactor fails open shortly > after takeoff. > You will not know it because everything on the main bus keeps working, > powered by the alternator. > The engine keeps running off from the battery until the battery goes > dead. Then it quits, but you won't know why. > To prevent that scenario, connect the secondary fuel pump and secondary > ignition to the main power bus. > Consider using Bob's Z-101 > http://www.aeroelectric.com/PPS/Adobe_Architecture_Pdfs/Z101B.pdf > Notice that in Z-101B, neither the main power bus nor the battery bus are > protected by a fuse. > The engine bus in Z-101B has two power sources: the battery and the > alternator. > It probably doesn't matter, but the part numbers for the fuel-level-sender > resistors indicate 475ohms, not 470. > If you use a terminal strip, do not depend on the screw to make the > electrical connection. > Put the 2 or 3 wires into one ring terminal and solder or crimp. Then put > the ring terminal under the screw. > A loose screw will not affect the connection. Use blue Loctite on screw > threads. > > -------- > Joe Gores > > > Read this topic online here: > > http://forums.matronics.com/viewtopic.php?p=500706#500706 > > ________________________________________________________________________________
Subject: Re: DC Wiring Diagrams for SPA Panther LS
From: C&K <yellowduckduo(at)gmail.com>
Date: Feb 15, 2021
One way of dealing with the EIS resistors is to route the wires through a small box with the resistors in the box. That provides strain relief for the relatively fragile resistors. Another is to tie the resistors to a strip of perforated circuit board and enclose it with shrinkwrap or self fusing tape. Please just give a reference to a website rather than the large attachments. Ken ________________________________________________________________________________
From: Charlie England <ceengland7(at)gmail.com>
Date: Feb 15, 2021
Subject: Re: DC Wiring Diagrams for SPA Panther LS
On Mon, Feb 15, 2021 at 2:13 PM C&K wrote: > > One way of dealing with the EIS resistors is to route the wires through > a small box with the resistors in the box. That provides strain relief > for the relatively fragile resistors. Another is to tie the resistors > to a strip of perforated circuit board and enclose it with shrinkwrap or > self fusing tape. > > With proper strain relief, a slightly higher wattage resistor can be used (for physical robustness) and one lead can be crimped directly in the connector of choice. Other end soldered, or crimped with butt splice to wire. Shrink insulation over everything. On detecting alternator power loss: That should be handled with a low voltage monitor/warning system (standalone or within the EFIS/engine monitor). With set point ~1V below alternator output voltage, you'll know if you lose the alternator. Charlie Charlie ________________________________________________________________________________
Subject: Re: DC Wiring Diagrams for SPA Panther LS
From: "user9253" <fransew(at)gmail.com>
Date: Feb 15, 2021
No, incandescent lamps do not need series resistors. You are correct that B&C shows a fuse between the battery contactor and the main power bus. But thousands of airplanes are flying without that fuse. Bob N does not recommend it. Do a good job installing the main power bus feeder to be sure that it will not short out, especially where it goes through the firewall. Then that fuse is not needed. B&C also shows a fuse at the battery contactor end of the regulator "B" lead. That one IS needed. On your drawing, there is fuse on the right side of the battery contactor. Rotate that fuse 90 degrees CCW. Then interchange the wire going to the regulator with the one going to the main power bus. See attached picture. I do not know about the contactor mounting. Follow the manufacturer's recommendation. Fusible links are more robust than fuses. But either will work if installed correctly using the proper size. Those 5 extra ohms will not make any difference at all in the fuel level display. Don't worry about it. 12AWG wire going to the capacitor is overkill. 18 AWG wire is easier to work with. I noticed "OUTPUT AT 12.8V" on your schematic. The voltage regulator output should be set at 13.8 minimum. 14.2 is volts better. The PTT wire going to the mic jack tip is not needed unless you will use a hand held mic that has a PTT switch. -------- Joe Gores Read this topic online here: http://forums.matronics.com/viewtopic.php?p=500714#500714 Attachments: http://forums.matronics.com//files/avc_1_fuse_163.jpg ________________________________________________________________________________
Subject: Re: DC Wiring Diagrams for SPA Panther LS
From: "user9253" <fransew(at)gmail.com>
Date: Feb 15, 2021
On your schematic, put a dot where wires connect to each other. On page E9, it is very confusing about which wires are connected and which are not. The tachometer might not be wired right, but it is hard to tell. -------- Joe Gores Read this topic online here: http://forums.matronics.com/viewtopic.php?p=500715#500715 ________________________________________________________________________________
Subject: Re: Rainy day project
From: "Tr8nicz" <jjonddew(at)yahoo.com>
Date: Feb 15, 2021
Any new updates on this? I just bought a new jack from 4WheelOnline. I needed a new jack for my truck too. Read this topic online here: http://forums.matronics.com/viewtopic.php?p=500719#500719 ________________________________________________________________________________
Subject: Re: DC Wiring Diagrams for SPA Panther LS
From: "jsajpf" <john.friday_adis(at)sbcglobal.net>
Date: Feb 16, 2021
Rick, Does the aircraft pitch trim system include a manual trim wheel or lever, or is the sole means of pitch trim the electric system? The electric trim circuit lacks a combination "arm" and "command" input (think of the split trim switch on many GA type certificated aircraft). Only a "command" input is shown. I think this single input is common in E/AB airplanes but I personally do not favor it. I think your relay circuit for pitch trim (rather than direct switching of motor current with a momentary switch) provides a measure of protection against uncommanded or runaway pitch trim, in that an opposite direction command at the switch will change the state of the motor legs, at least stopping further trim movement while the switch is held. I think a cutout switch to remove power from the motor circuit is advisable, located such that a free hand can get get to it, perhaps while on its way to the manual trim control (mine will be next to the manual trim wheel). Removal of power from the existing circuit would require the pilot to locate and open the trim circuit breaker while also fighting the trim forces, and countering the uncommanded trim in some way (e.g. restrain the manual trim control or holding opposite direction trim command with the switch). Does the trim servo connect to the trim system in a way that allows the servo input to be manually overriden; meaning a slip feature that would allow holding the manual trim input against servo motion? Perhaps the servo itself has this feature? These system safety measures may all be unnecessary on the class of airplane you're building. I advise you to consider what sort of trim induced control forces you might expect at the speeds and configuration combinations of the airplane, and then look to see how quickly the electric system might get you into trouble; rate of trim, range of trim, etc. The airplane I will build is noted for trim sensitivity at cruise speeds so I'll have at least two trim speeds (slow for flaps up, faster for flaps down). There are systems that allow continuously variable speeds as well, typically based on airspeed. I've seen some all-electric trim systems incorporate a direction reversal switch that would allow a measure of control redundancy should a single direction fail. Well done on the wiring diagrams! John Read this topic online here: http://forums.matronics.com/viewtopic.php?p=500724#500724 ________________________________________________________________________________
Subject: welding cable fat wire... EPDM, PVC, Neoprene?
From: "johnbright" <john_s_bright(at)yahoo.com>
Date: Feb 17, 2021
Does anyone have knowledge as to what insulation is best for OBAM welding cable fat wires? I see choices of EPDM, PVC, and Neoprene. From what I've seen, EPDM and PVC are rated 105C and Neoprene 150F which is 66C. SAE J1127 is commonly cited but I haven't found a copy of it. Thanks -------- John Bright, RV-6A, at FWF, O-360 Z-101 single batt dual alt SDS EM-5-F. john_s_bright(at)yahoo.com, Newport News, Va https://docs.google.com/document/d/1YOtPiA3AdUsQEYR4nodBESNAo21rxdnx4pFs7VxXfuI/edit?usp=sharing Read this topic online here: http://forums.matronics.com/viewtopic.php?p=500741#500741 ________________________________________________________________________________
Date: Feb 17, 2021
From: "Robert L. Nuckolls, III" <nuckolls.bob(at)aeroelectric.com>
Subject: Re: welding cable fat wire... EPDM, PVC, Neoprene?
At 04:58 PM 2/17/2021, you wrote: > > >Does anyone have knowledge as to what insulation is best for OBAM welding >cable fat wires? I see choices of EPDM, PVC, and Neoprene. From what I've >seen, EPDM and PVC are rated 105C and Neoprene 150F which is 66C. >SAE J1127 is commonly cited but I haven't found a copy of it. I think EPDM is the most prevalent in the consumer market. Here's one example for materials sold by the foot and in colors to boot. I'd have no heartburn over this wire in the 'fat' circuits. Bob . . . Un impeachable logic: George Carlin asked, "If black boxes survive crashes, why don't they make the whole airplane out of that stuff?" ________________________________________________________________________________
Date: Feb 17, 2021
From: "Robert L. Nuckolls, III" <nuckolls.bob(at)aeroelectric.com>
Subject: Re: welding cable fat wire... EPDM, PVC, Neoprene?
SORRY, THE LINK DIDN'T PASTE . . . > I think EPDM is the most prevalent in the > consumer market. Here's one example for > materials sold by the foot and in colors > to boot. https://tinyurl.com/ycanulc8 > I'd have no heartburn over this wire in > the 'fat' circuits. Bob . . . Un impeachable logic: George Carlin asked, "If black boxes survive crashes, why don't they make the whole airplane out of that stuff?" ________________________________________________________________________________
Subject: Re: welding cable fat wire... EPDM, PVC, Neoprene?
From: "johnbright" <john_s_bright(at)yahoo.com>
Date: Feb 17, 2021
Thanks Bob! -------- John Bright, RV-6A, at FWF, O-360 Z-101 single batt dual alt SDS EM-5-F. john_s_bright(at)yahoo.com, Newport News, Va https://docs.google.com/document/d/1YOtPiA3AdUsQEYR4nodBESNAo21rxdnx4pFs7VxXfuI/edit?usp=sharing Read this topic online here: http://forums.matronics.com/viewtopic.php?p=500745#500745 ________________________________________________________________________________
Date: Feb 19, 2021
From: "Robert L. Nuckolls, III" <nuckolls.bob(at)aeroelectric.com>
Subject: Interesting
A really BIG question in the evolution of lithium battery technology and markets has focused on stresses that star-gazer's proposals will put on supplies of lithium. Here's a tid-bit that popped up this morning on a recycling effort for lithium batteries. https://tinyurl.com/y9tbzruo Bob . . . Un impeachable logic: George Carlin asked, "If black boxes survive crashes, why don't they make the whole airplane out of that stuff?" ________________________________________________________________________________
Subject: Re: Interesting
From: Steve Williams <sbw(at)sbw.org>
Date: Feb 20, 2021
That IS interesting. "The closed loop lithium-ion battery resource recovery system yields ... sustainable production of battery-grade lithium, cobalt and nickel ..." "Sustainable," eh? OK, then, what's the environmental footprint of the recycling process? How does that compare to the original mining of these materials? Also, "The process recovers 95% of all lithium-ion battery materials ..." So recycling extends the life cycle of these materials 20X (minus the recycling footprint). Significant, but not, by definition, "sustainable." I'm often left scratching my head over the framing of news about energy production and storage. Yes, charging an electric car with centrally-generated electricity produces less carbon and particulates than a car that burns its own gas. (Don't forget the life cycle costs of solar panels, wind turbines, hydroelectric dams, and transmission. Even microgrids!) But, like adding lanes to a freeway, will electric cars increase miles driven? Even if not, is it "sustainable," or does it only push the environmental collapse further in the future? What's my point? I don't actually know. Mostly, I'd like to see news coverage frame this more honestly. Instead of claiming this makes batteries "sustainable," we might say this "reduces the environmental cost by X%." Even that doesn't tell the whole story, but at least it's closer to the truth. ________________________________________________________________________________
Date: Feb 20, 2021
From: ashleysc(at)broadstripe.net
Subject: Re: Interesting
Hi Steve; In other words you would like to see scientifically accurate articles written by scientifically ignorant writers. These are the same folks who would have us afraid of the element "Carbon," as in our "Carbon Footprint." Cheers! Stu. ----- Original Message ----- From: "Steve Williams" <sbw(at)sbw.org> Sent: Saturday, February 20, 2021 6:59:49 AM Subject: Re: AeroElectric-List: Interesting That IS interesting. "The closed loop lithium-ion battery resource recovery system yields ... sustainable production of battery-grade lithium, cobalt and nickel ..." "Sustainable," eh? OK, then, what's the environmental footprint of the recycling process? How does that compare to the original mining of these materials? Also, "The process recovers 95% of all lithium-ion battery materials ..." So recycling extends the life cycle of these materials 20X (minus the recycling footprint). Significant, but not, by definition, "sustainable." I'm often left scratching my head over the framing of news about energy production and storage. Yes, charging an electric car with centrally-generated electricity produces less carbon and particulates than a car that burns its own gas. (Don't forget the life cycle costs of solar panels, wind turbines, hydroelectric dams, and transmission. Even microgrids!) But, like adding lanes to a freeway, will electric cars increase miles driven? Even if not, is it "sustainable," or does it only push the environmental collapse further in the future? What's my point? I don't actually know. Mostly, I'd like to see news coverage frame this more honestly. Instead of claiming this makes batteries "sustainable," we might say this "reduces the environmental cost by X%." Even that doesn't tell the whole story, but at least it's closer to the truth. ________________________________________________________________________________
Subject: Re: Interesting
From: Charlie England <ceengland7(at)gmail.com>
Date: Feb 20, 2021
On 2/20/2021 8:59 AM, Steve Williams wrote: > > That IS interesting. > > "The closed loop lithium-ion battery resource recovery system yields > ... sustainable production of battery-grade lithium, cobalt and nickel > ..." > > "Sustainable," eh? OK, then, what's the environmental footprint of > the recycling process? How does that compare to the original mining > of these materials? > > Also, "The process recovers 95% of all lithium-ion battery materials > ..." So recycling extends the life cycle of these materials 20X > (minus the recycling footprint). Significant, but not, by definition, > "sustainable." > > I'm often left scratching my head over the framing of news about > energy production and storage. Yes, charging an electric car with > centrally-generated electricity produces less carbon and particulates > than a car that burns its own gas. (Don't forget the life cycle costs > of solar panels, wind turbines, hydroelectric dams, and transmission. > Even microgrids!) But, like adding lanes to a freeway, will electric > cars increase miles driven? > > Even if not, is it "sustainable," or does it only push the > environmental collapse further in the future? > > What's my point? I don't actually know. Mostly, I'd like to see news > coverage frame this more honestly. Instead of claiming this makes > batteries "sustainable," we might say this "reduces the environmental > cost by X%." Even that doesn't tell the whole story, but at least > it's closer to the truth. Whenever I see posts like this, I hear, 'This stuff isn't a perfect solution, so we should just let Big Oil and Big Coal run our lives until there's no livable environment.' Why not mentally insert one word ('more'), and be happy that improvements are being made? Charlie -- This email has been checked for viruses by Avast antivirus software. https://www.avast.com/antivirus ________________________________________________________________________________
From: Janet Amtmann <jgamtmann2(at)gmail.com>
Date: Feb 20, 2021
Subject: Apollo GX65 repair?
My last post seems to have vanished into the ether. My question, can anyone lead me to a person that can repair a NOS Apollo GX65 that was DOA out of the box (COM is inoperative)? I don't want to rebuild my entire instrument panel. J=C3=BCrgen Amtmann ________________________________________________________________________________
From: "n1dw" <n1deltawhiskey(at)comcast.net>
Subject: Repair of Apollo GX65
Date: Feb 20, 2021
Jurgen, Am responding to this previous inquiry prompted by your latest. The first inquiry below gave no information as to what tests, research, etc. that was conducted to verify the radio section of this unit was not working. And no, I do not know of a viable repair facility, but I have a thought or two. You mention the GPS lights up, but COM does not work. Does the display show active and standby frequencies at the bottom of the display? If not, the first thing that came to mind is that there is no connectivity between the NAV and COM functions other than the display screen. They are basically two separately functioning radios in a single box. I.e., they each have their own power and ground connections. Is yours wired up accordingly? After verifying the power connections are getting to the radio (if not, it could be defective wiring and/or pins in the connectors), then it would be time to check the antenna connections. If none of these work, then you may indeed have a faulty unit and your best bet is to try to find a seller willing to refund your money if the unit you purchase does not function in your system. If you get the same results with a 2nd unit, then I would be highly suspect that it is not the unit, but something else. For what it is worth, I do not think the GPS in these radios should be used unless you can find a source for updating the GPS card that provides that data. So the real question is whether or not you can find a compatible unit with a working COM function. Good luck. Regards, Doug Windhorn From: owner-aeroelectric-list-server(at)matronics.com On Behalf Of Janet Amtmann Sent: Friday, February 12, 2021 1:03 PM Subject: AeroElectric-List: Repair of Apollo GX65 Does anyone know of someone that can repair an old Apollo GX 65 GPS/COM radio? I bought it new many years ago and installed it in my RV6. On power-up it did not work. No xmit or receive. The GPS window lights up and cycles thru the initial tests, so the radio is getting power properly. I get no satisfaction from Garmin who bought the line and then deleted it from stock. I did a bench test and it is not transmitting or receiving. The trays will not accommodate a different radio, I would have to rebuild the whole instrument panel, but I'm leery of purchasing a used one (they are available) and might get one with the same or other problem. At the moment we are flying with a handheld, but that's a PITA. Any information would be welcome. Jurgen Amtmann ________________________________________________________________________________
Subject: Re: Interesting
From: Kelly McMullen <kellym(at)aviating.com>
Date: Feb 20, 2021
Sorry you chose to respond like a politician. Carbon footprint is NOT about elemental carbon. It is the carbon compounds your activities generate, like carbon dioxide and others that have a major impact on greenhouse gas effect. Might as well be afraid of di-hydrogen oxide, especially when temps are below zero. On 2/20/2021 8:31 AM, ashleysc(at)broadstripe.net wrote: > > Hi Steve; > In other words you would like to see scientifically accurate articles written by scientifically ignorant writers. > These are the same folks who would have us afraid of the element "Carbon," as in our "Carbon Footprint." > Cheers! Stu. > ________________________________________________________________________________
Subject: Re: Apollo GX65 repair?
From: Emil Sr <papa11(at)centurylink.net>
Date: Feb 21, 2021
On 2/20/2021 12:04 PM, Janet Amtmann wrote: > My last post seems to have vanished into the ether. My question, can > anyone lead me to a person that can repair a NOS Apollo GX65 that was > DOA out of the box (COM is inoperative)? I don't want to rebuild my > entire instrument panel. > > Jrgen Amtmann TRY Lafayette AVIATION IN lAFAYETTE iNDIANA (PERDUE) AIRPORT PULSAR 111 N143NB ________________________________________________________________________________
Date: Feb 21, 2021
From: ashleysc(at)broadstripe.net
Subject: Re: Interesting
Hi Kelly; Just for the record, you have missed my point, which was that the actual culprits are carbon dioxide and other carbon compounds, not elemental carbon, of which most all living things are composed, including ourselves. Also, for the record, I am an a-political retired electro-mechanical engineer. Cheers! Stu. ----- Original Message ----- From: "Kelly McMullen" <kellym(at)aviating.com> Sent: Saturday, February 20, 2021 6:57:11 PM Subject: Re: AeroElectric-List: Interesting Sorry you chose to respond like a politician. Carbon footprint is NOT about elemental carbon. It is the carbon compounds your activities generate, like carbon dioxide and others that have a major impact on greenhouse gas effect. Might as well be afraid of di-hydrogen oxide, especially when temps are below zero. On 2/20/2021 8:31 AM, ashleysc(at)broadstripe.net wrote: > > Hi Steve; > In other words you would like to see scientifically accurate articles written by scientifically ignorant writers. > These are the same folks who would have us afraid of the element "Carbon," as in our "Carbon Footprint." > Cheers! Stu. > ________________________________________________________________________________
Subject: Re: Peer review, Z16 Rotax engine, Ducati Regulator
From: "velletazjp" <velletazjp(at)gmail.com>
Date: Feb 24, 2021
Hi guys Project is moving forward except one step : I cant get my hand on a crowbar over voltage ( ref OVM-14 by BANDC ) unless I buy a complete kit ( which I dont really need) Does somebody knows where I could get one (AC spruce are out of stock) or does somebody have one in spare and would accept to send it to me ? Thank you all JP -------- 2020 contribution paid Read this topic online here: http://forums.matronics.com/viewtopic.php?p=500770#500770 ________________________________________________________________________________
From: Charlie England <ceengland7(at)gmail.com>
Date: Feb 24, 2021
Subject: Re: Peer review, Z16 Rotax engine, Ducati Regulator
On Wed, Feb 24, 2021 at 2:49 AM velletazjp wrote: > velletazjp(at)gmail.com> > > Hi guys > > Project is moving forward except one step : > > I can=99t get my hand on a crowbar over voltage ( ref OVM-14 by BAN DC ) > unless I buy a complete kit ( which I don=99t really need) > > Does somebody knows where I could get one (AC spruce are out of stock) or > does somebody have one in spare and would accept to send it to me ? > > > Thank you all > > JP > If you're willing to build your own, there are instructions on the AEC website. Or I can send you a copy, if that's easier. Charlie ________________________________________________________________________________
Subject: Re: Peer review, Z16 Rotax engine, Ducati Regulator
From: "user9253" <fransew(at)gmail.com>
Date: Feb 24, 2021
Crowbar Over Voltage Circuit -------- Joe Gores Read this topic online here: http://forums.matronics.com/viewtopic.php?p=500783#500783 Attachments: http://forums.matronics.com//files/ov_using_lm431_by_bob_123.jpg ________________________________________________________________________________
Subject: Re: Peer review, Z16 Rotax engine, Ducati Regulator
From: "user9253" <fransew(at)gmail.com>
Date: Feb 24, 2021
http://www.aeroelectric.com/DIY/DIY_Crowbar_OVP_F.pdf -------- Joe Gores Read this topic online here: http://forums.matronics.com/viewtopic.php?p=500785#500785 ________________________________________________________________________________
Subject: Shielded cable
From: "RobLewis" <thesandman1031(at)gmail.com>
Date: Feb 26, 2021
Can anyone help: Whats the go-to equivalent milspec number to 22759/16 for avionics hookups, except shielded? Read this topic online here: http://forums.matronics.com/viewtopic.php?p=500796#500796 ________________________________________________________________________________
Subject: Re: Shielded cable
From: "johnbright" <john_s_bright(at)yahoo.com>
Date: Feb 26, 2021
Hi Rob, This is what I came up with for notes on my schematic: ============================================ Individual conductors: M22759/16 (SAE AS22759) Tefzel insulated. Smaller than 22 awg is not used because it's fragile. Jacketed cables (shielded or unshielded): M22759/18 (/18 wire has thinner insulaton versus /16) Tefzel insulated wires jacketed with Tefzel per M27500 (NEMA WC27500) (It ends up being type TG in M27500). ============================================ Folks refer to MIL-W-22759 as M22759. It has officially been SAE-AS22759 since 2004. Folks refer to MIL-DTL-27500 as M27500. It has officially been NEMA WC27500 since 2001. Wiremasters.net is a good source for Tefzel wire. A couple documents uploaded to this posting: "M22759 Wiremasters summary" and "M27500 Wiremasters summary". -------- John Bright, RV-6A, at FWF, O-360 Z-101 single batt dual alt SDS EM-5-F. john_s_bright(at)yahoo.com, Newport News, Va https://docs.google.com/document/d/1YOtPiA3AdUsQEYR4nodBESNAo21rxdnx4pFs7VxXfuI/edit?usp=sharing Read this topic online here: http://forums.matronics.com/viewtopic.php?p=500797#500797 Attachments: http://forums.matronics.com//files/m22759_wiremasters_summary_163.pdf http://forums.matronics.com//files/m27500_wiremasters_summary_156.pdf ________________________________________________________________________________
Subject: good price on a good switch - Carling SPST
From: "prestonkavanagh" <preston.kavanagh(at)gmail.com>
Date: Feb 26, 2021
Today I realized I have the wrong tab width on some of my switches. I was surprised to find Carling quick connect SPST at $1.76 - less than brands of inferior quality. Seemed worth sharing: https://www.zoro.com/carling-technologies-toggle-switch-spst-10a-250v-quikconnct-2fa53-73-tabs/i/G3266566/ -------- PBK3 PA-12, BD-4, RV6a, gliders Read this topic online here: http://forums.matronics.com/viewtopic.php?p=500799#500799 ________________________________________________________________________________
Date: Feb 26, 2021
From: "Robert L. Nuckolls, III" <nuckolls.bob(at)aeroelectric.com>
Subject: Re: Shielded cable
At 06:43 AM 2/26/2021, you wrote: > >Can anyone help: > >What=99s the go-to equivalent milspec number to >22759/16 for avionics hookups, except shielded? There are dozen's of suitable wire products. I have a boat-load of 22AWG wire that came off RayChem spools more years back than I can recall. Make you a deal only my brother-in-law would get . . . also have assortment of high temperature wire in larger gages. Bob . . . Un impeachable logic: George Carlin asked, "If black boxes survive crashes, why don't they make the whole airplane out of that stuff?" ________________________________________________________________________________
Date: Feb 26, 2021
From: "Robert L. Nuckolls, III" <nuckolls.bob(at)aeroelectric.com>
Subject: Re: Peer review, Z16 Rotax engine, Ducati Regulator
At 06:07 PM 2/24/2021, you wrote: > >http://www.aeroelectric.com/DIY/DIY_Crowbar_OVP_F.pdf > >-------- >Joe Gores > > I'll dig around in old inventory tomorrow. Might be able to build you one that's even adjustable from outside the enclosure . . . Bob . . . Un impeachable logic: George Carlin asked, "If black boxes survive crashes, why don't they make the whole airplane out of that stuff?" ________________________________________________________________________________
Subject: Fuse Block - Breaker Combination
From: "MFleming" <sagriver(at)icloud.com>
Date: Feb 28, 2021
RV-7 project, deep into the wiring, basically a Z101 architecture. Fuse blocks are used for the main bus and clearance delivery bus but I do have a few breakers. Flap, trim and auto pilot breakers located near the fuse blocks. The field breaker is next tho the master switch. My question is: Do I protect the 12AWG wire going from the main fuse block stud to the breaker copper bus bar. It would seem to me, to be considered part of the fat wire coming from the master contactor but I'm unsure hence the post. The field breaker will get a fusible link, so I'm covered there. -------- Michael Fleming Read this topic online here: http://forums.matronics.com/viewtopic.php?p=500820#500820 ________________________________________________________________________________
Subject: Re: Fuse Block - Breaker Combination
From: "user9253" <fransew(at)gmail.com>
Date: Feb 28, 2021
It depends. Is the wire only a few inches long? Is it well supported so that even if a terminal breaks, the wire will not fall? If that wire has mechanical protection, then electrical protection is not necessary. -------- Joe Gores Read this topic online here: http://forums.matronics.com/viewtopic.php?p=500821#500821 ________________________________________________________________________________
Subject: EssaysWriters.com Review
From: "joannawebster" <joannawebster2228(at)gmail.com>
Date: Mar 01, 2021
Students these days are looking for ways to relieve the stress of academic writing by ordering custom writing services. Given the wide selection of companies out there, it can be difficult to discern the reliable ones from the companies that make promises they cannot keep. With that in mind, we are reviewing EssaysWriters.com on https://ratewritingservices.com/review/essayswriters-com-review.html to determine whether they are worth your hard-earned money. EssaysWriters.com offers a full range of writing services. The bulk of their orders come from students who need help with their essays, dissertations, annotated


January 19, 2021 - March 01, 2021

AeroElectric-Archive.digest.vol-qb